You are on page 1of 167

TRẦN NAM DŨNG

(chủ biên)

LỜI GIẢI VÀ BÌNH LUẬN ĐỀ THI CÁC


TỈNH, CÁC TRƯỜNG ĐẠI HỌC NĂM
HỌC 2009-2010

MATHSCOPE.ORG
dddd
Lời nói đầu

Kỳ thi VMO năm nay sẽ được tổ chức vào tháng 3/2010. Hiện nay các trường và các
tỉnh đang hoàn tất việc thi HSG cấp tỉnh và thành lập đội tuyển. Sau kỳ thi học kì I,
việc luyện thi cho kỳ thi VMO 2010 sẽ được khởi động tại tất cả các địa phương.

Nhằm giúp các bạn học sinh có thêm cơ hội trao đổi, học hỏi, rèn luyện kỹ năng giải
toán, chúng tôi thực hiện cuốn sách này.

Thông qua việc giải và bình luận các đề thi học sinh giỏi các tỉnh và các trường Đại
học, chúng tôi sẽ đưa ra những bài tập tương tự, nói thêm về phương pháp sử dụng
trong bài giải nhằm giúp các bạn nhìn rộng hơn về vấn đề, để có thể áp dụng cho
những bài toán khác.

Cuốn sách được sự tham gia về chuyên môn của các thầy cô giáo chuyên toán, các
cựu IMO, VMO. Ý kiến đóng góp, bình luận có thể gửi trực tiếp qua chủ đề mà
chúng tôi mở trên Mathscope.org hoặc theo địa chỉ trannamdung@ovi.com với
tiêu đề [4VMO2010]. Các thành viên có đóng góp sẽ được tôn vinh và nhận những
quà tặng ý nghĩa.

Cuốn sách được thực hiện với sự giúp đỡ của Nokia Vietnam (http://www.nokia.
com.vn).
TP HCM, ngày 02 tháng 12 năm 2009
Trần Nam Dũng

iii
iv Trần Nam Dũng (chủ biên)
Lời cảm ơn

Xin cảm ơn sự nhiệt tình tham gia đóng góp của các bạn:

1. Võ Quốc Bá Cẩn

2. Phạm Tiến Đạt

3. Phạm Hy Hiếu

4. Tạ Minh Hoằng

5. Nguyễn Xuân Huy

6. Mai Tiến Khải

7. Hoàng Quốc Khánh

8. Nguyễn Vương Linh

9. Nguyễn Lâm Minh

10. Nguyễn Văn Năm

11. Đinh Ngọc Thạch

12. Lê Nam Trường

13. Võ Thành Văn

Cùng rất nhiều bạn yêu toán khác.

v
vi Trần Nam Dũng (chủ biên)
Mục lục

Lời nói đầu iii

Lời cảm ơn v

I Đề toán và lời giải 1

1 Số học 3
1.1 Đề bài . . . . . . . . . . . . . . . . . . . . . . . . . . . . . . . . . 3
1.2 Lời giải . . . . . . . . . . . . . . . . . . . . . . . . . . . . . . . . 5

2 Phương trình, hệ phương trình 15


2.1 Đề bài . . . . . . . . . . . . . . . . . . . . . . . . . . . . . . . . . 15
2.2 Lời giải . . . . . . . . . . . . . . . . . . . . . . . . . . . . . . . . 17

3 Bất đẳng thức và cực trị 27


3.1 Đề bài . . . . . . . . . . . . . . . . . . . . . . . . . . . . . . . . . 27
3.2 Lời giải . . . . . . . . . . . . . . . . . . . . . . . . . . . . . . . . 29

4 Phương trình hàm và đa thức 43


4.1 Đề bài . . . . . . . . . . . . . . . . . . . . . . . . . . . . . . . . . 43
4.2 Lời giải . . . . . . . . . . . . . . . . . . . . . . . . . . . . . . . . 45

5 Hình học 57
5.1 Đề bài . . . . . . . . . . . . . . . . . . . . . . . . . . . . . . . . . 57
5.2 Lời giải . . . . . . . . . . . . . . . . . . . . . . . . . . . . . . . . 60

6 Tổ hợp 71
6.1 Đề bài . . . . . . . . . . . . . . . . . . . . . . . . . . . . . . . . . 71
6.2 Lời giải . . . . . . . . . . . . . . . . . . . . . . . . . . . . . . . . 74

vii
viii Trần Nam Dũng (chủ biên)

7 Dãy số 89
7.1 Đề bài . . . . . . . . . . . . . . . . . . . . . . . . . . . . . . . . . 89
7.2 Lời giải . . . . . . . . . . . . . . . . . . . . . . . . . . . . . . . . 91

II Một số bài giảng toán 99

8 Giải phương trình hàm bằng cách lập phương trình 101

9 Dãy truy hồi loại un+1 = f (un ) 107

10 Các định lý tồn tại trong giải tích và định lý cơ bản của đại số 113

11 Phép chứng minh phản chứng 123

12 Nguyên lý Dirichlet 127

13 Cauchy-Bunyakovski-Schwarz Inequality 137

A Đề luyện đội tuyển cho kỳ thi VMO 2010 145

B Hướng dẫn nội dung bồi dưỡng học sinh thi chọn học sinh giỏi Toán
Quốc gia lớp 12 THPT 151
Phần I

Đề toán và lời giải

1
Chương 1

Số học

“Toán học là bảo vật quý giá hơn bất cứ thứ gì khác mà chúng ta được thừa hường từ kho
tàng tri thức của nhân loại.”
Rene Descartes

1.1 Đề bài
n
1.1. Giả sử m, n là hai số nguyên dương thoả mãn là số lẻ với d = (m, n). Xác
d
định (am + 1, an − 1) với a là số nguyên dương lớn hơn 1.
1.2. Dãy số {an } được xác định như sau: a0 = 0, a1 = 1, a2 = 2, a3 = 6 và

an+4 = 2an+3 + an+2 − 2an+1 − an với mọi n ≥ 0.

(a) Chứng minh rằng an chia hết cho n với mọi n ≥ 1.


n a o∞
n
(b) Chứng minh rằng dãy số chứa vô số số hạng chia hết cho 2009.
n n=1
1.3. Cho m, n là các số nguyên dương nguyên tố cùng nhau, m là số chẵn. Tìm ước
số chung lớn nhất của m2 + n2 và m3 + n3 .
1.4. Cho các số nguyên dương a, b, c, d thỏa mãn ac + bd chia hết cho a2 + b2 .
Chứng minh rằng
(c2 + d 2 , a2 + b2 ) > 1.
1.5. Tìm tất cả các số nguyên dương k sao cho phương trình

x2 + y2 + x + y = kxy

có nghiệm nguyên dương.

3
4 Trần Nam Dũng (chủ biên)

1.6. Tìm tất cả các số nguyên dương x, y thoả mãn

x2 + 15y2 + 8xy − 8x − 36y − 28 = 0.

1.7. Chứng minh rằng


|12m − 5n | ≥ 7
với mọi m, n nguyên dương.

1.8. Cho n là số nguyên dương sao cho 3n − 1 chia hết cho 22009 . Chứng minh rằng

n ≥ 22007 .
100 +100
1.9. (1) Cho a = 52 . Chứng minh số a có ít nhất 25 chữ số 0 đứng liền
nhau.

(2) Chứng minh tồn tại vô số số tự nhiên n mà 5n có ít nhất 100 chữ số 0 đứng
liền nhau.

1.10. Cho f : N∗ → N∗ thoả mãn các điều kiện

(i) f (xy) = f (x) f (y) với mọi x, y thoả mãn (x, y) = 1;

(ii) f (x + y) = f (x) + f (y) với mọi bộ số nguyên tố x, y.

Hãy tính f (2), f (3), f (2009).

1.11. Tìm tất cả các bộ số tự nhiên a, b, c, d đôi một phân biệt thỏa mãn

a2 − b2 = b2 − c2 = c2 − d 2 .

1.12. Cho hai số nguyên dương p, q lớn hơn 1, nguyên tố cùng nhau. Chứng minh
rằng tồn tại số nguyên k sao cho (pq − 1)n k + 1 là hợp số với mọi số nguyên dương
n.
Lời giải và bình luận đề thi các tỉnh, các trường Đại học năm học 2009-2010 5

1.2 Lời giải


n
Bài 1.1. Giả sử m, n là hai số nguyên dương thoả mãn là số lẻ với d = (m, n).
d
Xác định (am + 1, an − 1) với a là số nguyên dương lớn hơn 1.
(Đại học Vinh)
 
m n n 2m n
Lời giải. Do d = (m, n) nên , = 1. Vì là số lẻ nên ta có , = 1,
d d d d d
suy ra (2m, n) = d. Theo định lý Bezout, tồn tại u, v nguyên sao cho 2mu + nv = d.
Đặt D = (am + 1, an − 1). Khi đó

am ≡ −1 (mod D),

suy ra
a2m ≡ 1 (mod D).
Ngoài ra ta đã có
an ≡ 1 (mod D).
Từ những điều trên, ta suy ra

ad = a2mu+nv ≡ 1 (mod D).

Do m = dm0 nên từ đây ta suy ra am ≡ 1 (mod D). Kết hợp với am ≡ −1 (mod D) ta
suy ra 2 ≡ 0 (mod D). Từ đây suy ra D = 1 hoặc D = 2. Dễ thấy với a lẻ thì D = 2
còn với a chẵn thì D = 1. Đó chính là kết luận của bài toán.

Bình luận. Đây là một bài toán khá căn bản về bậc của một số theo modulo. Trong
các bài toán như vậy, định lý Bezout luôn là một kết quả hữu ích.

Bài 1.2. Dãy số {an } được xác định như sau: a0 = 0, a1 = 1, a2 = 2, a3 = 6 và

an+4 = 2an+3 + an+2 − 2an+1 − an với mọi n ≥ 0.

(a) Chứng minh rằng an chia hết cho n với mọi n ≥ 1.


n a o∞
n
(b) Chứng minh rằng dãy số chứa vô số số hạng chia hết cho 2009.
n n=1
(Đại học Khoa học tự nhiên)

Lời giải. Phương trình đặc trưng của dãy {an } có dạng x4 − 2x3 − x2 + 2x + 1 = 0,
tương đương (x2 − x − 1)2 = 0. Từ đó số hạng tổng quát của an có dạng

an = c1 α n + c2 β n + n(c3 α n + c4 β n ),
6 Trần Nam Dũng (chủ biên)

trong đó α > β là các nghiệm của phương trình x2 − x − 1 = 0. Từ đây, từ các điều
1 1
kiện ban đầu, ta tìm được c1 = c2 = 0, c3 = √ , c4 = − √ . Suy ra
5 5
 
1 1
an = n √ α n − √ β n .
5 5
an
Từ đây ta được = Fn , với F1 = 1, F2 = 1, Fn+1 = Fn + Fn−1 với mọi n = 1, 2, . . .
n
tức là dãy số Fibonacci. Kết luận câu (a) đến đây là hiển nhiên.

Để giải phần (b), ta có thể đi theo các hướng sau.

Cách 1. Dùng quy nạp chứng minh rằng Fm+n = Fm+1 Fn + Fm Fn−1. Sau đó tiếp tục
dùng quy nạp chứng minh rằng Fkn chia hết cho Fn . Từ đây, để chứng minh kết luận
của bài toán, ta chỉ cần chỉ ra một giá trị nguyên dương n sao cho Fn chia hết cho
2009 là xong. Có thể tính toán được rằng F56 chia hết cho 49, còn F20 chia hết cho
41, từ đó F280 chia hết cho 2009.

Cách 2. Ta chứng minh mệnh đề tổng quát: Với mọi số nguyên dương N, tồn tại vô
số số hạng của dãy số Fibonacci chia hết cho N.

Để thực hiện điều này, ta bổ sung thêm số hạng F0 = 0 cho dãy Fibonacci. Chú
ý là ta vẫn có hệ thức Fn+1 = Fn + Fn−1 với mọi n = 0, 1, 2, . . . Gọi ri là số dư
trong phép chia Fi cho N. Xét N 2 + 1 cặp số dư (r0 , r1 ), (r1 , r2 ), . . . , (rN , rN+1 ).
Do 0 ≤ ri ≤ N − 1 nên chỉ có N 2 cặp giá trị (ri , ri+1 ) khác nhau. Theo nguyên lý
Dirichlet, tồn tại cặp chỉ số i < j sao cho (ri , ri+1 ) ≡ (r j , r j+1 ). Từ đây, do rk−1
chính là số dư trong phép chia rk+1 − rk cho N nên ta suy ra ri−1 = r j−1 , ri−2 = r j−2 ,
. . . , r0 = r j−i . Suy ra dãy số dư tuần hoàn với chu kỳ j − i. Vì r0 = 0 nên rk( j−i) = 0
với mọi k = 1, 2, . . . và ta có rk( j−i) chia hết cho N với mọi k = 1, 2, . . . (đpcm).

Bình luận. Ý tưởng dùng nguyên lý Dirichlet để chứng minh tính tuần hoàn của dãy
số dư không mới. Đề thi vô địch Liên Xô trước đây có câu: Chứng minh rằng trong
dãy số Fibonacci tồn tại ít nhất một số tận cùng bằng bốn chữ số 0.

Đề thi chọn đội tuyển Việt Nam năm 2004 cũng có ý tưởng tương tự:
Cho dãy số (xn ) (n = 1, 2, 3, . . .) được xác định bởi: x1 = 603, x2 = 102 và
p
xn+2 = xn+1 + xn + 2 xn+1 xn − 2 với mọi n ≥ 1.

Chứng minh rằng


(1) Tất cả các số hạng của dãy số đã cho đều là các số nguyên dương.

(2) Tồn tại vô hạn số nguyên dương n sao cho biểu diễn thập phân của xn có bốn
chữ số tận cùng là 2003.
Lời giải và bình luận đề thi các tỉnh, các trường Đại học năm học 2009-2010 7

(3) Không tồn tại số nguyên dương n mà biểu diễn thập phân của xn có bốn chữ
số tận cùng là 2004.

Bài 1.3. Cho m, n là các số nguyên dương nguyên tố cùng nhau, m là số chẵn. Tìm
ước số chung lớn nhất của m2 + n2 và m3 + n3 .
(Đồng Nai)

Lời giải. Do (m, n) nguyên tố cùng nhau và m chẵn nên n lẻ. Đặt

d = (m2 + n2 , m3 + n3 ).

Dễ thấy d lẻ. Do m3 + n3 = (m + n)(m2 + n2 − mn) nên từ đây suy ra

d | mn(m + n).

Từ đây lại suy ra d là ước của (m + n)3 . Giả sử d > 1. Khi đó gọi p là một ước số
nguyên tố của d thì p | (m + n)3 , suy ra p | m + n. Mặt khác

(m + n)2 − (m2 + n2 ) = 2mn,

suy ra p | 2mn. Vì p lẻ nên p | mn. Vì p nguyên tố và (m, n) = 1nên từ đây suy ra


p | m hoặc p | n. Nhưng do p | m + n nên từ đây lại suy ra p | n và tương ứng là p | m.
Mâu thuẫn. Vậy điều giả sử là sai, tức là d = 1.

Bài 1.4. Cho các số nguyên dương a, b, c, d thỏa mãn ac + bd chia hết cho a2 + b2 .
Chứng minh rằng
(c2 + d 2 , a2 + b2 ) > 1.
(Đại học Sư phạm)

Lời giải. Trước hết xét trường hợp (a, b) = 1. Giả sử p là một ước nguyên tố của
a2 + b2 . Khi đó p | ac + bd. Từ đẳng thức

(ac + bd)2 + (ad − bc)2 = (a2 + b2 )(c2 + d 2 ),

ta suy ra p | ad − bc. Từ đây, ta lần lượt có

p | c(ac + bd) + d(ad − bc) = a(c2 + d 2 ),

p | d(ac + bd) − c(ad − bc) = b(c2 + d 2 ).


Vì (a, b) = 1 nên theo định lý Bezout tồn tại u, v sao cho au + bv = 1. Từ các điều
trên, ta có

p | u · a(c2 + d 2 ) + v · b(c2 + d 2 ) = (au + bv)(c2 + d 2 ) = c2 + d 2 ,

suy ra p là ước số chung của a2 + b2 và c2 + d 2 , tức là (a2 + b2 , c2 + d 2 ) > 1.


8 Trần Nam Dũng (chủ biên)

.
Bây giờ giả sử (a, b) = D > 1. Đặt a = Dx, b = Dy thì ta có Dxc+Dyd .. D2 (x2 +y2 ),
.
suy ra xc + yd .. x2 + y2 . Theo kết quả ở trên thì (x2 + y2 , c2 + d 2 ) > 1. Từ đó, một
cách hiển nhiên (D2 (x2 + y2 ), c2 + d 2 ) > 1, tức là (a2 + b2 , c2 + d 2 ) > 1.

Bài toán được giải quyết hoàn toàn.

Bình luận. Định lý Bezout mọi lúc, mọi nơi!


Bài 1.5. Tìm tất cả các số nguyên dương k sao cho phương trình

x2 + y2 + x + y = kxy (1)

có nghiệm nguyên dương.


(Phổ thông Năng khiếu)
Lời giải. Giả sử k là một giá trị sao cho phương trình (1) có nghiệm nguyên dương.
Khi đó tồn tại nghiệm (x0 , y0 ) của (1) với x0 + y0 nhỏ nhất. Không mất tính tổng
quát, có thể giả sử x0 ≥ y0 . Xét phương trình bậc hai

x2 − (ky0 − 1)x + y20 + y0 = 0. (2)

Theo giả sử ở trên thì x0 là một nghiệm của (2). Theo định lý Viet thì

y20 + y0
x1 = ky0 − 1 − x0 =
x0
cũng là một nghiệm của (2). Dễ thấy x1 là một số nguyên dương, vì thế (x1 , y0 ) cũng
là một nghiệm nguyên dương của (1). Từ giả thiết x0 + y0 nhỏ nhất ta suy ra

x1 + y0 ≥ x0 + y0 .
y20 + y0
Tức là ≥ x0 , suy ra y20 + y0 ≥ x02 . Từ đây ta có bất đẳng thức kép
x0

y20 ≤ x02 ≤ y20 + y0 < (y0 + 1)2 ,


2
suy ra x0 = y0 . Thay vào (1) ta được 2 + = k, suy ra x0 chỉ có thể bằng 1 hoặc 2,
x0
tương ứng k bằng 4 hoặc 3. Với k = 3 ta có (2, 2) là nghiệm của (1), với k = 4 ta có
(1, 1) là nghiệm của (1). Vậy k = 3 và k = 4 là tất cả các giá trị cần tìm.

Ta cũng có thể đánh giá k khác một chút, như sau.

Cách 1. Từ đẳng thức x02 + y20 + x0 + y0 = kx0 y0 , chia hai vế cho x0 , y0 , ta được
x0 y0 1 1
+ + + = k.
y0 x0 y0 x0
Lời giải và bình luận đề thi các tỉnh, các trường Đại học năm học 2009-2010 9

x0 k 1
Mặt khác, cũng theo lý luận ở trên thì ky0 − 1 − x0 ≥ x0 nên suy ra ≤ − .
y0 2 2y0
Từ đó ta có
k 1 y0 1 1 k 1 y0 1 k 5
k≤ − + + + = + + + ≤ + .
2 2y0 x0 y0 x0 2 2y0 x0 x0 2 2

Từ đó suy ra k ≤ 5. Hơn nữa k chỉ có thể bằng 5 khi x0 = y0 = 1 (trường hợp này
dẫn đến mâu thuẫn). Trường hợp k = 3 ta có nghiệm x = y = 2, k = 4 ta có nghiệm
x = y = 1. Còn với k ≤ 2 thì rõ ràng là phương trình vô nghiệm.

Cách 2. Lý luận như trên thì

y20 + y0
x0 ≤ x1 = ≤ y0 + 1.
x0

Như vậy y0 + 1 nằm ngoài hai nghiệm của tam thức f (x) = x2 − (ky0 − 1)x + y20 + y0 ,
suy ra f (y0 + 1) ≥ 0. Từ đó
2(y0 + 1) 2
k≤ = 2 + ≤ 4.
y0 y0

Bình luận. Kỹ thuật sử dụng trong lời giải trên được gọi là kỹ thuật phương trình
Markov. Kỹ thuật này hiện nay đã trở nên khá quen thuộc. Dưới đây là một số bài
toán có thể giải được bằng kỹ thuật này:

x 2 + y2
1. Chứng minh rằng nếu x, y là các số nguyên dương sao cho n = là một
xy + 1
số nguyên thì n là một số chính phương.
(IMO 1988)

2. Hãy tìm tất cả các số nguyên dương n sao cho phương trình

x + y + z + t = n xyzt

có nghiệm nguyên dương.


(VMO 2002)

Sẽ thú vị nếu chúng ta xét bài toán tìm tất cả các nghiệm của (1) khi k = 3 và k = 4.
Bài 1.6. Tìm tất cả các số nguyên dương x, y thoả mãn

x2 + 15y2 + 8xy − 8x − 36y − 28 = 0.

(Cần Thơ)
10 Trần Nam Dũng (chủ biên)

Lời giải. Biến đổi phương trình đã cho, ta viết được nó dưới dạng

(x + 4y − 4)2 − (y + 2)2 = 40,

(x + 3y − 6)(x + 5y − 2) = 40.
Do x, y là các số nguyên dương và x + 3y − 6 < x + 5y − 2 nên ta có thể phân tích
40 = 1 · 40 = 2 · 20 = 4 · 10. Đến đây ta giải từng trường hợp.

Trường hợp 1. x + 3y − 6 = 1 và x + 5y − 2 = 0. Giả ra, ta tìm được x = −45.5 và


y = 17.5, loại.

Trường hợp 2. x + 3y − 6 = 2 và x + 5y − 2 = 20. Giải ra, ta tìm được x = −13 và


y = 7, loại.

Trường hợp 3. x + 3y − 6 = 4 và x + 5y − 2 = 10. Giải ra, ta tìm được x = 7 và y = 1,


nhận.

Vậy phương trình đã cho có một nghiệm nguyên dương duy nhất là (x, y) = (7, 1).

Bài 1.7. Chứng minh rằng


|12m − 5n | ≥ 7
với mọi m, n nguyên dương.
(Hải Phòng)

Lời giải vắn tắt. Giả sử ngược lại tồn tại m, n nguyên dương sao cho |12m − 5n | < 7.
Do |12m − 5n | không chia hết cho 2, 3, 5 nên chỉ có thể xảy ra trường hợp

|12m − 5n | = 1.

+ Nếu 12m − 5n = 1 thì xét modul 4 suy ra mâu thuẫn.

+ Nếu 12m − 5n = −1 thì xét modul 6 suy ra n chẵn, sau đó xét modul 13 suy ra mâu
thuẫn.

Bài 1.8. Cho n là số nguyên dương sao cho 3n − 1 chia hết cho 22009 . Chứng minh
rằng
n ≥ 22007 .
(Bình Định)

Lời giải. Vì n nguyên dương nên ta có thể đặt n = 2k m, với k, m ∈ N, m lẻ. Ta có


 k m  k   k m−1  k m−2 k

3n − 1 = 32 − 1 = 32 − 1 32 + 32 + · · · + 32 + 1 .
Lời giải và bình luận đề thi các tỉnh, các trường Đại học năm học 2009-2010 11

 k m−1  k m−2 .
+ · · · + 32 + 1, suy ra 3n − 1 .. 22009 khi và chỉ
k
Do m lẻ nên 32 + 32
.
khi 32 − 1 .. 22009 . Từ đây suy ra k ≥ 2, và ta có phân tích
k

k
 2   k−1 
32 − 1 = (3 − 1)(3 + 1)(32 + 1) 32 + 1 · · · 32 + 1
 2   k−1 
= 23 (32 + 1) 32 + 1 · · · 32 + 1 .
i
Nhận thấy rằng 32 + 1 (i = 1, 2, . . . , k − 1) chia hết cho 2 nhưng lại không chia
k
hết cho 4. Do đó 32 − 1 chia hết cho 2k+2 nhưng không chia hết cho 2k+3 . Điều
. .
này có nghĩa là 32 − 1 .. 22009 khi và chỉ khi 2k+2 .. 22009 , tức là k ≥ 2007. Vậy
k

n ≥ 22007 m ≥ 22007 . Đó là điều phải chứng minh.

Bình luận. Từ bài toán trên, ta có thể đưa ra bài toán tổng quát: Cho số nguyên
dương n sao cho 3n − 1 chia hết cho 2k , k ∈ N, k ≥ 2. Chứng minh rằng n ≥ 2k−2
.
(hoặc cũng có thể chứng minh n .. 2k−2 ).
100 +100
Bài 1.9. (1) Cho a = 52 . Chứng minh số a có ít nhất 25 chữ số 0 đứng liền
nhau.
(2) Chứng minh tồn tại vô số số tự nhiên n mà 5n có ít nhất 100 chữ số 0 đứng
liền nhau.
(Bắc Ninh)
100
Hướng dẫn. Hãy chứng minh rằng 52 +100 − 5100 tận cùng bằng ít nhất 100 chữ
số 0 (tức là chia hết cho 10100!) và 5100 < 1075 .

Bình luận. Bài toán này kiến thức sử dụng không khó nhưng phát biểu khá đẹp và
thú vị.
Bài 1.10. Cho f : N∗ → N∗ thoả mãn các điều kiện
(i) f (xy) = f (x) f (y) với mọi x, y thoả mãn (x, y) = 1;
(ii) f (x + y) = f (x) + f (y) với mọi bộ số nguyên tố x, y.
Hãy tính f (2), f (3), f (2009).
(Ninh Bình)
Lời giải. Thay x = 2, y = 3 vào (i), ta được f (6) = f (2) f (3). Thay x = y = 3 vào
(ii), ta được f (6) = 2 f (3). Từ đây suy ra f (2) = 2. Từ đó f (4) = 2 f (2) = 4. Đặt
f (3) = a, ta lần lượt tính được
f (5) = f (3) + f (2) = a + 2,
f (7) = f (5) + f (2) = a + 4,
f (12) = f (7) + f (5) = 2a + 6.
12 Trần Nam Dũng (chủ biên)

Mặt khác f (12) = f (3) f (4) = 4a nên ta suy ra 2a + 6 = 4a, tức là a = 3. Vậy
f (3) = 3. Từ đây suy ra f (5) = 5, f (7) = 7. Ta lại có

f (11) + f (3) = f (14) = f (2) f (7) = 2 · 7 = 14,

suy ra
f (11) = f (14) − f (3) = 11.
Để tính f (2009), ta sẽ lần lượt tính f (41) và f (49). Vì 41 là số nguyên tố nên

f (41) + f (3) = f (44) = f (4) f (11) = 4 f (11) = 44,

suy ra f (41) = 41. Ta có

f (49) = f (47) + f (2) = 2 + f (47).

f (47) + f (5) = f (52) = f (4) f (13) = 4( f (11) + f (2) = 4(11 + 2) = 52,

suy ra f (47) = 47 và f (49) = 49. Cuối cùng

f (2009) = f (41) f (49) = 41 · 49 = 2009.

Bình luận. Điều đáng ngại nhất trong lời giải bài này là rất dễ nhầm vì ngộ nhận. Sẽ
thú vị nếu xét bài toán tổng quát: Chứng minh f (n) = n với mọi n nguyên dương.
Bài 1.11. Tìm tất cả các bộ số tự nhiên a, b, c, d đôi một phân biệt thỏa mãn

a2 − b2 = b2 − c2 = c2 − d 2 .

(Đại học Khoa học tự nhiên)


Lời giải. Bài toán tương đương với việc tìm một cấp số cộng thực sự gồm bốn số
chính phương. Ta chứng minh rằng không tồn tại một cấp số cộng như vậy. Giả sử
ngược lại tồn tại bốn số chính phương A2 , B2 , C2 , D2 lập thành một cấp số cộng
tăng, tức là B2 − A2 = C2 − B2 = D2 −C2 . Trong các cấp số như thế, chọn cấp số có
công sai nhỏ nhất. Ta có thể giả sử rằng các số chính phương này đôi một nguyên
tố cùng nhau, và tính chẵn lẻ của các phương trình chứng tỏ rằng mỗi một số chính
phương này phải lẻ. Như vậy tồn tại các số nguyên nguyên tố cùng nhau u, v sao cho
C2 − A2
A = u − v, C = u + v, u2 + v2 = B2 , và công sai của cấp số cộng bằng = 2uv.
2
  
2 2 D+B D−B
Ta cũng có D − B = 4uv, và có thể viết thành = uv. Hai thừa
2 2
số ở vế trái nguyên tố cùng nhau, và u và v cũng thế. Như vậy tồn tại bốn số nguyên
Lời giải và bình luận đề thi các tỉnh, các trường Đại học năm học 2009-2010 13

đôi một nguyên tố cùng nhau a, b, c, d (trong đó có đúng một số chẵn) sao cho
u = ab, v = cd, D + B = 2ac và D − B = 2bd. Từ đây suy ra B = ac − bd, và như
vậy ta có thể thế vào phương trình u2 + v2 = B2 để được (ab)2 + (cd)2 = (ac − bd)2 .
Phương trình này là đối xứng đối với bốn biến số nên ta có thể giả sử c là chẵn và a,
b, d là lẻ. Từ phương trình bậc hai này ta suy ra c là hàm hữu tỷ của căn bậc hai của
a4 − a2 d 2 + d 4 , từ đó suy ra tồn tại số nguyên lẻ m sao cho a4 − a2 d 2 + d 4 = m2 .

Vì a và d là lẻ nên tồn tại các số nguyên nguyên tố cùng nhau x và y sao cho
a2 = k(x + y) và d 2 = k(x − y), trong đó k = ±1. Thay vào phương trình nói trên, ta
được x2 + 3y2 = m2 , từ đó rõ ràng là y phải là số chẵn và x lẻ. Đổi dấu x nếu cần, ta
 y 2  m + x   m − x 
có thể giả sử m + x chia hết cho 3, ta có 3 = , từ đó suy ra
2 2 2
m+x m−x
là ba lần số chính phương còn là số chính phương. Như vậy ta có các
2 2
m+x
số nguyên nguyên tố cùng nhau r, s (một số chẵn và một số lẻ) sao cho = 3r2 ,
2
m−x
= s2 , m = 3r2 + s2 , x = 3r2 − s2 và y = ±2rs.
2

Thay x và y vào các biểu thức của a2 và d 2 (và biến đổi nếu cần) ta được a2 =
k(s + r)(s − 3r) và d 2 = k(s − r)(s + 3r). Vì các thừa số ở vế phải là nguyên tố cùng
nhau nên bốn đại lượng (s − 3r), (s − r), (s + r), (s + 3r) phải có trị tuyệt đối chính
phương, với công sai 2r. Các đại lượng này tất cả phải cùng dấu vì nếu ngược lại
thì tổng của hai số chính phương lẻ bằng hiệu của hai số chính phương lẻ, tức là,
1 + 1 ≡ 1 − 1 (mod 4), mâu thuẫn.

Vì thế, ta phải có |3r| < s, và do m = 3r2 + s2 ta có 12r2 < m. Mặt rkhác, từ phương

2
trình bậc bốn ta có m < a2 +d 2 , như vậy ta có bất đẳng thức |2r| < max{a, d} .

3
Như vậy ta có bốn số chính phương lập thành một cấp số cộng với công sai |2r| <
|2abcd|, và số cuối cùng chính là công sai của cấp số cộng ban đầu. Điều này mâu
thuẫn với cách chọn bốn số chính phương ban đầu, phép chứng minh hoàn tất.

Bình luận. Đây là một bài toán kinh điển của lý thuyết phương trình Diophant.
Fermat đề xuất toán này năm 1640 trong bức thư gửi Frenicle. Sau đó ông có nói
là chứng minh được, nhưng không ai biết về chứng minh này. Weil nói rằng Euler
đã công bố chứng minh vào năm 1780, nhưng trong chứng minh có đôi chỗ có vấn
đề, và Weil cũng nói rằng một chứng minh tốt hơn được đưa ra bởi J.Itars vào năm
1973.

Bài toán này được đưa vào đề thi e rằng là hơi quá sức, bởi trong phòng thi, nếu
không biết trước, khó lòng có thể tìm ra được lời giải trong vòng 180 (cùng với 3 bài
toán khác). Tình huống này khiến ta nhớ đến bài 3 của kỳ TST 2009.
14 Trần Nam Dũng (chủ biên)

Lời giải trên đây được lấy từ

http://mathpages.com/home/kmath044/kmath044.htm.

Bạn có thể vào link này để đọc lại chứng minh và xem thêm những thông tin thú vị
về bài toán này.

Bài 1.12. Cho hai số nguyên dương p, q lớn hơn 1, nguyên tố cùng nhau. Chứng
minh rằng tồn tại số nguyên k sao cho (pq − 1)n k + 1 là hợp số với mọi số nguyên
dương n.
(Ninh Bình)

Lời giải. Vì (p, q) = 1 nên theo định lí phần dư Trung Hoa, tồn tại số nguyên k thoả
mãn 
k ≡ 1 (mod p)
.
k ≡ −1 (mod q)
Khi đó

+ Nếu n chẵn thì (pq − 1)n ≡ 1 (mod q), suy ra (pq − 1)n k ≡ −1 (mod q), dẫn tới
.
(pq − 1)n k + 1 .. q.

+ Nếu n lẻ thì (pq − 1)n ≡ −1 (mod p), suy ra (pq − 1)n k ≡ −1 (mod p), dẫn tới
.
(pq − 1)n k + 1 .. p.

Vậy (pq − 1)n k + 1 là hợp số với mọi số nguyên dương n.


Chương 2

Phương trình, hệ phương trình

“Mọi phát kiến của nhân loại đều có bàn tay hướng dẫn của Toán học, bởi vì chúng ta
không thể có một người chỉ đường nào khác.”
Charles Darwin

2.1 Đề bài
2.1. Giải phương trình

1 2 √
 
1 2x + 1
log2 (x + 2) + x + 2 = log2 + 1+ + 2 x + 2.
2 x x
2.2. Giải phương trình
√ √ 
9 4x + 1 − 3x − 2 = x + 3.

2.3. Giải hệ phương trình  2


 x = y+a
y2 = z + a ,
 2
z = x+a
trong đó a là tham số thoả mãn điều kiện 0 < a < 1.
2.4. Giải phương trình

sin x − cos x sin3 x − cos3 x


= .
sin 3x − cos 3x sin x + cos x
2.5. Giải hệ phương trình

x2 − 2xy + x + y = 0

.
x4 − 4x2 y + 3x2 + y2 = 0

15
16 Trần Nam Dũng (chủ biên)

2.6. Giải phương trình


p
3
−2x3 + 10x2 − 17x + 8 = 2x2 5x − x3 .

2.7. (a) Tìm tất cả các giá trị của tham số thực a sao cho phương trình

a(sin 2x + 1) + 1 = (a − 3)(sin x + cos x)

có nghiệm.

(b) Phương trình 2x − 1 − x2 = 0 có bao nhiêu nghiệm số thực? Hãy giải thích.

2.8. Giải hệ phương trình

x5 + xy4 = p
y10 + y6

√ .
4x + 5 + y2 + 8 = 6

2.9. Tìm tất cả các nghiệm thực của phương trình


p
3x2 + 11x − 1 = 13 2x3 + 2x2 + x − 1.

2.10. Giải trong tập hợp các số thực hệ phương trình sau
 2009

 ∑ xi = 2009


i=1 .
2009 2009
 8 6
 ∑ i

 x = ∑ i x
i=1 i=1

2.11. Cho a, b, c là các số thực dương. Giải hệ phương trình



1
 ax − aby + = bc2

xy



1

2
abz − bc x + = a .
 zx
1



 2
 bc − az + = ab
yz

2.12. Giải hệ phương trình

9y3 (3x3 − 1) = −125



.
45x2 y + 75x = 6y2
Lời giải và bình luận đề thi các tỉnh, các trường Đại học năm học 2009-2010 17

2.2 Lời giải


Bài 2.1. Giải phương trình

1 2 √
 
1 2x + 1
log2 (x + 2) + x + 2 = log2 + 1+ + 2 x + 2. (1)
2 x x
(Đại học Vinh)
 
1
Lời giải. Điều kiện để phương trình (1) xác định là x ∈ −2, − ∪ (0, +∞). Bây
2
giờ, ta biến đổi phương trình (1) như sau

√ √ 1 2
     
1 2 2
log2 x + 2 − 2 x + 2 + x + 3 = log2 2 + − 4+ + +4+ 1+ ,
x x x x

√ √ 1 2
     
1 1
log2 x + 2 − 2 x + 2 + x + 2 = log2 2 + −2 2+ + 2+ . (2)
x x x
Xét hàm số f (t) = log2t − 2t + t 2 với t > 0. Ta có
r r
0 1 1 2
f (t) = + 2t − 2 ≥ 2 2t − 2 = 2 − 2 > 0,
t ln 2 t ln 2 ln 2

nên f (t) là hàm đồng biến với t > 0. Mặt khác, ta thấy rằng phương trình (2) có
dạng
√  
1

f x+2 = f 2+ ,
x
nên từ việc sử dụng kết quả f (t) đồng biến, ta thấy rằng nó tương đương với
√ 1
x+2 = 2+ .
x
Bình phương hai vế và thu gọn, ta viết được phương trình này thành

x3 − 2x2 − 4x − 1 = 0.
√ √
3 + 13 3 − 13
Giải ra, ta tìm được x = −1 (nhận), x = (nhận) và x = (loại). Vậy
2( √ ) 2
3 + 13
tập nghiệm của phương trình đã cho là S = −1, .
2

Bình luận. Với bài toán vừa có hàm log (hay mũ) và vừa có hàm đa thức (phân
thức) thông thường thì việc nghĩ đến dùng tính đơn điệu của hàm số để giải là điều
dễ hiểu. Bài này chỉ khó hơn đề đại học một tí.
18 Trần Nam Dũng (chủ biên)

Bài 2.2. Giải phương trình


√ √ 
9 4x + 1 − 3x − 2 = x + 3.

(Hà Nội)

2 √ √
Lời giải. Điều kiện x ≥ . Nhân hai vế của phương trình với 4x + 1 + 3x − 2,
3
ta được
√ √ 
9[(4x + 1) − (3x − 2)] = (x + 3) 4x + 1 + 3x − 2 .

Sau khi thu gọn, ta viết được phương trình này dưới dạng
√ √ 
9(x + 3) = (x + 3) 4x + 1 + 3x − 2 .

Do x + 3 > 0 nên ta có phương trình tương đương


√ √
9= 4x + 1 + 3x − 2.

Đến đây ta có thể giải bằng nhiều cách.

√ √ x+3
Cách 1. Kết hợp với phương trình 4x + 1 − 3x − 2 = để được phương trình
9
√ x + 84
4x + 1 = từ đó giải được bằng cách bình phương hai vế.
9
√ √
Cách 2. Giải phương trình 9 = 4x + 1 + 3x − 2 bằng phương pháp bình phương
liên tiếp.
√ √
Cách 3. Chú ý rằng f (x) = 4x + 1 + 3x − 2 là một hàm số tăng trên miền xác
định. Do đó phương trình f (x) = 9 có không quá một nghiệm. Nhận thấy x = 6 là
nghiệm của phương trình f (x) = 9, suy ra x = 6 là nghiệm duy nhất của phương
trình f (x) = 9, và cũng là nghiệm duy nhất của phương trình đề bài.

Bài 2.3. Giải hệ phương trình


 2
 x = y+a
y2 = z + a ,
 2
z = x+a

trong đó a là tham số thoả mãn điều kiện 0 < a < 1.


(Ninh Bình)
Lời giải và bình luận đề thi các tỉnh, các trường Đại học năm học 2009-2010 19

Lời giải. Không mất tính tổng quát, ta giả sử x = max{x, y, z}. Từ đó suy ra z2 =
max{x2 , y2 , z2 }. Đến đây, ta xét hai trường hợp.

Trường hợp 1. z ≥ 0. Trong


r trường hợp này, ta dễ dàng tìm được nghiệm của hệ đã
1 1
cho là x = y = z = + + a.
2 4
Trường hợp 2. z < 0. Nếu x ≥ 0 thì từ phương trình thứ ba ta có z2 ≥ a, suy ra

z ≤ − a < −a, dẫn tới y2 < 0, mâu thuẫn. Vậy ta phải có 0 > x ≥ y. Từ x + a ≥ z + a
ta có z2 ≥ y2 , suy ra y ≥ z (vì y, z < 0). Như thế y + a ≥ z + a, dẫn tới x2 ≥ y2 , hay
x ≤ y (vì x, y < 0). Từ đây và từ x ≥ y, ta có x = y. Thay vào hai phương trình đầu,
ta tìm được x = y = z. Với kếtrquả vừa tìm được này, thay vào hệ đã cho ta dễ dàng
1 1
giải ra được x = y = z = − + a.
2 4
r
1 1
Tóm lại, hệ phương trình đã cho có hai nghiệm là x = y = z = + + a và
r 2 4
1 1
x=y=z= − + a.
2 4
Bài 2.4. Giải phương trình
sin x − cos x sin3 x − cos3 x
= .
sin 3x − cos 3x sin x + cos x
(Đồng Nai)
Lời giải. Sử dụng hằng đẳng thức
sin 3x − cos 3x = 3 sin x − 4 sin3 x + 3 cos x − 4 cos3 x
= (sin x + cos x)[3 − 4(sin2 x − sin x cos x + cos2 x)]
= (sin x + cos x)(2 sin 2x − 1),
ta được điều kiện để phương trình có nghĩa là (sin x + cos x)(2 sin 2x − 1) 6= 0 và
trong điều kiện đó, phương trình được rút gọn lại thành
sin x − cos x = (sin3 x − cos3 x)(2 sin 2x − 1),
tương đương
(sin x − cos x)[1 − (sin2 x + sin x cos x + cos2 x)(2 sin 2x − 1)] = 0,
hay  
2 3
(sin x − cos x) − sin 2x − sin 2x + 2 = 0.
2
Từ đó giải ra được phương trình.

Bình luận. Bài này giống đề thi đại học hơn, không có ý tưởng gì.
20 Trần Nam Dũng (chủ biên)

Bài 2.5. Giải hệ phương trình


 2
x − 2xy + x + y = 0
.
x4 − 4x2 y + 3x2 + y2 = 0

(Đồng Nai)

Lời giải. Hệ phương trình đã cho có thể viết lại thành


 2
(x + y) + x(1 − 2y) = 0
.
(x2 + y)2 + 3x2 (1 − 2y) ≥ 0

Từ phương trình thứ nhất, ta tìm được x2 + y = −x(1 − 2y). Thay vào phương trình
thứ hai, ta có
x2 (1 − 2y)2 + 3x2 (1 − 2y) = 0,
hay
2x2 (1 − 2y)(2 − y) = 0,
suy ra x = 0, hoặc 1 − 2y = 0, hoặc 2 − y = 0.

+ Xét x = 0. Khi đó từ x2 + y = −x(1 − 2y), ta tìm được x = y = 0.

1
+ Xét 1 − 2y = 0. Từ đây và từ x2 + y = −x(1 − 2y), ta suy ra x2 = −y = − < 0,
2
vô nghiệm.

+ Xét 2 − y = 0. Thay vào phương trình thứ nhất, ta có x2 − 3x + 2 = 0. Giải ra tìm


được x = 1 hoặc x = 2.

Tóm lại, hệ đã cho có ba nghiệm là (x, y) = (0, 0), (1, 2) và (2, 2).

Bài 2.6. Giải phương trình


p
3
−2x3 + 10x2 − 17x + 8 = 2x2 5x − x3 .

(Bình Định)

Lời giải. Dễ thấy x = 0 không phải là nghiệm của phương trình. Chia hai vế của
phương trình cho x3 , ta được
r
10 17 8 3 5
−2 + − 2 + 3 = 2 − 1,
x x x x2
tương đương p
3
8t 3 − 17t 2 + 10t − 2 = 2 5t 2 − 1, (1)
Lời giải và bình luận đề thi các tỉnh, các trường Đại học năm học 2009-2010 21

1
với t = (t 6= 0).
x
Ta tiếp tục biến đổi phương trình (1) thành
p
3
(2t − 1)3 + 2(2t − 1) = 5t 2 − 1 + 2 5t 2 − 1.

Xét hàm số f (x) = x3 + 2x thì f 0 (x) = 3x2 + 2 > 0 nên f là một hàm số tăng trên R.
Phương trình cuối cùng có thể viết lại thành
p 
3
f (2t − 1) = f 5t 2 − 1 .

Do f là hàm số tăng nên phương trình này tương đương với


p3
2t − 1 = 5t 2 − 1,

hay
8t 3 − 12t 2 + 6t − 1 = 5t 2 − 1.
√ √
17 ± 97 17 ± 97
Giải ra ta được t = 0 (loại), t = . Tương ứng ta tìm được x = .
16 12
Bài 2.7. (a) Tìm tất cả các giá trị của tham số thực a sao cho phương trình

a(sin 2x + 1) + 1 = (a − 3)(sin x + cos x)

có nghiệm.
(b) Phương trình 2x − 1 − x2 = 0 có bao nhiêu nghiệm số thực? Hãy giải thích.
h √ √ i
Hướng dẫn. (a) Đặt t = sin x + cos x thì t ∈ − 2, 2 . Đưa bài toán về tìm a
h √ √ i
sao cho phương trình at 2 + (3 − a)t + 1 = 0 có nghiệm t ∈ − 2, 2 . Có thể
3t + 1
giải bằng phương pháp tam thức bậc hai hoặc khảo sát hàm số y = trên đoạn
h √ √ i t − t2
− 2, 2 . Đáp số: a ≤ 1 hoặc a ≥ 9.

(b) Đặt f (x) = 2x − 1 − x2 thì f 00 (x) = 2x ln2 2 − 2. Phương trình 00 (x) = 0 có một
nghiệm thực, suy ra f 0 (x) có không quá hai nghiệm thực và f (x) có không quá ba
nghiệm thực. Mặt khác, ta có 0, 1 là nghiệm của f (x), ngoài ra f (4) = −1, f (5) = 6
nên f (x) có một nghiệm nữa nằm giữa 4 và 5. Suy ra số nghiệm thực của phương
trình là 3.
Bài 2.8. Giải hệ phương trình
 5
x + xy4 = py10 + y6 (1)

4x + 5 + y2 + 8 = 6 (2)

(Đồng Nai)
22 Trần Nam Dũng (chủ biên)

Lời giải. Nếu y = 0 thì từ phương trình (1) suy ra x = 0, và phương trình (2) không
được thoả mãn. Vậy y 6= 0. Chia hai vế của phương trình (1) cho y5 , ta được
 5
x x
+ = y5 + y. (3)
y y
Xét hàm số f (x) = x5 + x, ta có f 0 (x) =
5x4+ 1 > 0, suy ra f là hàm số tăng trên R.
x
Phương trình (3) có thể viết lại thành f = f (y) và do f là hàm tăng nên tương
y
x
đương với = y, suy ra x = y2 . Thay vào phương trình (2), ta được
y
√ √
4x + 5 + x + 8 = 6. (4)
Giải ra ta được x = 1 là nghiệm duy nhất của phương trình (4). Từ đó hệ ban đầu có
nghiệm duy nhất (x, y) = (1, 1) và (x, y) = (1, −1).

Ghi chú. Tham khảo thêm lời giải bài 2.2 (Hà Nội) và bài 2.6 (Bình Định).
Bài 2.9. Tìm tất cả các nghiệm thực của phương trình
p
3x2 + 11x − 1 = 13 2x3 + 2x2 + x − 1.
(Cần Thơ)
Bài 2.10. Giải trong tập hợp các số thực hệ phương trình sau
 2009

 ∑ xi = 2009


i=1 .
2009 2009
 8 6
 ∑ xi = ∑ xi


i=1 i=1

(Đại học Sư phạm)


Lời giải. Giả sử (x1 , x2 , . . . , x2009 ) là một nghiệm của hệ. Áp dụng bất đẳng thức
Cauchy-Schwarz, ta có
!2
2009 2009
2009 ∑ xi2 ≥ ∑ xi ,
i=1 i=1
suy ra
2009
∑ xi2 ≥ 2009. (1)
i=1

Bây giờ áp dụng bất đẳng thức Chebyshev cho các bộ số (x12 , x22 , . . . , x2009
2 ) và
6 6 6
(x1 , x2 , . . . , x2009 ) được sắp thứ tự như nhau, ta có
! !
2009 2009 2009
∑ xi2 ∑ xi6 ≤ 2009 ∑ xi8 . (2)
i=1 i=1 i=1
Lời giải và bình luận đề thi các tỉnh, các trường Đại học năm học 2009-2010 23

Từ (1) và (2) ta suy ra


2009 2009
∑ xi8 ≥ ∑ xi6 . (3)
i=1 i=1

Từ phương trình thứ hai của hệ đã cho ta suy ra dấu bằng xảy ra ở (3), tức là dấu
bằng phải xảy ra ở (1) và ở (2), tức là ta phải có tất cả các xi bằng nhau. Từ đó suy
ra tất cả các xi bằng 1. Vậy x1 = x2 = · · · = x2009 = 1 là nghiệm duy nhất của hệ
phương trình.
Bài 2.11. Cho a, b, c là các số thực dương. Giải hệ phương trình

1

 ax − aby + = bc2
xy



1

2
abz − bc x + = a .
 zx
1



 2
 bc − az + = ab
yz
(Phổ thông Năng khiếu)
Hướng dẫn. Viết hệ dưới dạng

1

 Ax − By + = C
xy



1

Bz −Cx + = A .
 zx
1


 C − Az + = B


yz
rồi giải hệ tìm A, B, C theo x, y, z (phương pháp giải theo tham số).
Bài 2.12. Giải hệ phương trình
 3 3
9y (3x − 1) = −125
.
45x2 y + 75x = 6y2

(Đồng Tháp)
Lời giải. Cách 1. Hệ phương trình đã cho tương đương với

27x3 y3 + 125 = 9y3



.
45x2 y + 75x = 6y2
3
Từ phương trình thứ nhất suy ra y 6= 0. Nhân hai vế của phương trình thứ hai với y,
2
ta được
27x3 y3 + 125 = 9y3
(
135 2 2 225 .
x y + xy = 9y3
2 2
24 Trần Nam Dũng (chủ biên)

Trong hệ hai phương trình này, trừ tương ứng vế theo vế cho ta
135 225
27(xy)3 − (xy)2 − xy + 125 = 0.
2 2
5 10 5
Giải ra ta tìm được xy = − , hoặc xy = , hoặc xy = . Thay vào phương trình
3 3 6
5
thứ nhất ở trên, ta tìm được tương ứng y = 0 (loại), y = 5 và y = . Từ đó suy ra các
2
2 1
nghiệm x tương ứng là x = và x = .
3 3
   
2 1 5
Tóm lại, hệ phương trình đã cho có hai nghiệm là (x, y) = ,5 , , .
3 3 2

Cách 2. Viết lại hệ phương trình dưới dạng

27x3 y3 + 125 = 9y3



.
45x2 y + 75x = 6y2

Từ phương trình thứ nhất suy ra y 6= 0. Do đó hệ trên có thể được viết dưới dạng
tương đương là 
125
 27x3 + 3 = 9


y
2 ,
45x 75x
+ 2 =6



y y
hay   3
5
 (3x)3 +

 =9
 y  .
5 5
 3x · · 3x + =6


y y
5
Đặt a = 3x và b = . Thế thì ta có
y

a3 + b3 = 9

.
ab(a + b) = 6

Bằng một chút biến đổi đơn giản, dễ thấy hệ này tương đương với

a+b = 3
.
ab = 2

2
Giải ra ta tìm được a = 2, b = 1 (tương ứng với x = , y = 5) hoặc a = 1, b = 2
3
1 5
(tương ứng với x = , y = ).
3 2
Lời giải và bình luận đề thi các tỉnh, các trường Đại học năm học 2009-2010 25

   
2 1 5
Vậy hệ phương trình đã cho có hai nghiệm là (x, y) = ,5 , , .
3 3 2

Bình luận. Thực chất bài hệ trên xuất phát từ đề thi đề nghị 30-4 lần thứ XIV của
trường THPT chuyên Lương Thế Vinh, Đồng Nai: Giải hệ phương trình
 3 3
8x y + 27 = 18y3
.
4x2 y + 6x = y2
26 Trần Nam Dũng (chủ biên)
Chương 3

Bất đẳng thức và cực trị

“Đừng quá lo lắng về những khó khăn bạn gặp phải trong toán học. Tôi dám chắc tôi còn
gặp nhiều khó khăn hơn bạn.”
Albert Einstein

3.1 Đề bài
3.1. Cho hai số thực x, y thỏa mãn 1 ≥ x ≥ y ≥ 0. Chứng minh rằng

x3 y2 + y3 + x2
≥ xy.
x 2 + y2 + 1

3.2. Cho a, b, c là ba số thực cho trước thỏa mãn D = ac − b2 > 0 và hàm số


f (x, y) = ax2 + 2bxy + cy2 . Chứng minh rằng tồn tại hai số nguyên u, v không đồng
thời bằng 0 sao cho r
D
| f (x, y)| ≤ 2 .
3
3.3. Tìm điều kiện của số dương k sao cho với mọi a, b, c dương thỏa mãn abc = 1,
ta luôn có
1 1 1 3
+ + ≥ .
ak (b + c) bk (c + a) ck (a + b) 2
3.4. Cho ba số thực a, b, c thỏa mãn

a < b < c, a + b + c = 0, ab + bc + ca = −3.

Chứng minh rằng

−2 < abc < 2 và − 2 < a < −1 < b < 1 < c < 2.

27
28 Trần Nam Dũng (chủ biên)

3.5. Cho hình chóp S.ABCD có đáy ABCD là hình thoi cạnh bằng a và SA = SB =
SC = a, trong đó a là một số thực dương cho trước.

(1) Chứng minh rằng SD < 3a.

(2) Xác định độ dài cạnh SD theo a để khối chóp S.ABCD có thể tích lớn nhất.
3.6. Cho tam giác ABC có BC = a, CA = b, AB = c. M là một điểm tuỳ ý bên trong
tam giác ABC. Chứng minh rằng
MB · MC MC · MA MA · MB
+ + ≥ 1.
bc ca ab
3.7. Cho hình chóp tứ giác đều S.ABCD. Gọi R và r lần lượt là bán kính hình cầu
R
ngoại tiếp và nội tiếp của nó. Tìm giá trị nhỏ nhất của .
r
3.8. Cho a, b, c là các số thực dương. Chứng minh rằng
 2  2  2
a b c 1
+ + ≥ .
a + 2b b + 2c c + 2a 3

3.9. Với ba số dương x, y, z, ta kí hiệu M là số lớn nhất trong ba số


   
x 1 1
ln z + ln + 1 , ln + ln(xyz + 1), ln y + ln +1 .
yz z xyz

Tìm giá trị nhỏ nhất có thể của M khi x, y, z thay đổi.
3.10. Cho x, y, z là ba số thực không âm có tổng bằng 3. Chứng minh rằng

x2 + y2 + z2 + xyz ≥ 4.

3.11. Cho các số thực dương a, b, c thỏa abc = 1. Chứng minh rằng
a b c
+ + ≤ 1.
a2 + 2 b2 + 2 c2 + 2
3x
3.12. (1) Xác định giá trị nhỏ nhất của hàm số y = x3/2 − trên (0, +∞).
2
(2) Cho các số dương a, b, c. Chứng minh rằng
r r r
a3 b3 c3 a b c
+ + ≥ + + .
b3 c3 a3 b c a
3.13. Giả sử phương trình x4 + ax3 + bx2 + cx + 1 = 0 có ít nhất một nghiệm thực.
Tìm giá trị nhỏ nhất của a2 + b2 + c2 .
Lời giải và bình luận đề thi các tỉnh, các trường Đại học năm học 2009-2010 29

3.2 Lời giải


Bài 3.1. Cho hai số thực x, y thỏa mãn 1 ≥ x ≥ y ≥ 0. Chứng minh rằng
x3 y2 + y3 + x2
≥ xy.
x 2 + y2 + 1
(Hà Nội)

Lời giải. Sử dụng bất đẳng thức AM-GM, ta có x3 y2 + y3 ≥ 2xy2 xy. Do đó, ta chỉ
cần chứng minh được

2y2 xy + x ≥ y(x2 + y2 + 1),
tương đương

(x − y) + 2y2 xy − y(x2 + y2 ) ≥ 0.
Do (1 − x2 )(x − y) ≥ 0 nên x − y ≥ x2 (x − y). Vậy ta chỉ cần chứng minh

x2 (x − y) + 2y2 xy − y(x2 + y2 ) ≥ 0.

Bất đẳng thức này được viết lại như sau



x2 (x − y) + y2 (x + y) − y(x2 + y2 ) ≥ y2 (x + y − 2 xy) ,

hay
√ √ 2
x(x − y)2 ≥ y2 x− y .
Ta có
√ √ 2 √ √ 2 √ √ 2 √ √ 2
x(x − y)2 = x x+ y x − y ≥ x 2 x − y ≥ y2 x − y ,

nên bất đẳng thức cuối đúng. Đẳng thức xảy ra khi x = y = 1 và x = y = 0.

Bình luận. Lời giải trên sử dụng kỹ thuật "thuần nhất hóa", một kỹ thuật rất hữu
hiệu đối với các dạng bài toán không thuần nhất. Chẳng hạn, bạn đọc có thể thử sử
dụng kỹ thuật này để giải hai bài toán sau:
1. Cho a, b, c là các số thực dương. Chứng minh rằng

a2 + b2 + c2 + 2abc + 1 ≥ 2(ab + bc + ca).

2. Cho ba số thực dương a, b, c. Chứng minh rằng

2(a2 + b2 + c2 ) + abc + 8 ≥ 5(a + b + c).

Một điều thú vị là sử dụng kết quả hai bài toán này, ta có thể giải bài toán thi APMO
2004: Cho a, b, c là các số thực dương. Khi đó

(a2 + 2)(b2 + 2)(c2 + 2) ≥ 9(ab + bc + ca).


30 Trần Nam Dũng (chủ biên)

Bài 3.2. Cho a, b, c là ba số thực cho trước thỏa mãn D = ac − b2 > 0 và hàm số
f (x, y) = ax2 + 2bxy + cy2 . Chứng minh rằng tồn tại hai số nguyên u, v không đồng
thời bằng 0 sao cho r
D
| f (u, v)| ≤ 2 .
3
(Đại học Khoa học tự nhiên)
Lời giải. Để ý rằng | − f (x, y)| = | f (x, y)| nên ta chỉ cần xét a ≥ 0 là được. Kết hợp
với giả thiết D = ac − b2 > 0, ta có a > 0 và c > 0. Lúc này ta có

by 2 D 2
 
f (x, y) = a x + + y ≥ 0.
a a

Do đó yêu cầu của bài toán tương đương với việc chỉ ra cặp số nguyên (u, v) không
đồng thời bằng 0 sao cho r
D
f (u, v) ≤ 2 .
3
Gọi A = {(x, y) | x ∈ Z, y ∈ Z, x2 + y2 > 0}. Thế thì ta có f (x, y) > 0 với mọi
(x, y) ∈ A. Gọi (u, v) ∈ A sao cho f (u, v) = min f (x, y) và đặt K = f (u, v),
(x, y)∈A
K > 0. Rõ ràng u, v phải là hai số nguyên tố cùng nhau, do đó tồn tại hai số nguyên
s, t sao cho
us − vt = 1. (∗)
Đặt g(x, y) = f (ux + ty, vx + sy), ta có

min g(x, y) = min f (ux + ty, vx + sy) = f (u, v) = K


(x, y)∈A (x, y)∈A

(đạt được tại x = 1 và y = 0). Đặt L = uta + vsc + btv + usb, sau một vài tính toán
đơn giản ta viết được

g(x, y) = f (ux + ty, vx + sy) = f (u, v) · x2 + 2Lxy + f (t, s) · y2 .

Từ (∗), ta có
K · f (t, s) = f (u, v) · f (t, s) = L2 + D,
suy ra
Ly 2 Dy2
 
g(x, y) = K x + + .
K K
Bây giờ, ta chọn số n như sau
    
L L 1
nếu ≤


2

K K 
n=   .
L L 1
+ 1 nếu >


2

K K
Lời giải và bình luận đề thi các tỉnh, các trường Đại học năm học 2009-2010 31


L 1
Dễ thấy n − ≤ , do đó
K 2

L 2 D K D
 
K ≤ g(n, −1) = K n − + ≤ + ,
K K 4 K
r
4D D
suy ra K 2 ≤ , hay K ≤ 2 . Đó là điều phải chứng minh.
3 3
Bài 3.3. Tìm điều kiện của số dương k sao cho với mọi a, b, c dương thỏa mãn
abc = 1, ta luôn có
1 1 1 3
+ + ≥ .
ak (b + c) bk (c + a) ck (a + b) 2
(Bắc Ninh)
1
Lời giải. Xét t > 0, cho a = b = t và c = . Bất đẳng thức đã cho trở thành
t2
1 2k−1 2t 2−k 3
t + 3 ≥ .
2 t +1 2
1
Nếu k < thì cho t → +∞, vế trái tiến dần về 0, trong khi vế phải là hằng số và lớn
2
1
hơn 0, mâu thuẫn. Do đó ta phải có k ≥ .
2
1
Tiếp theo, ta sẽ chứng minh k ≥ 2. Thật vậy, giả sử ≤ k < 2, cho t → 0, vế trái
2
tiến dần về 0, trong khi đó vế phải là hằng số là lớn hơn 0, mâu thuẫn. Vậy ta phải
có k ≥ 2.
Bây giờ ta sẽ chứng minh với k ≥ 2 thì bất đẳng thức đã cho đúng. Sử dụng bất đẳng
thức Chebyshev cho hai bộ đơn điều cùng chiều
   
1 1 1 1 1 1
, , và , , ,
ak−2 bk−2 ck−2 a2 (b + c) b2 (c + a) c2 (a + b)
ta được   
1 1 1 1
∑ ak (b + c) ≥ 3 ∑ ak−2 ∑ a2 (b + c) .
Mặt khác, theo bất đẳng thức AM-GM thì
1 1 1 3
+ + ≥√ = 3.
ak−2 bk−2 ck−2 3 k−2 k−2 k−2
a b c
Kết hợp với trên, ta có
1 1 1 1 1 1
+ + ≥ + + .
ak (b + c) bk (c + a) ck (a + b) a2 (b + c) b2 (c + a) c2 (a + b)
32 Trần Nam Dũng (chủ biên)

Do đó, ta chỉ cần chứng minh được


1 1 1 3
+ + ≥ ,
a2 (b + c) b2 (c + a) c2 (a + b) 2

tương đương
bc ca ab 3
+ + ≥ .
a(b + c) b(c + a) c(a + b) 2
Đây chính là bất đẳng thức Nesbitt được áp dụng cho ba số dương bc, ca, ab. Tóm
lại, ta có k ≥ 2 là tập hợp các giá trị cần tìm.

Bình luận. Có thể thấy bài này cho ta một kết quả tổng quát cho bài toán thi IMO
1995: Nếu a, b, c là ba số thực dương thỏa mãn abc = 1 thì
1 1 1 3
+ + ≥ .
a3 (b + c) b3 (c + a) c3 (a + b) 2

Mặt khác khi thay lũy thừa k ở từng phân số bởi a, b, c ta được một bất đẳng thức
thú vị với chiều ngược lại là
1 1 1 3
+ + ≤ .
aa (b + c) bb (c + a) cc (a + b) 2

Chứng minh bất đẳng thức này bằng cách dùng bất đẳng thức Bernoulli.

Bài 3.4. Cho ba số thực a, b, c thỏa mãn

a < b < c, a + b + c = 0, ab + bc + ca = −3.

Chứng minh rằng

−2 < abc < 2 và − 2 < a < −1 < b < 1 < c < 2.

(Ninh Bình)

Lời giải. Xét hàm số

f (x) = (x − a)(x − b)(x − c) = x3 − (a + b + c)x2 + (ab + bc + ca)x − abc


= x3 − 3x − abc.

Ta có f 0 (x) = 3(x2 − 1), f 0 (x) có hai nghiệm x = −1 và x = 1. Do f (x) có ba nghiệm


phân biên biệt a < b < c nên ta phải có −1 < b < 1 và f (−1) f (1) < 0. Mà f (−1) =
2 − abc, f (1) = −(2 + abc), nên −(2 − abc)(2 + abc) < 0, hay −2 < abc < 2. Lại
có f (−2) = −(2+abc) = f (1) và f (2) = 2−abc = f (−1), suy ra f (−2) f (−1) < 0
và f (1) f (2) < 0. Điều này này có nghĩa là −2 < a < −1 và 1 < c < 2.
Lời giải và bình luận đề thi các tỉnh, các trường Đại học năm học 2009-2010 33

Tóm lại, ta có

−2 < abc < 2 và − 2 < a < −1 < b < 1 < c < 2.

Đó là điều phải chứng minh.

Bình luận. Bài này bản chất chính là bài chọn đội tuyển Anh năm 1995: Cho a, b, c
là các số thực thỏa mãn đồng thời các tính chất a < b < c, a + b + c = 6 và ab +
bc + ca = 9. Chứng minh rằng

0 < a < 1 < b < 3 < c < 4.

Nhưng đã bị che giấu đi, người ra đề đã sử dụng phép đặt đổi biến (a, b, c) 7→
(a + 2, b + 2 c + 2) rồi biến đổi để ra được bài "Ninh Bình" trên.

Bài 3.5. Cho hình chóp S.ABCD có đáy ABCD là hình thoi cạnh bằng a và SA =
SB = SC = a, trong đó a là một số thực dương cho trước.

(1) Chứng minh rằng SD < 3a.

(2) Xác định độ dài cạnh SD theo a để khối chóp S.ABCD có thể tích lớn nhất.

(Đồng Nai)

Lời giải. d
S

B C

A D

(1) Xét chóp S.ABC có các cạnh bên bằng nhau nên SH là đường cao của chóp với H
là tâm đường tròn ngoại tiếp tam giác ABC. Do AB = BC nên H thuộc đường trung
trực BD của AC, suy ra AC⊥(SBD).

Tương tự với chóp A.BSD có các cạnh bên bằng nhau mà AO⊥(SBD) nên O là tâm
đường tròn ngoại tiếp tam giác SBD. Vì OB = OD nên tam giác SBD vuông tại S.
Từ đó ta tính được
BD2 = SB2 + SD2 = a2 + SD2 ,
34 Trần Nam Dũng (chủ biên)

suy ra

BD2 a2 + SD2 3a2 − SD2


AO2 = AB2 − BO2 = AB2 − = a2 − = .
4 4 4

Do AO2 > 0 nên ta phải có SD2 < 3a2 , hay SD < 3a.

(2) Theo trên, ta có AC = 2AO = 3a2 − SD2 . Do tam giác SBD vuông tại S có SH
là đường cao nên
2SSBD SB · SD a · SD
SH = = = .
BD BD BD
Do vậy
1 1 1 p
VS.ABCD = SH · SABCD = SH · BD · AC = a · SD · 3a2 − SD2 .
3 6 6
Theo bất đẳng thức AM-GM thì
p SD2 + (3a2 − SD2 ) 3a2
SD · 3a2 − SD2 ≤ = ,
2 2
suy ra
a3
VS.ABCD ≤ .
4

3
Đẳng thức xảy ra khi và chỉ khi SD2 = 3a2 − SD2 , hay SD = a. Vậy thể tích hình
√ 2
3
chóp S.ABCD đạt giá trị lớn nhất khi SD = a.
2
Bài 3.6. Cho tam giác ABC có BC = a, CA = b, AB = c. M là một điểm tuỳ ý bên
trong tam giác ABC. Chứng minh rằng
MB · MC MC · MA MA · MB
+ + ≥ 1.
bc ca ab
(Bình Định)
Lời giải. Trên mặt phẳng phức, giả sử các đỉnh A, B, C của tam giác ABC lần lượt
có tọa vị là u, v, w. Giả sử tọa vị của M là x. Thế thì ta có a = |v − w|, b = |w − u|,
c = |u − v|, MA = |x − u|, MB = |x − v| và MC = |x − w|. Do đó bất đẳng thức cần
chứng minh trở thành
|x − v||x − w| |x − w||x − u| |x − u||x − v|
+ + ≥ 1,
|u − v||u − w| |v − w||v − u| |w − u||w − v|
hay
(x − v)(x − w) (x − w)(x − u) (x − u)(x − v)
(u − v)(u − w) + (v − w)(v − u) + (w − u)(w − v) ≥ 1.

Lời giải và bình luận đề thi các tỉnh, các trường Đại học năm học 2009-2010 35

Sử dụng bất đẳng thức cơ bản |m| + |n| + |p| ≥ |m + n + p|, ta có



(x − v)(x − w) (x − w)(x − u) (x − u)(x − v)
(u − v)(u − w) (v − w)(v − u) (w − u)(w − v) ≥
+ +

(x − v)(x − w) (x − w)(x − u) (x − u)(x − v)
≥ + + .
(u − v)(u − w) (v − w)(v − u) (w − u)(w − v)

Mặt khác, dễ dàng kiểm tra được

(x − v)(x − w) (x − w)(x − u) (x − u)(x − v)


+ + = 1,
(u − v)(u − w) (v − w)(v − u) (w − u)(w − v)

nên từ trên ta có ngay điều phải chứng minh.

Bình luận. Đây là một bài bất đẳng thức hay ứng dụng số phức. Bất đẳng thức này
sẽ rất khó chứng minh nếu chỉ dùng các kiến thức của hình học phẳng sơ cấp.

Bài 3.7. Cho hình chóp tứ giác đều S.ABCD. Gọi R và r lần lượt là bán kính hình
R
cầu ngoại tiếp và nội tiếp của nó. Tìm giá trị nhỏ nhất của .
r
(Bình Định)

Lời giải. Gọi h là chiều cao của hình chóp và a là độ dài của cạnh hình vuông đáy.
Ta dễ dàng tính được

a2 + 2h2 a a2 + 4h2 − a2
R= , r= .
4h 4h
Do đó
R a2 + 2h2
= √ .
r a a2 + 4h2 − a2
Sử dụng bất đẳng thức AM-GM, ta có
√  √ 
p 2 + 1 a2 + 2 − 1 (a2 + 4h2 )
a a2 h + 4h2 − a2 ≤ − a2
√  2
= 2 − 1 (a2 + 2h2 ),

suy ra
a2 + 2h2 1 √
√ ≥√ = 2 + 1.
a a2 + 4h2 − a2 2−1
√  √ 
Đẳng thức xảy ra khi và chỉ khi 2 + 1 a2 = 2 − 1 (a2 + 4h2 ), hay a2 =
√  R √
2 2 − 1 h2 . Vậy giá trị nhỏ nhất của là 2 + 1.
r
36 Trần Nam Dũng (chủ biên)

Bài 3.8. Cho a, b, c là các số thực dương. Chứng minh rằng


 2  2  2
a b c 1
+ + ≥ .
a + 2b b + 2c c + 2a 3
(Đại học Sư phạm)

Lời giải. Cách 1. Sử dụng bất đẳng thức Cauchy-Schwarz, ta có

(a + 2b)2 ≤ 3(a2 + 2b2 ).

Do đó, ta chỉ cần chứng minh được

a2 b2 c2
+ + ≥ 1.
a2 + 2b2 b2 + 2c2 c2 + 2a2
Sử dụng bất đẳng thức Cauchy-Schwarz một lần nữa, ta được

a2 b2 c2 (a2 + b2 + c2 )2
+ + ≥ = 1.
a2 + 2b2 b2 + 2c2 c2 + 2a2 a2 (a2 + 2b2 ) + b2 (b2 + 2c2 ) + c2 (c2 + 2a2 )

Bài toán được chứng minh xong. Dễ thấy đẳng thức chỉ xảy ra khi a = b = c.

Cách 2. Ta sẽ chứng minh bất đẳng thức chặt hơn là


 2  2  2
a b c 18abc
+ + + ≥ 1.
a + 2b b + 2c c + 2a (a + 2b)(b + 2c)(c + 2a)
a b c
Đặt x = ,y= và c = . Khi đó ta có
a + 2b b + 2c c + 2a
2b 1 2c 1 2c 1
= − 1, = − 1, = − 1,
a x b y a z
suy ra    
1 1 1
8= −1 −1 −1 ,
x y z
hay
9xyz = 1 − (x + y + z) + xy + yz + zx.
Bất đẳng thức cần chứng minh trở thành

x2 + y2 + z2 + 18xyz ≥ 1.

Thay 18xyz = 2 − 2(x + y + z) + 2(xy + yz + zx) vào, ta viết được bất đẳng thức này
thành
x2 + y2 + z2 − 2(x + y + z) + 2(xy + yz + zx) + 1 ≥ 0,
Lời giải và bình luận đề thi các tỉnh, các trường Đại học năm học 2009-2010 37

hay
(x + y + z − 1)2 ≥ 0.

Bình luận. Bất đẳng thức đã cho vẫn đúng khi thay giả thiết a, b, c > 0 bởi giả thiết
(a + 2b)(b + 2c)(c + 2a) 6= 0.
Bài 3.9. Với ba số dương x, y, z, ta kí hiệu M là số lớn nhất trong ba số
   
x 1 1
ln z + ln + 1 , ln + ln(xyz + 1), ln y + ln +1 .
yz z xyz

Tìm giá trị nhỏ nhất có thể của M khi x, y, z thay đổi.
(Đại học Sư phạm)
Lời giải. Từ định nghĩa của số M, ta có
   
x 1
M ≥ ln z + ln +1 , M ≥ ln y + ln +1 ,
yz xyz
suy ra
   
x 1 (x + yz)(xyz + 1)
2M ≥ ln z + ln + 1 + ln y + ln + 1 = ln .
yz xyz xyz

Mặt khác, theo bất đẳng thức AM-GM thì


(x + yz)(xyz + 1)
≥ 4,
xyz

suy ra 2M ≥ ln 4, hay M ≥ ln 2. Dễ thấy đẳng thức xảy ra khi x = y = z = 1 nên ta


có kết luận min M = ln 2.
Bài 3.10. Cho x, y, z là ba số thực không âm có tổng bằng 3. Chứng minh rằng

x2 + y2 + z2 + xyz ≥ 4.

(Lương Thế Vinh, Đồng Nai)


Lời giải. Ta biết rằng trong ba số x − 1, y − 1 và z − 1 luôn có hai số cùng dấu với
nhau. Giả sử hai số đó là x − 1 và y − 1, thế thì ta có z(x − 1)(y − 1) ≥ 0, suy ra

xyz ≥ z(x + y − 1) = z(2 − z).

Mặt khác, theo bất đẳng thức Cauchy-Schwarz thì

(x + y)2 (3 − z)2
x 2 + y2 ≥ = .
2 2
38 Trần Nam Dũng (chủ biên)

Do vậy, ta chỉ cần chứng minh được bất đẳng thức sau là đủ

(3 − z)2
+ z2 + z(2 − z) ≥ 4.
2
1
Biến đổi tương đương ta được (z − 1)2 ≥ 0, hiển nhiên đúng. Ta có đẳng thức xảy
2
ra khi và chỉ khi x = y = z = 1.
Bài 3.11. Cho các số thực dương a, b, c thỏa abc = 1. Chứng minh rằng
a b c
+ + ≤ 1.
a2 + 2 b2 + 2 c2 + 2
(Kon Tum)
Lời giải. Sử dụng bất đẳng thức AM-GM, ta có
a a 1 1
≤ = − .
a2 + 2 2a + 1 2 2(2a + 1)

Do đó, ta chỉ cần chứng minh


     
1 1 1 1 1 1
− + − + − ≤ 1,
2 2(2a + 1) 2 2(2b + 1) 2 2(2c + 1)
hay
1 1 1
+ + ≥ 1. (∗)
2a + 1 2b + 1 2c + 1
Đến đây có nhiều cách đánh giá.

Cách 1. Giả sử c = min{a, b, c}, ta có ab ≥ 1. Sau đó sử dụng bất đẳng thức quen
thuộc
1 1 2
+ ≥√ ∀x, y > 0, xy ≥ 1
x+1 y+1 xy + 1
cho x = 2a, y = 2b, ta có
1 1 2
+ ≥ √ .
2a + 1 2b + 1 2 ab + 1

Vậy ta chỉ cần chứng minh được


2 1
√ + ≥ 1.
2 ab + 1 2c + 1
√ 1
Công việc này khá đơn, ta chỉ cần đặt ẩn t = ab rồi thay c = 2 vào rồi biến đổi
t
tương đương là được.
Lời giải và bình luận đề thi các tỉnh, các trường Đại học năm học 2009-2010 39

Cách 2. Đặt a = x3 , b = y3 và c = z3 , ta có
1 1 xyz yz
= 3 = 3 = ,
2a + 1 2x + 1 2x + xyz yz + 2x2

suy ra bất đẳng thức (∗) tương đương với


yz zx xy
2
+ 2
+ ≥ 1.
yz + 2x zx + 2y xy + 2z2

Sử dụng bất đẳng thức Cauchy-Schwarz, ta được

yz zx xy (yz + zx + xy)2
+ + ≥ = 1.
yz + 2x2 zx + 2y2 xy + 2z2 yz(yz + 2x2 ) + zx(zx + 2y2 ) + xy(xy + 2z2 )

Đẳng thức xảy ra khi a = b = c = 1.

Cách 3. Quy đồng và rút gọn, ta có bất đẳng thức (∗) tương đương

(2a + 1)(2b + 1) + (2b + 1)(2c + 1) + (2c + 1)(2a + 1) ≥ (2a + 1)(2b + 1)(2c + 1),

hay
a + b + c ≥ 3.
Bất đẳng thức này đúng theo AM-GM.

Bình luận. Một bài toán thú vị được đặt ra từ bài toán này, đó là: Tìm tất cả các giá
trị của k để bất đẳng thức sau
a b c 3
+ + ≤
a2 + k b2 + k c2 + k 1+k
đúng với mọi a, b, c dương thỏa mãn abc = 1.
Đây là một bài toán khó. Hiện đã chứng minh được bất đẳng thức này đúng cho
1
≤ k ≤ 5 (với lời giải sơ cấp), còn giá trị tốt nhất của k là một số rất lẻ.
5

Nói riêng về bất đẳng thức (∗), nó chính là một trường hợp riêng của bài toán chọn
đội tuyển Romania năm 1999: Cho a1 , a2 , . . . , an là các số thực dương thỏa mãn
a1 a2 · · · an = 1. Khi đó
1 1 1
+ +···+ ≤ 1.
a1 + n − 1 a2 + n − 1 an + n − 1
1 1 1
Trong bất đẳng thức này, cho n = 3 và thay lần lượt a1 , a2 , a3 bởi , , , ta sẽ thu
a b c
được (∗).
40 Trần Nam Dũng (chủ biên)

3x
Bài 3.12. (1) Xác định giá trị nhỏ nhất của hàm số y = x3/2 − trên (0, +∞).
2
(2) Cho các số dương a, b, c. Chứng minh rằng
r r r
a3 b3 c3 a b c
3
+ 3
+ ≥ + + .
b c a3 b c a
(Cần Thơ)
Lời giải. (1) Áp dụng bất đẳng thức AM-GM, ta có

x3/2 + x3/2 + 1 ≥ 3x.

Từ đó suy ra
3x 1
y = x3/2 − ≥− .
2 2
Đẳng thức xảy ra khi x = 1.

(2) Sử dụng kết quả câu (1), ta có


r r r
a3 3 a 1 b3 3 b 1 c3 3 c 1
3
≥ · − , ≥ · − , ≥ · − .
b 2 b 2 c3 2 c 2 a3 2 a 2
Cộng lại ta được
r r r
a3 b3 c3
 
3 a b c 3
+ + ≥ + + − .
b3 c3 a 3 2 b c a 2

Do đó ta chỉ cần chứng minh


 
3 a b c 3 a b c
+ + − ≥ + + .
2 b c a 2 b c a

Bất đẳng thức này tương đương với


a b c
+ + ≥ 3,
b c a
hiển nhiên đúng theo AM-GM. Ta có đẳng thức xảy ra khi và chỉ khi a = b = c.

Bình luận. Đây là một dạng bài cũ và không có ý tưởng gì mới mẻ. Hơn nữa, việc
cho thêm câu (1) lại càng làm cho nó trở nên tầm thường hơn, vì kết quả câu (1) đã
gợi mở ngay đường đi nước bước của câu (2).
Bài 3.13. Giả sử phương trình x4 + ax3 + bx2 + cx + 1 = 0 có ít nhất một nghiệm
thực. Tìm giá trị nhỏ nhất của a2 + b2 + c2 .
(Phổ thông Năng khiếu)
Lời giải và bình luận đề thi các tỉnh, các trường Đại học năm học 2009-2010 41

Lời giải. Gọi x0 là nghiệm của phương trình đã cho. Dễ thấy x0 6= 0, từ đó suy ra
 
1 c
b = − x02 + 2 + ax0 + .
x0 x0

Do đó
c 2
 
2 2 2 2 22 1
a + b + c = a + c + x0 + 2 + ax0 + .
x0 x0
Sử dụng bất đẳng thức Cauchy-Schwarz, ta được
"  #" #
c 2 1 2
  
2 2 2 1 2
a + c + x0 + 2 + ax0 + (−x0 ) + − +1 ≥
x0 x0 x0
c 2
 
c 2 1
≥ −ax0 − + x0 + 2 + ax0 +
x0 x0 x0
 2
1
= x02 + 2 ,
x0
suy ra
1 2
 
2
x0 + 2
2 2 2 x0
a +b +c ≥ .
1
x02 + 2 + 1
x0
Mặt khác dễ dàng chứng minh được

1 2
 
2
x0 + 2
x0 4
≥ ,
1 3
x02 + 2 + 1
x0

với dấu bằng đạt được tại x0 = ±1, nên


4
a2 + b2 + c2 ≥ .
3
Đẳng thức xảy ra khi và chỉ khi

a c 1 c

 = = x02 + 2 + ax0 +

 −x0 1 x0 x0



x0

.
 x0 ± 1 
 

 2 1 c
 b = − x0 + 2 + ax0 +


x0 x0
42 Trần Nam Dũng (chủ biên)

2 2 2
Giải hệ này ta tìm được a = b = c = − (ứng với x0 = 1) và a = c = , b = −
3 3 3
2 2 2 4
(ứng với x0 = −1). Vậy giá trị nhỏ nhất của a + b + c là .
3

Bình luận. Có thể thấy rằng bài toán này được lấy ý tưởng từ bài toán Tournament
of the Towns 1993: Nếu a, b là các số thực sao cho phương trình x4 + ax3 + 2x2 +
bx + 1 = 0 có ít nhất một nghiệm thực thì a2 + b2 ≥ 8.
Chương 4

Phương trình hàm và đa thức

“Toán học có cội rễ sâu xa trong đời sống hàng ngày và là nền tảng của mọi tiến bộ kĩ
thuật.”
N. A. Court

4.1 Đề bài
4.1. Tìm tất cả các hàm liên tục f : R+ → R+ thỏa mãn

f ( f (xy) − xy) + x f (y) + y f (x) = f (xy) + f (x) f (y) ∀x, y ∈ R+ .

4.2. Tìm tất cả các hàm f : R+ → R+ sao cho


f (xy)
f (x3 + y) = f 3 (x) + ∀x, y ∈ R+ .
f (x)

4.3. Tìm tất cả các hàm liên tục f : R → R thỏa mãn


(i) f đơn ánh;

(ii) f (2x − f (x)) = x;

(iii) Tồn tại x0 sao cho f (x0 ) = x0 .


4.4. Tìm tất cả các hàm f liên tục trên R thỏa mãn

f (x) = f (1 − cos x) ∀x ∈ R.

4.5. Tìm tất cả các hàm số f xác định trên R thoả

f (x + y) + f (x − y) = 2 f (x) cos y ∀x, y ∈ R.

43
44 Trần Nam Dũng (chủ biên)

4.6. Tìm tất cả các đa thức P(x, y) có hệ số thực thỏa mãn

P(x, y) = P(x + y, y − x) ∀x, y ∈ R.

4.7. Tìm tất cả các hàm f : N → N thỏa mãn điều kiện

f ( f (n)) = 3n ∀n ∈ N.

4.8. Giả sử đa thức P(x) với hệ số thực thỏa mãn điều kiện

P(n) = 12010 + 22010 + · · · + n2010 ∀n ∈ N∗ .


 
1
Tính P − .
2

4.9. Cho đa thức f (x) = x4 −2011x3 +(2010+a)x2 −2009x +a, với a là số nguyên.
Chứng minh rằng đa thức này không thể có hai nghiệm nguyên (phân biệt hay trùng
nhau).

4.10. Tìm đa thức P(x) thỏa mãn điều kiện P(1) = 2010 và

(x − y)P(x + y) − (x + y)P(x − y) = 4xy(x2 − y2 ) ∀x, y ∈ R.

4.11. Tìm đa thức f (x) khác đa thức không, có hệ số hữu tỉ và có bậc nhỏ nhất thỏa
mãn √ √  √ √
3 3 3
f 3 + 9 = 4 3 + 9.

4.12. Tìm tất cả các hàm số f (x) liên tục trên R thỏa mãn

f (x + y) + f (xy) = f (x) + f (y) + f (x) f (y) ∀x, y ∈ R.

4.13. Tìm tất cả các hàm số f : N∗ → N∗ thoả mãn đồng thời các điều kiện sau đây

(i) f (m) > f (n) với mọi m > n thuộc N∗ ;

(ii) f ( f (n)) = 4n + 9 với mọi n thuộc N∗ ;

(iii) f ( f (n) − n) = 2n + 9 với mọi n thuộc N∗ .

4.14. Tìm tất cả các hàm số f : Q → R thỏa mãn

f (x + y) = f (x) f (y) − f (xy) + 1 ∀x, y ∈ Q.


Lời giải và bình luận đề thi các tỉnh, các trường Đại học năm học 2009-2010 45

4.2 Lời giải


Bài 4.1. Tìm tất cả các hàm liên tục f : R+ → R+ thỏa mãn

f ( f (xy) − xy) + x f (y) + y f (x) = f (xy) + f (x) f (y) ∀x, y ∈ R+ .

(Đại học Vinh)


Lời giải. Chọn y = 1 ta được f [(x) − x] + x f (1) + f (x) = f (x) + f (x) f (1). Từ đây
suy ra
f [ f (x) − x] = f (1)[ f (x) − x].
Phương trình đầu có thể viết lại thành

f (1)[ f (xy) − xy] = f (xy) − xy + [ f (x) − x][ f (y) − y].

Khi đó đặt g(x) = f (x) − x ta được f (1)g(xy) = g(xy) + g(x)g(y) hay

g(1)g(xy) = g(x)g(y). (1)

Cũng từ phương trình đầu ta suy ra được f (x) > x và như vậy g(x) > 0. Vì f (x) liên
tục nên g(x) cũng liên tục. Thay x = eu , y = ev với u, v thuộc R và đặt g(et ) = h(t)
thì từ (1) ta suy ra
h(0)h(u + v) = h(u)h(v) (2)
với mọi u, v thuộc R, trong đó h(u) > 0 với mọi u thuộc R và h cũng là hàm liên tục
trên R. Tiếp tục đặt k(x) = ln h(x) thì từ (2) ta suy ra

k(0) + k(u + v) = k(u) + k(v) (3)

với mọi u, v thuộc R và k cũng là hàm liên tục. Từ đây dễ dàng suy ra (sử dụng
phương trình hàm Cauchy cho hàm số m(x) = k(x) − k(0)) k(x) = ax + b với a, b là
các hằng số thực. Như vậy

h(x) = eax+b , g(x) = g(eln x ) = ea ln x+b = eb (xa ).

Cuối cùng f (x) = x + Cxa trong đó C là một hằng số dương và a là một hằng số
thực.

Bình luận. Bài toán này là một bài toán khá căn bản. Điểm mấu chốt là đưa được
về phương trình (1). Kỹ thuật giải các phương trình hàm dạng (1) có thể tham khảo
thêm ở cuốn Phương trình hàm của GS Nguyễn Văn Mậu.
Bài 4.2. Tìm tất cả các hàm f : R+ → R+ sao cho
f (xy)
f (x3 + y) = f 3 (x) + ∀x, y ∈ R+ .
f (x)
(Đại học Khoa học tự nhiên)
46 Trần Nam Dũng (chủ biên)

Lời giải. Chọn y = 1 ta được

f (x + 1) = f 3 (x) + 1. (1)

Chọn x = 1 ta được
f (y)
f (y + 1) = f 3 (1) + . (2)
f (1)
Đặt f (1) = a thì sử dụng (1) ta lần lượt tính được

f (2) = a3 + 1, f (9) = f (23 + 1) = (a3 + 1)3 + 1. (3)

Mặt khác, sử dụng (2) ta có


f (2) 1
f (3) = a3 + = a3 + a2 + ,
a a
f (3) 1
f (4) = a3 + = a3 + a2 + a + 2 ,
a a
1
f (5) = a3 + a2 + a + 1 + 3 , . . . ,
a
3 2 1 1 1 1 1
f (9) = a + a + a + 1 + + 2 + 3 + 4 + 7 . (4)
a a a a a
Từ (3) và (4) ta suy ra
1 1 1 1 1
(a3 + 1)3 + 1 = a3 + a2 + a + 1 + + 2 + 3 + 4 + 7 .
a a a a a
Quy đồng và phân tích nhân tử ta được phương trình này tương đương với

(a − 1)(a3 − a + 1)g(a) = 0,

trong đó

g(a) = a12 + a11 + 2a10 + 4a9 + 5a8 + 6a7 + 7a6 + 6a5 + 5a4 + 4a3 + 3a2 + 2a + 1.

Giải ra ta tìm được a = 1. Vậy ta có f (x + 1) = f (x) + 1 và f (x3 ) = f 3 (x). Từ đây


p
suy ra f (x + n) = f (x) + n với mọi n nguyên dương. Với r = thuộc Q+ , ta tính
q
f ((r + q2 )3 ) bằng hai cách như sau

Một mặt
f ((r + q2 )3 ) = f 3 (r + q2 ) = [ f (r) + q2 ]3 .
Mặt khác
p2 2
 
p 4
f ((r + q ) ) = f r + 3 · 2 · q + 3 · · q + q = f (r3 + 3p2 + 3pq3 + q6 )
2 3 3 6
q q
= f (r3 ) + 3p2 + 3pq3 + q6 = f 3 (r) + 3p2 + 3pq3 + q6 .
Lời giải và bình luận đề thi các tỉnh, các trường Đại học năm học 2009-2010 47

Từ đây ra được phương trình

q2 f 2 (r) + q4 f (r) = p2 + pq3 .


p
Giải phương trình này, với chú ý f (r) > 0, ta được f (r) = = r. Vậy f (r) = r với
q
mọi r thuộc Q+ . Bây giờ để ý rằng với mọi x, y > 0 ta có
 √  √ √
3 √
3
  f ( 3 xy) f ( 3 xy)
f (x + y) = f 3
x +y = f 3
x + √ = f (x) + √ > f (x),
f ( 3 x) f ( 3 x)

suy ra f (x) là hàm tăng trên R+ . Cuối cùng, với mỗi x thuộc R+ , xét dãy số hữu tỷ
{un } tăng và dần về x (suy ra un ≤ x), ta có f (x) ≥ f (un ) = un . Chuyển sang giới
hạn, ta được
f (x) ≥ x.

Tương tự, xét dãy số hữu tỷ {vn } giảm và tiến dần về x (suy ra vn ≥ x), khi đó ta có
f (x) ≤ f (vn ) = vn . Chuyển sang giới hạn, ta được

f (x) ≤ x.

Kết hợp lại, ta được f (x) = x với mọi x thuộc R+ . Vậy f (x) = x với mọi x thuộc R+
là nghiệm duy nhất của bài toán.

Bình luận. Cái khó khăn và “gian nan” nhất của bài toán là tính f (1), khi tính f (1)
xong rồi thì bài toán trở nên vô cùng đơn giản. Để tính f (1) ta đưa về giải phương
trình nhận f (1) làm nghiệm bằng cách tính f (9) theo hai cách dựa vào (1) và (2).

Sau khi tính được f (1) = 1 ta dễ dàng chứng minh được f (x) = x trên Q+ và từ f (x)
đồng biến, ta có f (x) = x với mọi số thực dương x.

Xem thêm bài giảng “giải phương trình hàm bằng cách lập phương trình” ở cùng tài
liệu này.

Bài 4.3. Tìm tất cả các hàm liên tục f : R → R thỏa mãn

(i) f đơn ánh;

(ii) f (2x − f (x)) = x;

(iii) Tồn tại x0 sao cho f (x0 ) = x0 .

(Đại học Khoa học tự nhiên)


48 Trần Nam Dũng (chủ biên)

Gợi ý. Chúng ta liên hệ bài toán này với một bài toán sau: “Tìm g(x) liên tục thỏa
mãn điều kiện g(x + g(x)) = g(x)”. Chỉ cần đặt g(x) = x − f (x) ta sẽ được bài toán
này.

Dễ dàng chứng minh quy nạp được rằng g(x + ng(x)) = g(x). Nếu tồn tại g(x) khác
g(y) ta có thể chọn y trên khoảng x, x + g(x). Và tính liên tục được thể dùng một
cách linh hoạt ở chỗ tồn tại một đường thẳng x − ny = c với n tự nhiên, c tùy ý thỏa
mãn nó cắt đồ thị trong khoảng x, x + g(x) tại hai điểm a và b, như vậy ta được
a − n f (a) = c, b − n f (b) = c, suy ra f (c) = f (a) = f (b), vô lí vì hai điểm cắt nhau
là phân biệt.
Bài 4.4. Tìm tất cả các hàm f liên tục trên R thỏa mãn
f (x) = f (1 − cos x) ∀x ∈ R.
(Bắc Ninh)
Gợi ý. Bài toán này có cách làm quen thuộc là, xét dãy {an } trong đó a0 tùy ý và
an+1 = 1 − cos an . Ta chứng minh dãy này hội tụ đến 0. Từ đó áp dụng tính liên tục
ta suy ra f (x) = const.
Bài 4.5. Tìm tất cả các hàm số f xác định trên R thoả
f (x + y) + f (x − y) = 2 f (x) cos y ∀x, y ∈ R.
(Bình Định)
π π
Lời giải. Thay x bởi x + , y bởi , ta được f (x + π) + f (x) = 0. Từ đó suy ra
2 2
f (−x + π) = − f (−x).
Thay x bởi 0, y bởi x, ta được
f (x) + f (−x) = 2 f (0) cos x.
Từ đây kết hợp với trên ta có
f (π − x) = − f (−x) = f (x) − 2 f (0) cos x.
π π
Bây giờ, thay x bởi , y bởi x − , ta được
2 2
π   π π 
f (x) + f (π − x) = 2 f cos x − = 2f sin x.
2 2 2
Thay f (π − x) = f (x) − 2 f (0) cos x vào, ta tìm được
π 
f (x) = f (0) cos x + f sin x.
2
Thay vào thử lại thấy thoả mãn. Vậy f (x) = a cos x + b sin x với a, b là các hằng số
thực bất kì là tất cả các hàm số thoả mãn điều kiện đề bài.
Lời giải và bình luận đề thi các tỉnh, các trường Đại học năm học 2009-2010 49

Bài 4.6. Tìm tất cả các đa thức P(x, y) có hệ số thực thỏa mãn

P(x, y) = P(x + y, y − x) ∀x, y ∈ R.

(Hải Phòng)
Lời giải. Với mọi x, y ∈ R, ta có

P(x, y) = P(x + y, y − x) = P((x + y) + (y − x), (y − x) − (x + y)) = P(2y, −2x).

Từ đó suy ra

P(x, y) = P(2y, −2x) = P(2(−2x), −2(2y)) = P(−4x, −4y)


= P(2(−4y), −2(−4x)) = P(−8y, 8x) = P(2(8x), −2(−8y))
= P(16x, 16y).

Đến đây, tiến hành quy nạp, ta tìm được

P(x, y) = P(16−n x, 16−n y) ∀n ∈ N.

Cho n tiến đến vô cùng ta được P(x, y) = P(0, 0) = const. Vậy P(x, y) = a với a là
một hằng số bất kì, chính là đáp số của bài toán.
Bài 4.7. Tìm tất cả các hàm f : N → N thỏa mãn điều kiện

f ( f (n)) = 3n ∀n ∈ N.

(Đại học Sư phạm)


Gợi ý. Ý tưởng là xây dựng hàm. Dễ thấy f là đơn ánh. Từ f ( f (n)) = 3n, ta suy
ra f (3n) = f ( f ( f (n))) = 3 f (n). Từ đây cho n = 0 suy ra f (0) = 0. Với mọi n > 0
thì cũng từ f ( f (n)) = 3n ta suy ra f (n) 6= n. Từ đẳng thức f (3n) = 3 f (n) ta suy ra
nếu n = 3k m thì f (n) = 3k f (m). Như vậy hàm số f sẽ hoàn toàn xác định nếu ta tìm
được f (m) với m không chia hết cho 3.
Bài 4.8. Giả sử đa thức P(x) với hệ số thực thỏa mãn điều kiện

P(n) = 12010 + 22010 + · · · + n2010 ∀n ∈ N∗ .


 
1
Tính P − .
2
(Đại học Sư phạm)
Lời giải vắn tắt. Ý tưởng là dùng chia hết ta chứng minh rằng tồn tại vô hạn n để
P(n) chia hết cho 2n + 1. Thật vậy chỉ cần chọn 2n + 1 = p với p nguyên tố, ta chứng
minh
p − 1 2k ..
 
A = 12k + 22k + · · · + . p.
2
50 Trần Nam Dũng (chủ biên)

p + 1 2k
 
. .
Đặt B = + · · · + (p − 1)2k . Dễ thấy A − B .. p. Ta chứng minh A + B .. p.
2
Ta có ta có nhận xét nếu {1, 2, 3, . . . , p − 1} là hệ thặng dư thu gọn môđun p thì
{a, 2a, . . . , (p − 1)a} cũng là hệ thặng dư thu gọn môđun p (với (a, p) = 1). Như
.
vậy A + B ≡ (A + B)a2k (mod p) và ta chỉ cần chọn a thỏa mãn a2k − 1 6 .. p. Việc này
có thể chọn  với p > 2k(ví dụchọn a là căn nguyên thuỷ môđun
 dễ dàng  p).Như vậy
p − 1 .. 1 .. 1
ta có P . p. Do đó P − . p. Chọn p đủ lớn ta suy ra P − = 0.
2 2 2

Bài 4.9. Cho đa thức f (x) = x4 − 2011x3 + (2010 + a)x2 − 2009x + a, với a là số
nguyên. Chứng minh rằng đa thức này không thể có hai nghiệm nguyên (phân biệt
hay trùng nhau).
(Phú Yên)

Lời giải. Giả sử x0 là nghiệm nguyên của f (x), ta có f (x0 ) = 0, f (1) = 2a − 2009
là số lẻ. Suy ra f (x0 ) − f (1) là số lẻ. Mặt khác, theo định lý Bezout, f (x0 ) − f (1)
chia hết cho x0 − 1 nên x0 − 1 là số lẻ. Suy ra x0 là số chẵn.

Giả sử ngược lại, đa thức f (x) có hai nghiệm x1 , x2 . Ta xét hai trường hợp.

Trường hợp 1. f (x) có hai nghiệm nguyên x1 , x2 với x1 6= x2 . Khi đó

f (x1 ) − f (x2 )
0=
x1 − x2
= x1 + x12 x2 + x1 x22 + x23 − 2011(x12 + x1 x2 + x22 ) + (2010 + a)(x1 + x2 ) − 2009.
3

Mâu thuẫn vì vế phải là một số lẻ, do x1 , x2 đều chẵn.

Trường hợp 2. f (x) có hai nghiệm nguyên x1 = x2 . Khi đó ta có f 0 (x1 ) = f 0 (x2 ) = 0.


Nhưng điều này không xảy ra vì f 0 (x1 ) = 4x13 − 6033x12 + 2(2010 + a)x1 − 2009 là
số lẻ, do x1 chẵn.

Vậy điều giả sử là sai. Bài toán được chứng minh.

Bình luận. Trong tuyển tập đề thi Olympic 30/4 năm 2007 có đề toán này với f (x) =
x4 − 2007x3 + (2006 + a)x2 − 2005x + a. Rõ ràng cả hai bài toán này đều là trường
hợp đặc biệt của một bài toán tổng quát hơn: Nếu f (x) là một đa thức với hệ số
nguyên sao cho f (1) lẻ và f 0 (0) lẻ thì f (x) không thể có hai nghiệm nguyên.

Bài 4.10. Tìm đa thức P(x) thỏa mãn điều kiện P(1) = 2010 và

(x − y)P(x + y) − (x + y)P(x − y) = 4xy(x2 − y2 ) ∀x, y ∈ R.

(Cần Thơ)
Lời giải và bình luận đề thi các tỉnh, các trường Đại học năm học 2009-2010 51

Lời giải. Thay y = x ta được P(0) = 0. Đặt P(x) = xQ(x) và thay vào phương trình
thì ta được
Q(x + y) − Q(x − y) = 4xy.
Thay y = x ta được
Q(2x) − Q(0) = 4x2 .
Từ đó suy ra Q(x) = x2 +Q(0). Do Q(1) = 1·Q(1) = P(1) = 2010 nên Q(0) = 2009.
Vậy Q(x) = x2 + 2009 và P(x) = x(x2 + 2009).

Bài 4.11. Tìm đa thức f (x) khác đa thức không, có hệ số hữu tỉ và có bậc nhỏ nhất
thỏa mãn √ √  √ √
3 3 3
f 3 + 9 = 4 3 + 9.

(Ninh Bình)

Lời giải. Trước hết, ta sẽ chứng minh bổ đề sau.


√ √
Bổ đề. Nếu a, b, c là các số hữu tỉ thỏa mãn a 3 9 + b 3 3 + c = 0 thì a = b = c = 0.
√ √ √
Chứng minh. Giả sử a 6= 0. Do a 3 9 + b 3 3 + c = 0 nên ta có 3 3 là một nghiệm của
đa thức f (x) = ax2 + bx + c.

Với g(x) = x3 − 3, tồn tại duy nhất cặp đa thức h(x), r(x) ∈ Q[x] thỏa mãn g(x) =
f (x)h(x) + r(x) và deg r(x) < deg f (x). Vì g(x) bất khả quy nên deg r(x) = 1 hay
r(x) = cx + d với c, d ∈ Q, c 6= 0.
√ √
Mà 3 3 cũng là một nghiệm của g(x) nên 3 3 cũng là nghiệm của r(x), suy ra
√ √ d √
c 3 3 + d = 0, hay 3 3 = − . Từ đây suy ra 3 3 là số hữu tỉ, mâu thuẫn. Vậy ta phải
c
có a = 0. Tương tự suy ra a = b = c = 0.

1
Trở lại bài toán, giả sử tồn tại f (x) thỏa mãn đề bài, xét F(x) = [ f (x2 + x) − x2 ].
√  √ 4
Ta có F(x) ∈ Q[x] và F 3 3 = 3.

Với G(x) = x3 − 3, tồn tại duy nhất cặp đa thức H(x),


√ R(x)√∈ Q[x] thỏa mãn F(x) =
3
G(x)H(x) + R(x) và 0 < deg R(x) < 3. Dễ thấy R 3 = 3.

Giả sử R(x) = ax2 + bx + c với a, b, c ∈ Q. Thế thì ta có


√3
√3

a 9 + b 3 + c = 3.

Lấy bình phương hai vế rồi thu gọn, ta được


√ √
(b2 + 2ac) 9 + (3a2 + 2bc) 3 + 6ab + c2 = 3.
3 3
52 Trần Nam Dũng (chủ biên)

Sử dụng kết quả bổ đề trên, ta có


 2
 b + 2ac = 0
3a2 + 2bc = 0 ,
6ab + c2 = 3

suy ra
 
a=b√ =0
 ( c=± 3

 a 6= 0 .

b √
=± 3
a

Điều này có nghĩa là 3 ∈ Q, mâu thuẫn.

Vậy không tồn tại đa thức f (x) thỏa mãn yêu cầu của đề bài.
√ √ n √ √
(Chú ý có thể lí luận như sau: Với mọi n ∈ N thì 3
9 + 3 3 = a 3 9 + b 3 3 + c, với
a, b, c ∈ Q.

Giả sử tồn tại f (x) √


thỏa mãn
√ đề bài, khi
√ đó theo tính chất trên ta thấy rằng tồn tại a,
b, c ∈ Q sao cho a 3 9 + b 3 3 + c = 3. Đến đây ta tiến hành như trong lời giải ở
trên.)

Bài 4.12. Tìm tất cả các hàm số f (x) liên tục trên R thỏa mãn

f (x + y) + f (xy) = f (x) + f (y) + f (x) f (y) ∀x, y ∈ R.

(Ninh Bình)

Bình luận. Bài 12 và bài 14 thực sự là tương đương với nhau (Ở bài 14, ta đặt
f (x) = 1 + g(x) thì sẽ có g(x + y) + g(xy) = g(x)g(y) + g(x) + g(y). Dưới đây ta
trình bày lời giải một bài toán khó hơn cả hai bài trên. Bài toán này tương đương với
bài toán Indian MO 2003:

Bài toán. Tìm tất cả các hàm số f : R → R thoả mãn điều kiện

f (x + y) = f (x) f (y) − f (xy) + 1 (1)

với x, y thuộc R.

Lời giải. Cho y = 1 vào phương trình (1) thì ta được

f (x + 1) = f (1) f (x) − f (x) + 1.


Lời giải và bình luận đề thi các tỉnh, các trường Đại học năm học 2009-2010 53

Từ đó, nếu đặt f (1) = a thì ta lần lượt có

f (2) = a · a − 2 + 1 = a2 − a + 1,
f (3) = a(a2 − a + 1) − (a2 − a + 1) + 1 = a3 − 2a2 + 2a,
f (4) = a(a3 − 2a2 + 2a) − (a3 − 2a2 + 2a) + 1 = a4 − 3a3 + 4a2 − 2a + 1.

Mặt khác, nếu thay x = y = 2 vào (1) thì ta được

f (4) = f (2) f (2) − f (4) + 1.

Từ đó ta được phương trình

2(a4 − 3a3 + 4a2 − 2a + 1) = (a2 − a + 1)2 + 1.

Giải phương trình này ta tìm được a = 0, hoặc a = 1, hoặc a = 2. Như vậy ta có 3
trường hợp để xét.

Trường hợp 1. a = 0. Khi đó f (x + 1) = 1 − f (x). Từ đó f (0) = 0, f (−1) = 0. Thay


y = −1 vào (1), ta được f (x − 1) = 1 − f (−x). Thay x = −1, y = x vào (1), ta được
f (1 − x) = 1 − f (−x). Suy ra f (x − 1) = f (1 − x), tức f là hàm chẵn. Thay y = −x
vào (1), ta được 1 = f (x) f (−x) − f (−x2 ) + 1, suy ra f 2 (x) − f (x2 ) = 0. Cuối cùng,
1
thay y = x vào (1), ta được f (2x) = f 2 (x) − f (x2 ) + 1 = 1. Thay x = vào đẳng
2
thức này, ta suy ra mâu thuẫn. Vậy a 6= 0.

Trường hợp 2. a = 1. Khi đó f (x + 1) = 1 với mọi x, tức là f (x) = 1 với mọi x.

Trường hợp 3. a = 2. Khi đó ta có f (x + 1) = f (x) + 1, từ đó suy ra f (−1) = 0.


Thay y = −1 thì ta được f (x − 1) = 1 − f (−x), suy ra

f (−x) = 1 − f (x − 1) = 1 − [ f (x) − 1] = 2 − f (x).

Thay y bởi −y vào (1), ta được

f (x − y) = f (x) f (−y) − f (−xy) + 1 = f (x)[2 − f (y)] + f (xy) − 2 + 1. (2)

Cộng (1) với (2), ta được

f (x + y) + f (x − y) = 2 f (x).

Thay y = x vào (2) và chú ý rằng f (0) = 1 thì ta được

f 2 (x) = f (x2 ) + 2 f (x) − 2,

suy ra
[ f (x) − 1]2 = f (x2 ) − 1.
54 Trần Nam Dũng (chủ biên)

Từ đó, nếu đặt g(x) = f (x) − 1 thì ta có

g(0) = 0, g(x + y) + g(x − y) = 2g(x), g(x2 ) = g2 (x).

Từ đây, áp dụng lý thuyết căn bản về phương trình hàm Cauchy, dễ dàng tìm được
g(x) = x.

Vậy có hai hàm số thoả mãn điều kiện đề bài là f (x) = 1 và f (x) = 1 + x.

Ghi chú. Nếu xét hàm f : Q → Q thì sẽ làm bài toán dễ hơn.

Bài 4.13. Tìm tất cả các hàm số f : N∗ → N∗ thoả mãn đồng thời các điều kiện sau
đây

(i) f (m) > f (n) với mọi m > n thuộc N∗ ;

(ii) f ( f (n)) = 4n + 9 với mọi n thuộc N∗ ;

(iii) f ( f (n) − n) = 2n + 9 với mọi n thuộc N∗ .

(Phổ thông Năng khiếu)

Lời giải. Cách 1. Vì f tăng thực sự nên f (n) ≥ n với mọi n ∈ N∗ . Ngoài ra, cũng do
f tăng thực sự nên với mọi m, n ∈ N∗ mà m < n ta có n − m ≤ f (n) − f (m). Từ đó
ta có

[ f (n + 1) − (n + 1)] − [ f (n) − n] ≤ f ( f (n + 1) − n − 1)) − f ( f ((n) − n)),

hay
f (n + 1) − f (n) − 1 ≤ 2(n + 1) + 9 − (2n + 9).
Như thế
f (n + 1) − f (n) ≤ 3 ∀n ∈ N∗ .
Giả sử có n mà f (n + 1) = f (n) + 1, nghĩa là f (n + 1) − (n + 1) = f (n) − n. Từ đó

f ( f (n + 1) − (n + 1)) = f ( f (n) − n).

Dẫn đến 2(n + 1) + 9 = 2n + 9, mâu thuẫn. Tóm lại, ta có

2 ≤ f (n + 1) − f (n) ≤ 3 ∀n ∈ N∗ . (∗)

Ta có f ( f (n)) − f ( f (n) − n) = 2n. Mặt khác từ (∗) ta lại có

f ( f (n)) − f ( f (n) − n) = [ f ( f (n)) − f ( f (n) − 1)] + [ f ( f (n) − 1) − f ( f (n) − 2)]+


+ · · · + [ f ( f (n) − n + 1) − f ( f (n) − n)] ≥ 2n.
Lời giải và bình luận đề thi các tỉnh, các trường Đại học năm học 2009-2010 55

Từ đó suy ra

[ f ( f (n)) − f ( f (n) − 1)] = [ f ( f (n) − 1) − f ( f (n) − 2)] = · · · =


= [ f ( f (n) − n + 1) − f ( f (n) − n)] = 2.

Giả sử f (2) = f (1) + 3. Khi đó f (2) − 2 = f (1) + 1 > f (1). Từ đó f ( f (2) − 2) >
f ( f (1)) hay 2 · 2 + 9 > 4 · 1 + 9, vô lí. Như vậy ta phải có f (2) = f (1) + 2.

Lại giả sử f (3) = f (2) + 3. Khi đó f (3) − 3 = f (2). Như thế f ( f (3) − 3) = f ( f (2))
hay 2 · 3 + 9 = 2.4 + 9, vô lí. Như vậy ta cũng có f (3) = f (2) + 2.

Giả sử có k ≥ 4 sao cho f (k + 1) = f (k) + 3. Khi đó do 3 ≤ k − 1 nên sử dụng (∗)


ta được

f ( f (k)) = f ( f (k + 1) − 3) = f ( f (k + 1) − k − 1) + 2(k − 2),

hay 4k + 9 = 2(k + 1) + 9 + 2k − 4, vô lí. Do đó với mọi n ta có f (n + 1) = f (n) + 2.

Như thế dãy { f (n)} là cấp số cộng với công sai là 2. Vì thế f (n) = 2n + b. Thay
biểu thức của f (n) vào hệ thức f ( f (n)) = 4n + 9 ta được

4n + 9 = f (2n + b) = 2(2n + b) + b.

Từ đó b = 3 và f (n) = 2n + 3. Dễ thấy f (n) = 2n + 3 thỏa mãn (i) và (iii). Vậy hàm


số duy nhất cần tìm là f (n) = 2n + 3 với mọi n ∈ N∗ .

Cách 2. (Vắn tắt) Thay n bởi 2n trong (iii), ta được

f ( f (2n) − 2n) = 4n + 9.

So sánh với (ii), ta suy ra


f (n) = f (2n) − 2n.
Từ đó sử dụng quy nạp theo n (xét n chẵn, n lẻ) để chứng minh f (n) = 2n + 3 với
mọi n nguyên dương.

Bài 4.14. Tìm tất cả các hàm số f : Q → R thỏa mãn

f (x + y) = f (x) f (y) − f (xy) + 1 ∀x, y ∈ Q.

(Bắc Giang)
56 Trần Nam Dũng (chủ biên)
Chương 5

Hình học

“Giữa những bộ óc thông minh ngang nhau và trong những điều kiện tương tự, ai có tinh
thần hình học thì người đó sẽ thắng và thu được một cường lực hoàn toàn mới mẻ.”
Blaise Pascal

5.1 Đề bài
5.1. Cho tam giác ABC nội tiếp đường tròn (O) và ngoại tiếp đường tròn (I). Gọi D
là một điểm trên đoạn BC, đường tròn (P) tiếp xúc với DC, DA tại E, F và tiếp xúc
trong với (O) tại K. Chứng minh rằng E, F, I thẳng hàng.

5.2. Cho hình lập phương ABCD.A0 B0C0 D0 cạnh bằng a. Với M là một điểm thuộc
cạnh AB, chọn điểm N thuộc cạnh D0C0 sao cho AM + D0 N = a.

(1) Chứng minh rằng MN đi qua một điểm cố định khi M thay đổi.

(2) Tính thể tích chóp B0 .A0 MCN theo a. Xác định vị trí của M để khoảng cách từ
B0 tới mặt phẳng (A0 MCN) đạt giá trị lớn nhất. Tính khoảng cách lớn nhất đó
theo a.

(3) Tìm quỹ tích hình chiếu vuông góc của C xuống MN khi M chạy trên AB.

5.3. Cho đường tròn (O) và hai điểm biên B, C sao cho B, C không phải là đường
kính. Điểm A chuyển động trên cung lớn BC (khác B, C). Gọi M là trung điểm cạnh
AB và N là hình chiếu vuông góc của M lên AC. Cho trước số thực a khác 1 và gọi K
là điểm chia đoạn HN theo tỉ số a, với H là trung điểm cạnh BC. Vẽ đường thẳng d
qua K và vuông góc với HN. Chứng minh rằng d luôn tiếp xúc với một đường cong
cố định.

57
58 Trần Nam Dũng (chủ biên)

5.4. Cho hai đường thẳng a, b cắt nhau tại M và không vuông góc với nhau. Dựng
parabol tiếp xúc a tại A và tiếp xúc b tại B, với A, B là hai điểm cho trước thuộc a, b.
5.5. Cho tam giác ABC vuông cân tại A. D, E, F lần lượt thuộc các cạnh BC, CA,
AB sao cho tam giác DEF vuông cân tại D. Tìm tập hợp trung điểm I của EF.
5.6. Cho khối tứ diện ABCD có thể tích là V. Diện tích các tam giác ABC, ABD lần
lượt là S1 , S2 . Gọi x là số đo góc tạo bởi hai mặt phẳng (ABC) và (ABD). M là một
điểm thuộc cạnh CD sao cho khoảng cách từ M đến hai mặt phẳng (ABC)và (ABD)
bằng nhau.
2S1 S2 sin x CM S1
(a) Chứng minh rằng V = và = .
3AB DM S2
(b) Tính diện tích tam giác AMB theo V, S1 , S2 , x.
5.7. Cho KL và KN là các tiếp tuyến của đường tròn (C), với L, N thuộc (C). Lấy
M bất kì trên đường thẳng KN (M, K khác phía so với N). Giả sử (C) cắt đường tròn
ngoại tiếp tam giác KLM tại điểm thứ hai là P. Q là chân đường vuông góc hạ từ N
xuống ML. Chứng minh rằng

∠MPQ = 2∠KML.

5.8. Cho tam giác ABC cân tại A, đường cao AH. Đường tròn (O) đi qua A, B cắt
đoạn AH tại K. Điểm L thuộc đoạn AB sao cho KL k AC. Gọi E = BK ∩CL. Đường
thẳng AE cắt lại (O) tại điểm thứ hai F. Chứng minh rằng

∠AFL = ∠BAC.

5.9. Cho tam giác ABC. Dựng các điểm X, Y sao cho hai tam giác ABX, ACY đồng
dạng ngược hướng. Dựng các điểm T, K sao cho các tam giác BXA, BTC, KXY
đồng dạng cùng hướng. Chứng minh rằng hai tam giác BTC và KXY có chung tâm
đường tròn ngoại tiếp.
5.10. Cho tam giác ABC cân tại A. Gọi H là trung điểm của BC, D là hình chiếu của
H trên cạnh AC, M là trung điểm HD. Chứng minh rằng AM vuông góc với BD.
5.11. Tam giác ABC (AB > AC) nội tiếp (O). Phân giác ngoài tại A cắt (O) tại E.
Gọi F là hình chiếu của E trên AB. Chứng minh rằng

2AF = AB − AC.

5.12. Cho tứ giác nội tiếp ABCD. Gọi a, b, c, d theo thứ tự là phân giác ngoài của
các góc ∠DAB, ∠ABC, ∠BCD, ∠CDA. K = a ∩ b, L = b ∩ c, M = b ∩ d, N = d ∩ a.
Chứng minh rằng tứ giác KLMN nội tiếp một đường tròn và đường tròn đó có bán
KM · LN
kính bằng .
AB + BC +CD + DA
Lời giải và bình luận đề thi các tỉnh, các trường Đại học năm học 2009-2010 59

5.13. Cho tam giác ABC nhọn, không cân, nội tiếp đường tròn (O). Các đường cao
AA0 , BB0 , CC0 đồng quy tại H. Các điểm A1 , A2 thuộc (O) sao cho đường tròn ngoại
tiếp các tam giác A1 B0C0 , A2 B0C0 tiếp xúc với (O). Tương tự ta có các điểm B1 , B2
và các điểm C1 , C2 . Chứng minh rằng các đường thẳng A1 A2 , B1 B2 , C1C2 đồng quy
tại một điểm thuộc OH.
5.14. Cho tam giác nhọn ABC có trung tuyến CM vuông góc với phân giác trong
AL tại đỉnh A (với M, L lần lượt thuộc các cạng AB, BC). Đặt AC = b, AB = c.
(a) Chứng minh rằng

→ b −→ c −→
AL = AB + AC.
b+c c+b
(b) Giả sử CM = k · AL, k > 0. Chứng minh rằng

9 − 4k2
cos A = .
9 + 4k2

5.15. Cho hình vuông ABCD. Trên các cạnh CB và CD lần lượt lấy các điểm E, F
BE DF 1 − k
sao cho = k và = , với 0 < k < 1. Đoạn thẳng BD cắt AE và AF tại H
BC DC 1 + k
và G tương ứng. Đường vuông góc với EF kẻ từ A cắt BD tại P. Chứng minh rằng
PG DG
= .
PH BH
5.16. Trong mặt phẳng (P)cho điểm O cố định và dlà đường thẳng quay quanh O.
Lấy S ngoài (P)có hình chiếu vuông góc trên (P) là H, với H 6≡ O. Qua S dựng
đường vuông góc với mặt phẳng xác định bởi S và d. Đường thẳng này cắt (P) tại
N. Tìm quỹ tích điểm N khi d thay đổi.
5.17. Cho đường tròn tâm O và một dây cung AB cố định không là đường kính. Một
điểm P thay đổi trên cung lớn AB. Gọi I là trung điểm của AB. Lấy các điểm M, N
trên các tia PA, PB tương ứng sao cho ∠PMI = ∠PNI = ∠APB.
(a) Chứng minh đường cao kẻ từ đỉnh P của tam giác PMN đi qua một điểm cố
định.

(b) Chứng minh rằng đường thẳng Euler của tam giác PMN đi qua một điểm cố
định.
5.18. Cho tam giác ABC nội tiếp đường tròn (O). Gọi I, I1 , I2 , I3 là tâm đường tròn
nội tiếp và bàng tiếp các góc A, B, C tương ứng. Đường tròn ngoại tiếp tam giác
II2 I3 cắt (O) tại hai điểm M1 , N1 . Gọi J1 là giao điểm của AI và (O). Kí hiệu d1 là
đường thẳng đi qua J1 và vuông góc với M1 N1 . Tương tự xác định các đường thẳng
d2 , d3 . Chứng minh các đường thẳng d1 , d2 , d3 đồng quy tại một điểm.
60 Trần Nam Dũng (chủ biên)

5.2 Lời giải


Bài 5.1. Cho tam giác ABC nội tiếp đường tròn (O) và ngoại tiếp đường tròn (I).
Gọi D là một điểm trên đoạn BC, đường tròn (P) tiếp xúc với DC, DA tại E, F và
tiếp xúc trong với (O) tại K. Chứng minh rằng E, F, I thẳng hàng.
(Đại học Vinh)
Lời giải. Trước hết, ta có hai nhận xét khá quan trọng sau, chứng minh chi tiết xin
dành cho bạn đọc.

Nhận xét 1. AB là dây của một đường tròn tâm (O). Đường tròn (l) tiếp xúc với dây
AB tại K và tiếp xúc trong với (O) tại T. Khi đó L là trung điểm của cung AB không
chứa T và LA2 = LK · LT.

Nhận xét 2. Điểm M là trung điểm cung BC không chứa A của đường tròn ngoại tiếp
tam giác ABC. Điểm I thuộc đoạn MA sao cho MI = MB. Khi đó I là tâm đường
tròn nội tiếp tam giác ABC.

Bây giờ ta sẽ đi chứng minh bài toán đã cho. Giả sử KF cắt đường tròn (O) tại
G. Theo nhận xét 1, ta có G là điểm chính giữa cung BC. Gọi I là giao điểm của
AG với EF. Ta có ∠IEK = ∠IAK = ∠FKD nên tứ giác AEIK nội tiếp, suy ra
∠AIK = ∠EFK = ∠AEK. Từ đây ta suy ra ∆EFK đồng dạng với ∆IAK (g.g), do đó
∠EKA = ∠GKI = ∠GIF, vì thế ∆GIF đồng dạng với ∆GKI (g.g). Từ kết quả này
ta tìm được GI 2 = GF · GK. Lại cũng từ nhận xét 1, ta có GC2 = GF · GK. Do đó,
GC = GI và ta suy ra I là tâm nội tiếp của ∆ABC (Theo nhận xét 2). Từ đây ta suy
ra ngay điều phải chứng minh.
Bài 5.2. Cho hình lập phương ABCD.A0 B0C0 D0 cạnh bằng a. Với M là một điểm
thuộc cạnh AB, chọn điểm N thuộc cạnh D0C0 sao cho AM + D0 N = a.
(1) Chứng minh rằng MN đi qua một điểm cố định khi M thay đổi.
(2) Tính thể tích chóp B0 .A0 MCN theo a. Xác định vị trí của M để khoảng cách từ
B0 tới mặt phẳng (A0 MCN) đạt giá trị lớn nhất. Tính khoảng cách lớn nhất đó
theo a.
(3) Tìm quỹ tích hình chiếu vuông góc của C xuống MN khi M chạy trên AB.
(Hà Nội)
Bài 5.3. Cho đường tròn (O) và hai điểm biên B, C sao cho B, C không phải là
đường kính. Điểm A chuyển động trên cung lớn BC (khác B, C). Gọi M là trung
điểm cạnh AB và N là hình chiếu vuông góc của M lên AC. Cho trước số thực a
khác 1 và gọi K là điểm chia đoạn HN theo tỉ số a, với H là trung điểm cạnh BC.
Vẽ đường thẳng d qua K và vuông góc với HN. Chứng minh rằng d luôn tiếp xúc
với một đường cong cố định.
(Đại học Khoa học tự nhiên)
Lời giải và bình luận đề thi các tỉnh, các trường Đại học năm học 2009-2010 61

Bài 5.4. Cho hai đường thẳng a, b cắt nhau tại M và không vuông góc với nhau.
Dựng parabol tiếp xúc a tại A và tiếp xúc b tại B, với A, B là hai điểm cho trước
thuộc a, b.
(Đại học Khoa học tự nhiên)

Bài 5.5. Cho tam giác ABC vuông cân tại A. D, E, F lần lượt thuộc các cạnh BC,
CA, AB sao cho tam giác DEF vuông cân tại D. Tìm tập hợp trung điểm I của EF.
(Bắc Ninh)

Lời giải. Dễ thấy tứ giác AEDF là tứ giác nội tiếp mà DE = DF nên ta suy ra AD là
phân giác của góc ∠EAF ≡ ∠BAC. Từ đó suy ra D phải là trung điểm của BC. Gọi
M1 , M2 lần lượt là hình chiếu của D lên AB và AC, khi đó M1 , M2 theo thứ tự sẽ là
trung điểm của AB và AC. Gọi I là trung điểm của EF, thế thì ta có DI ⊥ EF. Từ đó,
theo định lý đường thằng Simpson cho điểm D ∈ (AEF), ta thu được ngay M1 , M2
và I thẳng hàng. Vậy I luôn di động trên M1 M2 cố định. Ta có được điều phải chứng
minh.

Bài 5.6. Cho khối tứ diện ABCD có thể tích là V. Diện tích các tam giác ABC, ABD
lần lượt là S1 , S2 . Gọi x là số đo góc tạo bởi hai mặt phẳng (ABC) và (ABD). M
là một điểm thuộc cạnh CD sao cho khoảng cách từ M đến hai mặt phẳng (ABC)và
(ABD) bằng nhau.

2S1 S2 sin x CM S1
(a) Chứng minh rằng V = và = .
3AB DM S2

(b) Tính diện tích tam giác AMB theo V, S1 , S2 , x.

(Ninh Bình)

Bài 5.7. Cho KL và KN là các tiếp tuyến của đường tròn (C), với L, N thuộc (C).
Lấy M bất kì trên đường thẳng KN (M, K khác phía so với N). Giả sử (C) cắt đường
tròn ngoại tiếp tam giác KLM tại điểm thứ hai là P. Q là chân đường vuông góc hạ
từ N xuống ML. Chứng minh rằng

∠MPQ = 2∠KML.

(Hải Phòng)

Lời giải.
62 Trần Nam Dũng (chủ biên)

Gọi P0 ≡ MP ∩ (O), P0 6= P và Q0 ≡ ON ∩ ML. Khi ấy, ta có

MN 2 = MQ · MQ0 = MP · MP0 ,

suy ra PQQ0 P0 là tứ giác nội tiếp. Vì vậy ∠MPQ = ∠P0 Q0 M. Ta đi chứng minh
LP0 k MK.

Thật vậy, ta có

(PM, PN) ≡ (LP0 , LN) ≡ (PL, PN) − (PL, PM) ≡ (NL, NK) + (KN, KL)
≡ (LN, LK) ≡ (NK, NL) ≡ (MK, NL) (mod π),

nên LP0 k MK. Dẫn đến ON ⊥ P0 L, suy ra OQ0 ⊥ P0 L. Nhưng O ∈ đường trung trực
của LP0 , vì vậy Q0 ∈ đường trung trực của LP0 , theo đó ta được

∠P0 Q0 M = 2 · ∠Q0 LP0 ≡ 2 · ∠MLP0 = 2 · ∠KML (do LP0 k MK),

suy ra ∠MPQ = 2 · ∠KML. Đó là điều phải chứng minh.

Bài 5.8. Cho tam giác ABC cân tại A, đường cao AH. Đường tròn (O) đi qua A,
B cắt đoạn AH tại K. Điểm L thuộc đoạn AB sao cho KL k AC. Gọi E = BK ∩CL.
Đường thẳng AE cắt lại (O) tại điểm thứ hai F. Chứng minh rằng

∠AFL = ∠BAC.
Lời giải và bình luận đề thi các tỉnh, các trường Đại học năm học 2009-2010 63

Bài 5.9. Cho tam giác ABC. Dựng các điểm X, Y sao cho hai tam giác ABX, ACY
đồng dạng ngược hướng. Dựng các điểm T, K sao cho các tam giác BXA, BTC,
KXY đồng dạng cùng hướng. Chứng minh rằng hai tam giác BTC và KXY có chung
tâm đường tròn ngoại tiếp.
(Đại học Sư phạm)

Bài 5.10. Cho tam giác ABC cân tại A. Gọi H là trung điểm của BC, D là hình
chiếu của H trên cạnh AC, M là trung điểm HD. Chứng minh rằng AM vuông góc
với BD.
(Đồng Nai)

Lời giải. Gọi K là hình chiếu của B lên AC. Khi ấy ta có HD k BK, lại có H là trung
điểm của BC dẫn đến D là trung điểm của KC. Qua B, A, vẽ các tia By, Ax. Khi ấy,
ta thu được (AH, AD, AM, Ax) = (BC, BK, BD, By) = −1. Nhưng dễ thấy, AH,
AD, Ax lần lượt vuông góc với BC, BK, By. Suy ra AM ⊥ BD. Đó là điều phải chứng
minh.

Bài 5.11. Tam giác ABC (AB > AC) nội tiếp (O). Phân giác ngoài tại A cắt (O) tại
E. Gọi F là hình chiếu của E trên AB. Chứng minh rằng

2AF = AB − AC.

(Đồng Nai)

Lời giải. Trước tiên chúng ta dễ thấy rằng E là điểm chính giữa cung BC chứa A.
Trên đoạn BA lấy điểm M sao cho BM = AC. Ta thấy

EB = EC, ∠EBM = ∠ECA, BM = CA.

Do đó hai tam giác EBM và ECA bằng nhau nên EM = EA. Mặt khác do EF ⊥ AM
nên FM = FA. Như vậy

2AF = AM = AB − MB = AB − AC.

Bình luận. Bài toán này có thể nhìn nhận là hệ quả trực tiếp của định lý Archimedes,
cụ thể với kí hiệu như trên định lý Archimedes cho biết BF = FA + AC và từ đây
việc suy ra kết luận của bài toán thật đơn giản.

Bài 5.12. Cho tứ giác nội tiếp ABCD. Gọi a, b, c, d theo thứ tự là phân giác ngoài
của các góc ∠DAB, ∠ABC, ∠BCD, ∠CDA. K = a ∩ b, L = b ∩ c, M = b ∩ d, N =
d ∩ a. Chứng minh rằng tứ giác KLMN nội tiếp một đường tròn và đường tròn đó
KM · LN
có bán kính bằng .
AB + BC +CD + DA
(Đại học Sư phạm)
64 Trần Nam Dũng (chủ biên)

Lời giải. (a) Đặt E = AD ∩ BC, ∠AEB = α. Ta có E, K, M thẳng hàng vì cùng nằm
trên đường phân giác trong góc ∠AEB. Chú ý rằng K là tâm
 nội tiếp ∆EAB, M là
◦ α ◦ α
tâm bàng tiếp ∆ECD nên ∠NKL + ∠NML = 90 + + 90 − = 180◦ . Vậy
2 2
KLMN là tứ giác nội tiếp.

(b) Theo định lý hàm số sin ta có

NL NL
R=  α = α.
2 sin 90◦ + 2 cos
2 2

Do đó yêu cầu của bài toán tương đương với việc chứng minh

NL NL · KM
α = AB + BC +CA + AD ,
2 cos
2

hay
α
AB + BC +CA + AD = 2 cos KM.
2

Gọi X, Y là hình chiếu của K, M lên BC. Ta có

α α α
cos KM = cos EM − cos EK = EY − EX
2 2 2
EC + DC + ED EB + EA − AB
= +
2 2
AB + BC +CA + AD
= .
2

Phép chứng minh được hoàn tất.

Bài 5.13. Cho tam giác ABC nhọn, không cân, nội tiếp đường tròn (O). Các đường
cao AA0 , BB0 , CC0 đồng quy tại H. Các điểm A1 , A2 thuộc (O) sao cho đường tròn
ngoại tiếp các tam giác A1 B0C0 , A2 B0C0 tiếp xúc với (O). Tương tự ta có các điểm
B1 , B2 và các điểm C1 , C2 . Chứng minh rằng các đường thẳng A1 A2 , B1 B2 , C1C2
đồng quy tại một điểm thuộc OH.
(Đại học Sư phạm)

Lời giải.
Lời giải và bình luận đề thi các tỉnh, các trường Đại học năm học 2009-2010 65

Gọi (Oa ) và (O0a ) lần lượt là đường tròn ngoại tiếp các tam giác 4A1 B0C0 và
4A2 B0C0 . Gọi ta và ta0 lần lượt là các tiếp tuyến chung tại A1 và A2 của (Oa ) và
(O0a ) với (O). Kí hiệu ([BC]) ám chỉ đường tròn đường kính BC.

Ta có ta , BC và B0C0 lần lượt là trục đẳng phương của các cặp đường tròn ((Oa ), (O)),
((O), ([BC])) và (([BC]), (Oa )). Do đó ta , BC và B0C0 đồng quy tại A3 . Lập luận
tương tự, ta cũng có ta , BC và B0C0 đồng quy tại A03 , vì vậy A3 ≡ A03 . Hay nói cách
khác, A3 ≡ B0C0 ∩ BC. Xác định tương tự cho B3 , C3 .

Bây giờ, gọi Ma là trung điểm của BC. Do (A3 A0 BC) = −1 nên theo hệ thức Maclau-
ren, ta thu được A3 B · A3C = A3 A0 · A3 Ma . Từ đó suy ra A3 có cùng phương tích wrt
(O) và đường tròn 9-điểm Euler, kí hiệu là (E ) wrt 4ABC. Lập luận tương tự cho
B3 , C3 . Ta kết luận A3 , B3 , C3 thẳng hàng (∗). Suy ra đường thẳng d đi qua A3 , B3 ,
C3 là trục đẳng phương của (O) và (E ). Vậy nên OE ≡ OH ⊥ d (∗∗).

Để ý rằng A3 , B3 , C3 lần lượt là cực của A1 A2 , B1 B2 , C1C2 wrt (O). Kết hợp với (∗),
ta suy ra A1 A2 , B1 B2 , C1C2 đồng quy tại điểm S đồng thời cũng là cũng là cực của d
wrt (O), do đó OS ⊥ d. Kết hợp với (∗∗), ta suy ra S ∈ OH.

Tóm lại, các đường thẳng A1 A2 , B1 B2 , C1C2 đồng quy tại một điểm thuộc OH.

Bài 5.14. Cho tam giác nhọn ABC có trung tuyến CM vuông góc với phân giác
trong AL tại đỉnh A (với M, L lần lượt thuộc các cạng AB, BC). Đặt AC = b, AB = c.
66 Trần Nam Dũng (chủ biên)

(a) Chứng minh rằng



→ b −→ c −→
AL = AB + AC.
b+c c+b

(b) Giả sử CM = k · AL, k > 0. Chứng minh rằng

9 − 4k2
cos A = .
9 + 4k2

(Kon Tum)

Bài 5.15. Cho hình vuông ABCD. Trên các cạnh CB và CD lần lượt lấy các điểm
BE DF 1 − k
E, F sao cho = k và = , với 0 < k < 1. Đoạn thẳng BD cắt AE và AF
BC DC 1 + k
tại H và G tương ứng. Đường vuông góc với EF kẻ từ A cắt BD tại P. Chứng minh
rằng
PG DG
= .
PH BH
(Phú Yên)

Bài 5.16. Trong mặt phẳng (P)cho điểm O cố định và dlà đường thẳng quay quanh
O. Lấy S ngoài (P)có hình chiếu vuông góc trên (P) là H, với H 6≡ O. Qua S dựng
đường vuông góc với mặt phẳng xác định bởi S và d. Đường thẳng này cắt (P) tại
N. Tìm quỹ tích điểm N khi d thay đổi.
(Phú Yên)

Bài 5.17. Cho đường tròn tâm O và một dây cung AB cố định không là đường kính.
Một điểm P thay đổi trên cung lớn AB. Gọi I là trung điểm của AB. Lấy các điểm
M, N trên các tia PA, PB tương ứng sao cho ∠PMI = ∠PNI = ∠APB.

(a) Chứng minh đường cao kẻ từ đỉnh P của tam giác PMN đi qua một điểm cố
định.

(b) Chứng minh rằng đường thẳng Euler của tam giác PMN đi qua một điểm cố
định.

(Phổ thông Năng khiếu)

Lời giải.
Lời giải và bình luận đề thi các tỉnh, các trường Đại học năm học 2009-2010 67

(a) Gọi K và Q lần lược là giao điểm của IN với PA và IM với PB. Từ đó, theo giải
thiết đầu bài, ta thu được MKQN là tứ giác nội tiếp. Gọi F là tâm đường tròn ngoại
tiếp 4OAB. Ta có

∠AFI = ∠AOB = 2∠APB = 180◦ − ∠PKN = ∠MKN ≡ ∠AKI.

Dẫn đến A, K, F, I đồng viên. Suy ra ∠FKA = ∠FIA = 90◦ . Lập luận tương tự ta
cũng thu được, ∠FQE = 90◦ . Dẫn đến P, K, Q nội tiếp đường tròn đường kính PF.
Từ đây ta thấy, PF cũng chính là đường cao hạ từ P đến MN. Mà F cố định. Ta suy
ra điều cần chứng minh.

(b) Gọi O0 là tâm đường tròn ngoại tiếp tam giác PMN và H là trực tâm của tam
giác PMN. Ta có O0 nằm trên trung trực của PM. Mặt khác, ta sẵn có Q nằm trên
trung trực của PM. Do đó, QO0 ⊥ PA. Tương tự ta cũng có O0 K ⊥ PQ. Bây giờ gọi
K 0 , Q0 lần lượt là hình chiếu của K, Q lên PQ, PK và M 0 , N 0 lần lượt là hình chiếu
của M, N lên PN, PM. Khi ấy ta có các hệ thức sau

O0 K 0 · O0 K = O0 Q0 · O0 Q, IQ · IM = IA · IN, HN · HN 0 = HM · HM 0 .

Điều này ám chỉ O0 , I, H có cùng phương tích với đường tròn đường kính KN và
đường tròn đường kính QM. Suy ra, O0 , I, H thằng hàng. Kết luận, O0 H, tức đường
thẳng Euler của tam giác PMN luôn đi qua I cố định. Ta có điều phải chứng minh.
Bài 5.18. Cho tam giác ABC nội tiếp đường tròn (O). Gọi I, I1 , I2 , I3 là tâm đường
tròn nội tiếp và bàng tiếp các góc A, B, C tương ứng. Đường tròn ngoại tiếp tam
68 Trần Nam Dũng (chủ biên)

giác II2 I3 cắt (O) tại hai điểm M1 , N1 . Gọi J1 là giao điểm của AI và (O). Kí hiệu
d1 là đường thẳng đi qua J1 và vuông góc với M1 N1 . Tương tự xác định các đường
thẳng d2 , d3 . Chứng minh các đường thẳng d1 , d2 , d3 đồng quy tại một điểm.
(Phổ thông Năng khiếu)
Lời giải.
I1

I2 I3

Dễ thấy I là trực tâm của tam giác 4I1 I2 I3 . Do vậy, nếu gọi O1 và O0 lần lượt là tâm
đường tròn ngoại tiếp tam giác 4II2 I3 và 4I1 I2 I3 . Theo một kết quả quen thuộc, ta
có O1 và O0 là ảnh của nhau qua phép đối xứng với đường thẳng I2 I3 và I1 IO1 O0 là
hình bình hành. Suy ra, I1 O1 đi qua trung điểm của O0 I, đồng thời cũng chính là tâm
đường tròn 9-điểm Euler của 4I1 I2 I3 , do vậy ba điểm I1 , O, O1 thẳng hàng. Hơn
nưã, để ý rằng M1 N1 chính là trục đẳng phương của (ABC) và (O1 ) nên I1 O ⊥ M1 N1 .
Lập luận tương tự cho các đỉnh I2 , I3 . Khi ấy, ta thu được, các đường thằng d10 , d20 ,
d30 qua I1 , I2 , I3 lần lượt vuông góc với M1 N1 , M2 N2 , M3 N3 đồng quy tại O.

Xét phép vị tự tâm I, tỉ số k = 21 , biến I1 , I2 , I3 lần lượt thành J1 , J2 , J3 . Cho nên


H (I, k) biến d10 , d20 , d30 lần lượt thành các đường thằng d1 , d2 , d3 . Dẫn đến d1 , d2 ,
d3 đồng quy tại ảnh của O qua H (I, k). Ta thu được điều phải chứng minh.

Chú ý. Bài toán này còn có thể giải ngắn gọn hơn bằng định lý Carnot mở rộng. Nội
dung của định lý như sau:

Xét hai tam giác ABC và tam giác A0 B0C0 . Khi ấy các đường thẳng qua A, B, C vuông
góc lần lượt với B0C0 , C0 A0 và A0 B0 đồng quy khi và chỉ khi các đường vuông góc kẻ
lần lượt từ A0 , B0 , C0 đến BC, CA, AB đồng quy.
Lời giải và bình luận đề thi các tỉnh, các trường Đại học năm học 2009-2010 69

Chứng minh xin giành cho bạn đọc.


70 Trần Nam Dũng (chủ biên)
Chương 6

Tổ hợp

“Không có bài toán nào không giải được. Chúng ta phải biết và sẽ biết.”
David Hilbert

6.1 Đề bài
6.1. Một người ham thích làm toán mỗi ngày làm một hoặc hai bài toán, nhưng mỗi
tuần làm không quá 10 bài. Chứng minh rằng có một số ngày liên tiếp người ấy làm
đúng 30 bài toán.

6.2. Sau khi khai trương được đúng 10 ngày, một nhân viên thư viện cho biết

(1) Mỗi ngày có đúng tám người đến đọc sách;

(2) Không có người nào đến thư viện một ngày quá một lần ;

(3) Trong hai ngày bất kỳ của 10 ngày đó thì có ít nhất là 15 người khác nhau
cùng đến thư viện.

Căn cứ đồng thời cả ba điều kiện mà nhân viên thư viện cung cấp hãy cho biết số
người tối thiểu đã đến thư viện trong 10 ngày nói trên là bao nhiêu?

6.3. Chứng minh rằng nếu chọn ra 15 số bất kỳ từ tập hợp {2, 3, . . . , 2010} sao cho
chúng đôi một nguyên tố cùng nhau thì sẽ có ít nhất một số nguyên tố được chọn.

6.4. Cho n là một số nguyên dương. Chứng minh rằng với mọi dãy a1 , a2 , . . . , an ta
luôn chọn được số tự nhiên k ≤ n sao cho

|(a1 + · · · + ak ) − (ak+1 + · · · + an )| ≤ max{|a1 |, |a2 |, . . . , |an |}.

71
72 Trần Nam Dũng (chủ biên)

6.5. Giả sử ta có thể chọn được n số phân biệt từ tập {1, 2, 3, . . . , 2n − 1} sao cho
các số được chọn không có hai số nào chia hết cho nhau. Chứng minh rằng không
có số nào trong các số trên nhỏ hơn 2k , trong đó k là số xác định bởi điều kiện
3k < 2n < 3k+1 .

6.6. Cho tập hợp X = {1, 2, . . . , 2010}. Tìm số nguyên N lớn nhất sao cho mỗi
hoán vị ωX = (a1 , a2 , . . . , a2010 ) của X đều tồn tại 30 số hạng liên tiếp có tổng
không nhỏ hơn N.

6.7. Tập hợp các số nguyên dương được tô bởi hai màu đen và trắng. Giả thuyết
rằng, tổng của hai số khác màu luôn bị tô màu đen và có vô hạn số bị tô màu trắng.
Chứng minh rằng tổng và tích của hai số bị tô màu trắng cũng sẽ bị tô màu trắng.

6.8. Cho tập hợp S = {1, 2, 3, . . . , n}. Tìm số cách chia tập S thành 3 tập con khác
rỗng sao cho mỗi tập con không chứa hai số nguyên liên tiếp.

6.9. Cho n là số nguyên lớn hơn 1 và P1 , P2 , . . . , Pn là các tập con có hai phần tử và
đôi một phân biệt của tập hợp S = {1, 2, 3, . . . , n} thỏa mãn tính chất: nếu i 6= j mà
Pi ∩ Pj 6= 0/ thì tồn tại k để Pk = {i, j}. Chứng minh rằng với mỗi số i ∈ S xuất hiện
đúng hai lần trong các tập Pj với j = 1, 2, . . . , n.

6.10. Cho số nguyên n không nhỏ hơn 3. Giả sử mỗi số nguyên dương không lớn
hơn Cn1 +Cn2 +Cn3 được tô một trong hai màu xanh hoặc đỏ. Chứng minh tồn tại dãy
các số cùng màu thỏa mãn

(1) x1 < x2 < · · · < xn ;

(2) x2 − x1 ≤ x3 − x2 ≤ · · · ≤ xn − xn−1 ≤ Cn2 .

6.11. Cho tập hợp A gồm n ≥ 5 phần tử. Xét k tập con bất kì gồm ba phần tử của A.
Hãy tìm số k nhỏ nhất sao cho với mọi cách chọn k tập con trên luôn tồn tại hai tập
con có chung nhau đúng một phần tử.

6.12. Cho tập S = {1, 2, 3, . . . , 2009}. A là tập con có n phần tử của S. Tìm n nhỏ
a
nhất sao với mọi cách chọn tập A thì trong A luôn có hai phần tử a, b mà = 3.
b
6.13. Cho tập A = {1, 2, 3, . . . , 2009}. Chứng minh rằng, có thể tô màu mỗi phần
tử của tập A bằng một trong hai màu đen trắng sao cho mọi cấp số cộng công sai
khác 0 gồm 18 phần tử của A đều được tô bởi đủ cả hai màu.

6.14. Tập hợp A ⊂ R được gọi là có tính chất T nếu A có không ít hơn bốn phần tử
và ab + cd thuộc A với mọi a, b, c, d phân biệt thuộc A.

(a) Hãy chỉ ra tập hợp A gồm bốn phần tử, có tính chất T.

(b) Có hay không tập hợp A ⊂ (0, +∞) gồm bốn phần tử và có tính chất T.
Lời giải và bình luận đề thi các tỉnh, các trường Đại học năm học 2009-2010 73

6.15. Cho số nguyên dương n > 10. Tìm m ∈ N∗ lớn nhất thỏa mãn điều kiện: Tồn
tại m tập con A j của tập A = {1, 2, 3, . . . , 2n}, mỗi tập con gồm n phần tử sao cho
|Ai ∩ A j ∩ Ak | ≤ 1 với mọi 1 ≤ i < j < k ≤ n.

6.16. Cho A = {1, 2, . . . , 2n}. Một tập con của A được gọi là tốt nếu nó có đúng
hai phần tử x, y và |x − y| ∈ {1, n}. Tìm số các tập hợp {A1 , A2 , . . . , An } thoả mãn
điều kiện Ai là tập con tốt với mọi i = 1, 2, . . . , n và A1 ∪ A2 ∪ · · · ∪ An = A.
74 Trần Nam Dũng (chủ biên)

6.2 Lời giải


Bài 6.1. Một người ham thích làm toán mỗi ngày làm một hoặc hai bài toán, nhưng
mỗi tuần làm không quá 10 bài. Chứng minh rằng có một số ngày liên tiếp người ấy
làm đúng 30 bài toán.

Lời giải. Gọi ai là số bài toán người đó làm trong ngày thứ i. Theo giả thiết, ai = 1
hoặc ai = 2, và do trong một tuần người đó làm không quá 10 bài, nên phải tồn tại
vô hạn k ∈ N sao cho ak = 1. (1)

Đặt Si = a1 + · · · + ai (quy ước S0 = 0). Như vậy, bài toán tương đương với việc
chứng minh tồn tại i 6= j sao cho S j = Si + 30 (khi i, j thỏa mãn điều kiện đó thì
ai+1 + · · · + a j = 30 và bài toán kết thúc). (∗)

Giả sử S j 6= Si + 30 ∀i, j ∈ N. Do các ai chỉ nhận giá trị 1 hoặc 2, nên Si+1 − Si ≤ 2.
Kết hợp với giả thiết phản chứng, suy ra tồn tại i, j ∈ N sao cho S j = Si + 31.

Ta sẽ chứng minh ai+1 = a j+1 = 2. Thật vậy,

+ Nếu ai+1 = 1, thì S j − Si+1 = 30 (vô lí). Do đó ai+1 = 2.

+ Nếu a j+1 = 1, thì S j+1 − Si+1 = 30 (vô lí). Do đó a j+1 = 2.

Vậy ta phải có ai+1 = a j+1 = 2. Quá trình được lặp lại với ai+2 , a j+2 , . . . , do đó
ak = 2 với mọi k ≥ i, tức số các chỉ số k sao cho ak = 1 là hữu hạn. (2)

Từ (1) và (2) rõ ràng là mâu thuẫn. Vậy điều giả sử là sai, hay tồn tại i, j sao cho
S j = Si + 30. Kết hợp với nhận xét (∗), chứng minh của bài toán kết thúc.

Bình luận. Ta có thể chứng minh kết luận của bài toán mà không dùng đến điều
kiện mỗi tuần không làm quá 10 bài. Cách làm vẫn là phản chứng. Ý tưởng cơ bản
là sẽ có một số ngày liên tiếp người này làm 29 bài toán, và sau đó sẽ là những ngày
làm hai bài.

Bài 6.2. Sau khi khai trương được đúng 10 ngày, một nhân viên thư viện cho biết

(1) Mỗi ngày có đúng tám người đến đọc sách;

(2) Không có người nào đến thư viện một ngày quá một lần ;

(3) Trong hai ngày bất kỳ của 10 ngày đó thì có ít nhất là 15 người khác nhau
cùng đến thư viện.

Căn cứ đồng thời cả ba điều kiện mà nhân viên thư viện cung cấp hãy cho biết số
người tối thiểu đã đến thư viện trong 10 ngày nói trên là bao nhiêu?
Lời giải và bình luận đề thi các tỉnh, các trường Đại học năm học 2009-2010 75

Lời giải. Gọi xi là số người đến đọc sách được i ngày (i = 1, 2, . . . , K) và K ≤ 10.
Gọi n là số người đã đến thư viện trong 10 ngày đó thì ta có

n = x1 + x2 + · · · + xK , (1)


80 = x1 + 2x2 + · · · + KxK . (2)
Gọi y là số cách chọn hai ngày sao cho không có người nào đến thư viện quá một
lần trong hai ngày đó. Vì trong hai ngày bất kỳ của 10 ngày có ít nhất là 15 người
khác nhau cùng đến thư viện, nên trong hai ngày bất kỳ của 10 ngày đó có không
quá một người đến thư viện trong cả hai ngày đó. Như vậy, ta có
2
C10 = C22 x2 +C32 x3 + · · · +CK2 xK + y. (3)

2 1 (i − 2)(i − 3) 2 1
Nhận xét rằng xi − ixi + Ci2 xi = xi ≥ 0. Lấy (1) − (2) + (3), ta
  3 3 6 3 3
115
có n ≥ + 1, suy ra n ≥ 39.
3

Vậy số người tối thiểu đi đến thư viện trong 10 ngày là 39. Bảng số liệu dưới đây
cho thấy giá trị này có thể đạt được (Ai là tập hợp chỉ số của những người đến thư
viện vào ngày thứ i).

A1 = {1, 3, 4, 5, 6, 7, 8, 9}, A2 = {1, 10, 11, 12, 13, 14, 15, 16},
A3 = {1, 17, 18, 19, 20, 21, 22, 23}, A4 = {2, 3, 10, 17, 24, 25, 26, 27},
A5 = {2, 4, 11, 18, 28, 29, 30, 31}, A6 = {2, 5, 12, 19, 32, 33, 34, 35},
A7 = {6, 13, 20, 32, 28, 24, 36, 37}, A8 = {7, 14, 21, 33, 29, 39, 26, 38},
A9 = {8, 15, 22, 34, 30, 39, 25, 36}, A10 = {9, 16, 23, 35, 31, 27, 38, 37}.

Bình luận. Bài này có thể phát biểu dưới ngôn ngữ tập hợp như sau: Cho A1 , A2 ,
. . . , A10 là các tập con của {1, 2, . . . , n} thoả mãn đồng thời các điều kiện
(i) |Ai | = 8 với mọi i = 1, 2, . . . , 10.

(ii) |Ai ∩ A j | = 1 với mọi i khác j.


Tìm giá trị nhỏ nhất của n để điều này có thể xảy ra.

Bài này được lấy trong tuyển tập đề thi Olympic 30/4 năm 2007. Tuy nhiên có lẽ là
nó cũng được lấy từ một tài liệu khác.
Bài 6.3. Chứng minh rằng nếu chọn ra 15 số bất kỳ từ tập hợp {2, 3, . . . , 2010}
sao cho chúng đôi một nguyên tố cùng nhau thì sẽ có ít nhất một số nguyên tố được
chọn.
76 Trần Nam Dũng (chủ biên)

Lời giải. Giả sử tồn tại 15 hợp số dương a1 < a2 < · · · < a15 thuộc đoạn [2, 2010]
sao cho chúng đôi một nguyên tố cùng nhau.

Gọi p1 , p2 , . . . , p15 là ước nguyên tố nhỏ nhất của a1 , a2 , . . . , a15 . Rõ ràng các số pi
phải đôi một phân biệt (ngược lại, nếu tồn tại i 6= j sao cho pi = p j thì (ai , a j ) ≥ pi ).

Gọi j là chỉ số sao cho p j = max{p1 , p2 , . . . , p15 } thì kiểm tra trực tiếp được
p j ≥ 47. Và khi đó, a j ≥ p2i ≥ 472 > 2010 (vô lí).

Vậy điều giả sử là sai và ta có điều phải chứng minh.


Bài 6.4. Cho n là một số nguyên dương. Chứng minh rằng với mọi dãy a1 , a2 , . . . ,
an ta luôn chọn được số tự nhiên k ≤ n sao cho

|(a1 + · · · + ak ) − (ak+1 + · · · + an )| ≤ max{|a1 |, |a2 |, . . . , |an |}.

(Đại học Khoa học tự nhiên)

Bình luận. Điều khó chịu nhất trong bài này là số lượng số hạng rất lớn trong dấu
trị tuyệt đối và sẽ rất dễ gây rối trong quá trình giải quyết. Ta sẽ tìm cách để giảm số
giá trị này đi.

Lời giải. Đặt S0 = 0, Si = a1 + · · · + ai , Ti = (a1 + · · · + ai ) − (ai+1 + · · · + an ) =


2Si − Sn và H = max{|a1 |, |a2 |, . . . , |an |}. Theo đề bài, ta cần chứng minh tồn tại
số k sao cho |Tk | ≤ H (đến đây, số số hạng trong dấu trị tuyệt đối chỉ còn là 2, và bài
toán đã gọn hơn nhiều).

Vì T0 = −Sn và Tn = Sn nên tồn tại chỉ số k < n sao cho Tk Tk+1 ≤ 0. Từ đây ta có

2H ≥ 2|ak+1 | = |Tk+1 − Tk | = |Tk+1 | + |Tk |,

suy ra ít nhất một trong hai bất đẳng thức |Tk+1 | ≤ H, |Tk | ≤ H đúng và ta có điều
phải chứng minh.
Bài 6.5. Giả sử ta có thể chọn được n số phân biệt từ tập {1, 2, 3, . . . , 2n − 1}
sao cho các số được chọn không có hai số nào chia hết cho nhau. Chứng minh rằng
không có số nào trong các số trên nhỏ hơn 2k , trong đó k là số xác định bởi điều
kiện 3k < 2n < 3k+1 .
(Đại học Khoa học tự nhiên)
Lời giải. Một lượng điều kiện khá ít và khá khó sử dụng . . . Cần được gỡ rối theo
từng bước.

Giả sử dãy số a1 < a2 < · · · < an thuộc đoạn [1, 2n − 1] thỏa mãn hai số bất kì không
chia hết cho nhau. Ta cần chứng minh a1 ≥ 2k .
Lời giải và bình luận đề thi các tỉnh, các trường Đại học năm học 2009-2010 77

Trước hết, đặt ai = 2bi · ci , trong đó ci lẻ. Từ giả thiết, ta suy ra được các ci phải đôi
một phân biệt và nằm trong đoạn [1, 2n − 1], tức (c1 , c2 , . . . , cn ) là một hoán vị của
(1, 3, . . . , 2n − 1).

Bây giờ xét hai trường hợp sau.

+ c1 = 1 (c1 = 3 cách giải tương tự). Khi đó, a1 = 2b1 . Xét dãy s với si = 2ti · 3i . Rõ
ràng dãy s là một phần của dãy a ban đầu, nên nó cũng phải thỏa mãn các tính chất
của dãy a, tức hai số bất kì trong dãy s không chia hết cho nhau. Như vậy dãy t là
một dãy mà các phần tử đôi một phân biệt và nhỏ hơn b1 , và do phần tử lớn nhất
trong dãy s là sk = 3k nên b1 ≥ k, tức a1 ≥ 2k .

+ c1 ≥ 5. Giả sử a1 < 2k . Như thế ta có 2b1 · c1 < 2k , suy ra 5 ≤ c1 < 2k−b1 , hay
k − b1 ≥ 3. Mặt khác, ta lại có

c1 · 3b1 +1 < 2k−b1 · 3b1 < 9 · 2k−b1 −3 · 3b1 +1 < 32 · 3k−b1 −3 · 3b1 +1 = 3k ,

nên tồn tại dãy s0 , s1 , . . . , sb1 +1 là dãy con của dãy a sao cho si = 2ti · 3i · c1 . Lưu ý
rằng t0 = b1 . Do s là dãy con của a nên hai phần tử bất kì trong s không chia hết cho
nhau. Theo đó, các phần tử t1 , t2 , . . . , tb1 +1 phải đôi một phân biệt và nhỏ hơn b1 ,
mà rõ ràng điều này là vô lí. Vậy điều giả sử là sai, hay ta có a1 ≥ 2k .

Bài 6.6. Cho tập hợp X = {1, 2, . . . , 2010}. Tìm số nguyên N lớn nhất sao cho
mỗi hoán vị ωX = (a1 , a2 , . . . , a2010 ) của X đều tồn tại 30 số hạng liên tiếp có tổng
không nhỏ hơn N.
(Đại học Sư phạm)

Lời giải. Trước hết, ta tìm giá trị lớn nhất của N. Xét một hoán vị (a1 , a2 , . . . , a2010 )
của (1, 2, . . . , n). Theo giả thiết, mỗi nhóm 30 số hạng liên tiếp đều có tổng không
nhỏ hơn N, nên

1 + · · · + 2010 = (a1 + · · · + a30 ) + (a31 + · · · + a60 ) + · · · + (a1981 + · · · + a2010 )


≥ 67N.

Từ đây suy ra
2010 · 2011
N≤ = 30165.
67
Bây giờ ta chỉ ra một hoán vị sao cho không thể thay N bằng số nhỏ hơn. Thật vậy,
kiểm tra trực tiếp hoán vị (2010, 1, 2009, 2, . . . , 1006, 1005) là hoán vị đảm bảo
không thể thay N bởi số nhỏ hơn.

Tóm lại ta có giá trị lớn nhất của N là 30165.


78 Trần Nam Dũng (chủ biên)

Bài 6.7. Tập hợp các số nguyên dương được tô bởi hai màu đen và trắng. Giả thuyết
rằng, tổng của hai số khác màu luôn bị tô màu đen và có vô hạn số bị tô màu trắng.
Chứng minh rằng tổng và tích của hai số bị tô màu trắng cũng sẽ bị tô màu trắng.
(Đồng Nai)

Lời giải. Bài này đã từng xuất hiện trong kì thi VMEO 2006, và lời giải của nó như
sau.

Do tập các số nguyên dương bị chặn dưới, và tồn tại vô số số được tô màu trắng, nên
tồn tại số nguyên dương bé nhất được tô màu trắng. Kí hiệu số đó là p.

+ Bước 1. Ta sẽ chứng minh bất kì số nào được tô màu trắng cũng là bội của p. Thật
vậy, giả sử tồn tại số k = l p + r (0 < r < p) được tô màu trắng. Khi đó, theo cách
chọn số p, ta có p < k. Mặt khác, do k = p + (k − p) và k, p cùng có màu trắng, nên
k − p cũng phải có màu trắng (ngược lại thì k sẽ có màu đen).

Tiếp tục như vậy, ta sẽ có các số k − 2p, k − 3p, . . . , k − l p có màu trắng, tức r
có màu trắng. Nhưng do 0 < r < p nên điều này trái với cách chọn p là số nguyên
dương bé nhất được tô màu trắng. Do vậy, tất cả các số được tô màu trắng đều phải
là bội của p.

+ Bước 2. Ta sẽ chứng minh bất kì số nào là bội của p đều được tô màu trắng. Thật
vậy, giả sử tồn tại số k = pq có màu đen. Khi đó,

(q + 1)p = qp + p có màu đen;

(q + 2)p = (q + 1)p + p có màu đen;

···

Như vậy, tất cả các số lớn hơn k và là bội của p đều có màu đen. Mặt khác, theo
bước 1 thì tất cả các số màu trắng đều là bội của p. Như vậy, số lượng số màu trắng
phải là hữu hạn, trái giả thiết số lượng số màu trắng là vô hạn. Vậy điều giả sử là sai
và bước 2 được giải quyết.

Từ hai bước trên ta có kết luận rằng, một số có màu trắng khi và chỉ khi số đó là bội
của p. Từ đây, kết luận của bài toán là hiển nhiên.

Bài 6.8. Cho tập hợp S = {1, 2, 3, . . . , n}. Tìm số cách chia tập S thành ba tập con
khác rỗng sao cho mỗi tập con không chứa hai số nguyên liên tiếp.
(Đại học Sư phạm)
Lời giải và bình luận đề thi các tỉnh, các trường Đại học năm học 2009-2010 79

Lời giải. Kí hiệu S(n) là số cách chia tập S thành ba tập con không chứa khác rỗng
mà bất kì tập con nào cũng không chứa hai phần tử liên tiếp nhau. Ta sẽ tìm cách
tính S(n + 1) theo S(n).

Giả sử ta đã chia được ba tập con và tổng số phần tử của chúng là n. Bổ sung thêm
phần tử n + 1. Ta có hai khả năng xảy ra.

+ Khả năng 1. n + 1 không tạo thành một tập con mới (tức tập chứa n + 1 có ít nhất
một phần tử khác). Khi đó, rõ ràng ta có hai cách bổ sung n + 1 (vào một trong hai
tập không chứa n). Vậy số cách xây dựng tập con trong trường hợp này là 2S(n).

+ Khả năng 2. n + 1 tạo thành một tập con mới. Khi đó, n số từ 1 đến n phải nằm
trong hai tập hợp còn lại. Có thể thấy ngay chỉ có một cách chia thỏa mãn (một tập
chứa các số chẵn và tập còn lại chứa các số lẻ). Do đó, số cách trong trường hợp này
là một cách.

Vậy ta thu được công thức truy hồi

S(n + 1) = 2S(n) + 1,

hay
S(n + 1) + 1 = 2[S(n) + 1]. (∗)
Mặt khác, kiểm tra trực tiếp ta có S(3) = 1. Kết hợp với (∗), ta dễ dàng tìm được
công thức tổng quát của S(n) là S(n) = 2n−2 − 1 với n ≥ 3.

Như vậy, số cách chia tập hợp thỏa mãn đề bài là S(1) = S(2) = 0 và S(n) = 2n−2 − 1
với n ≥ 3.

Bình luận. Bài này không khó trong việc lập công thức truy hồi, nhưng lại rất dễ
sai (vì quên rằng n + 1 có thể tự lập thành một tập mới). Để tránh bị bỏ sót, nên tính
trực tiếp một số giá trị đầu tiên.

Bài 6.9. Cho n là số nguyên lớn hơn 1 và P1 , P2 , . . . , Pn là các tập con có hai phần
tử và đôi một phân biệt của tập hợp S = {1, 2, 3, . . . , n} thỏa mãn tính chất: nếu
i 6= j mà Pi ∩ Pj 6= 0/ thì tồn tại k để Pk = {i, j}. Chứng minh rằng với mỗi số i ∈ S
xuất hiện đúng hai lần trong các tập Pj với j = 1, 2, . . . , n.

Lời giải. Với mỗi 1 ≤ i ≤ n, kí hiệu xi là số lần xuất hiện của số i trong các tập hợp.
Theo giả thiết, ta có x1 + x2 + · · · + xn = 2n.

Cũng theo giả thiết, nếu |Pi ∩ Pj | =


6 0 thì tồn tại Pk = {i, j}. Và rõ ràng, ứng với mỗi
bộ (i, j) mà |Pi ∩ Pj | 6= 0 (không kể thứ tự của i và j) thì các Pk là phân biệt. Cho
nên số các bộ (i, j) mà |Pi ∩ Pj | 6= 0 không vượt quá n.
80 Trần Nam Dũng (chủ biên)

Nhưng, số bộ số này lại bằng


n
(x12 + · · · + xn2 ) − (x1 + · · · + xn )
∑ Cx2i = 2
i=1
(x1 + · · · + xn )2 x1 + · · · + xn
≥ − ≥ 2n − n = n.
2n 2

Như vậy dấu bằng trong bất đẳng thức trên phải xảy ra, tức x1 = · · · = xn = 2 và đây
chính là điều phải chứng minh.

Bài 6.10. Cho số nguyên n không nhỏ hơn 3. Giả sử mỗi số nguyên dương không
lớn hơn Cn1 +Cn2 +Cn3 được tô một trong hai màu xanh hoặc đỏ. Chứng minh tồn tại
dãy các số cùng màu thỏa mãn

(1) x1 < x2 < · · · < xn ;

(2) x2 − x1 ≤ x3 − x2 ≤ · · · ≤ xn − xn−1 ≤ Cn2 .

Lời giải. Ta sẽ chứng minh bài toán bằng quy nạp. Giả sử mệnh đề của bài toán đã
đúng tới n, tức luôn tồn tại dãy 1 ≤ x1 < x2 < · · · < xn ≤ Cn1 +Cn2 +Cn3 có cùng màu
và thỏa mãn x2 − x1 ≤ x3 − x2 ≤ · · · ≤ xn − xn−1 ≤ Cn2 . Không mất tính tổng quát,
giả sử n số này có màu đỏ. Xét mệnh đề với n + 1.

+ Giả sử tồn tại số xn+1 cũng có màu đỏ và Cn2 ≤ xn+1 − xn ≤ Cn+1


2 . Khi đó,

2 2
xn+1 ≤ Cn+1 + xn ≤ Cn+1 +Cn1 +Cn2 +Cn3 = Cn+1
1 2
+Cn+1 3
+Cn+1 − 1,

nên số xn+1 thỏa mãn toàn bộ các điều kiện để được bổ sung vào dãy, tức ta có dãy
x1 , x2 , . . . , xn+1 thỏa mãn đề bài.

+ Nếu tất cả các số xn + i, với Cn2 ≤ i ≤ Cn+1


2 đều có màu xanh, thì chú ý rằng
2 −C 2 = (n + 1)n n(n − 1)
Cn+1 n − = n, nên dãy xn + i gồm n + 1 số và ngay lập tức
2 2
thỏa mãn tất cả các điều kiện của bài toán.

Vậy mệnh đề đúng với n + 1. Theo đó, ta chỉ cần chứng minh bài toán với n = 3, tức
cần chứng minh tồn tại ba số x1 , x2 , x3 sao cho

(1) x1 , x2 , x3 cùng màu.

(2) 1 ≤ x1 < x2 < x3 ≤ 7.

(3) x2 − x1 ≤ x3 − x2 ≤ 3.
Lời giải và bình luận đề thi các tỉnh, các trường Đại học năm học 2009-2010 81

Ta sử dụng phản chứng. Giả sử tồn tại cách tô sao cho không tìm được ba số như
vậy. Không mất tính tổng quát, giả sử 2 được màu đỏ. Nếu 2 được tô màu đỏ, dẫn
đến cả 3, 4, 5 đều có màu xanh (nếu không, thì số được tô màu đỏ sẽ hợp với 1, 2
tạo thành dãy thỏa mãn đề bài). Nhưng khi đó, (3, 4, 5) lại thỏa mãn cả ba điều kiện
trên, mâu thuẫn. Do vậy 2 có màu xanh.

Bây giờ giả sử 3 có màu đỏ. Lí luận tương tự như trên, ta được 5 và 6 có màu xanh,
4 và 7 có màu đỏ, dẫn đến dãy (1, 4, 7) thỏa mãn cả ba điều kiện trên, mâu thuẫn.
Do vậy 3 có màu xanh. Khi đó, 4, 5, 6 có màu đỏ và dãy (4, 5, 6) lại thỏa mãn cả
ba điều kiện kể trên.

Như vậy, điều mà ta đã giả sử ở trên là sai. Hay nói một cách khác, bài toán đúng
với n = 3. Phép chứng minh được hoàn tất.

Bài 6.11. Cho tập hợp A gồm n ≥ 5 phần tử. Xét k tập con bất kì gồm ba phần tử
của A. Hãy tìm số k nhỏ nhất sao cho với mọi cách chọn k tập con trên luôn tồn tại
hai tập con có chung nhau đúng một phần tử.
(Đại học Khoa học tự nhiên)

Lời giải. Một cách tự nhiên, không ít bạn đã nghĩ như sau: Chọn các tập Ai−2 =
(1, 2, i) với 3 ≤ i ≤ n. Ta chọn được n − 2 tập như thế, và khi chọn thêm một tập
nữa thì luôn có hai tập giao nhau tại đúng một phần tử. Nên dự đoán số tập là n − 1
và tìm cách đi chứng minh nó. Bây giờ, cần kiểm chứng lại xem điều đó có đúng
hay không . . .

Ta kí hiệu các tập con của A là A1 , A2 , . . . , AKn và đánh số các phần tử của tập A từ
1 đến n (trong đó, Kn là giá trị tốt nhất để tồn tại Kn tập hợp, mà hai tập hợp bất kì
giao nhau tại 0 hoặc 2 phần tử). Ta sẽ chứng minh Kn = Kn−4 + 4.

Theo bước suy luận ở trên, rõ ràng ta có Kn ≥ n − 1. Theo đó, do |Ai | = 3 nên tồn
tại một phần tử thuộc vào ít nhất ba tập hợp. Không mất tính tổng quát, giả sử phần
tử đó là 1, và nó thuộc vào ba tập hợp A1 , A2 , A3 .

Theo giả thiết, |A1 ∩ A2 | = 2, nên tồn tại một phần tử khác (kí hiệu là 2) thuộc vào
cả hai tập hợp này, tức là A1 = {1, 2, 3} và A2 = {1, 2, 4}. Xét tập hợp A3 , ta có hai
trường hợp nhỏ.

+ Trường hợp 1. 2 ∈ A3 . Khi đó, A3 = {1, 2, 5}, và do đó, nếu một tập Ai nào đó có
phần tử chung với ít nhất một trong ba tập A1 , A2 , A3 , nó sẽ phải có dạng (1, 2, i).
Như thế, số tập tối đa có thể chọn là max{n − 1, Kn−6 + 3} ≤ max{n − 1, Kn−4 + 3}.

+ Trường hợp 2. 2 6∈ A3 . Khi đó, A3 = (1, 3, 4), và kiểm tra trực tiếp cho thấy nếu
Ai có phần tử chung với ít nhất một trong ba tập đó thì nó chỉ có dạng duy nhất là
82 Trần Nam Dũng (chủ biên)

(2, 3, 4). Như thế, Kn − 4 tập còn lại sẽ là tập con của {5, 6, . . . , n}, và như thế thì
số tập con tối đa trong tình huống này là Kn−4 + 4.

Kết hợp cả hai trường hợp này ta được Kn = Kn−4 + 4. Vậy ta chỉ cần xét bài toán
tại trường hợp cơ sở là n = 5, 6, 7, 8, thậm chí có thể xét ngay tại trường hợp n = 1,
2, 3, 4. Kiểm tra trực tiếp cho thấy K1 = K2 = 0, K3 = 1, K4 = 4. Do đó, giá trị lớn
nhất của Kn sẽ là 
n
 với n ≡ 0 (mod 4)
Kn = n − 1 với n ≡ 1 (mod 4) .

n − 2 với n ≡ 2, 3 (mod 4)

Và việc chỉ ra các tập hợp thỏa mãn đề bài xin được dành cho các bạn (chú ý đến
các bước chứng minh phía trên để dựng).

Tóm lại, ta có kết luận giá trị nhỏ nhất của m để trong m tập con bất kì của A luôn
có hai tập giao nhau tại đúng một phần tử là

 n + 1 với n ≡ 0 (mod 4)

m= n với n ≡ 1 (mod 4) .

n − 1 với n ≡ 2, 3 (mod 4)

Bài toán được giải quyết hoàn toàn.

Bài 6.12. Cho tập S = {1, 2, 3, . . . , 2009}. A là tập con có n phần tử của S. Tìm n
a
nhỏ nhất sao với mọi cách chọn tập A thì trong A luôn có hai phần tử a, b mà = 3.
b
(Bắc Ninh)

Lời giải. Với mỗi số nguyên dương x, ta kí hiệu v(x) là số nguyên không âm lớn nhất
thỏa mãn 3v(x) | x (nói cách khác, nếu đặt v(x) = k thì x chia hết cho 3k và không
chia hết cho 3k+1 ). Bây giờ, ta chia tập S thành các tập con Si thỏa mãn

Si = {x ∈ S | v(x) = i}.

Ta chứng minh rằng số phần tử được chọn là nhiều nhất nếu ta chọn các tập S0 ,
S2 , S4 , . . . Thật vậy, với mỗi số nguyên dương k không chia hết cho 3, xét tập Tk =
{k, 3k, 32 k, . . .}. Rõ ràng các tập Tk phủ kín S.

Xét một giá trị k bất kì. Khi đó, trong tập A được chọn sẽ không tồn tại hai phần tử
liên tiếp cùng nằm trong Tk . Như thế, nếu đặt s là số nguyên  không
 âm lớn nhất sao
s s+1
cho 3 · k < 2009 thì ta chỉ có thể chọn được nhiều nhất phần tử thuộc Tk ,
2
và giá trị đó đạt được nếu ta chọn các số k, 32 k, 34 k, . . .
Lời giải và bình luận đề thi các tỉnh, các trường Đại học năm học 2009-2010 83

Do mệnh đề trên đúng với số k nguyên dương bất kì, nên gộp lại, số phần tử được
chọn sao cho không có phần tử nào nhiều gấp ba lần phần tử kia nằm trong các tập
S0 , S2 , . . . Tính toán trực tiếp, ta có

|S0 | = 1340, |S1 | = 446, |S2 | = 149, |S3 | = 50 |S4 | = 16,

|S5 | = 6, |S6 | = 2, |Si | = 0 ∀i ≥ 7.


Và do đó, nếu ta chọn 1507 phần tử nằm trong các tập S0 , S2 , S4 , S6 thì bất kì hai
phần tử a, b phân biệt nào thuộc S cũng đều thỏa mãn a 6= 3b. Vậy tập hợp A cần có
ít nhất 1508 phần tử, hay giá trị nhỏ nhất của n là 1508.
Bài 6.13. Cho tập A = {1, 2, 3, . . . , 2009}. Chứng minh rằng, có thể tô màu mỗi
phần tử của tập A bằng một trong hai màu đen trắng sao cho mọi cấp số cộng công
sai khác 0 gồm 18 phần tử của A đều được tô bởi đủ cả hai màu.
(Hải Phòng)
Lời giải. Rất có thể một số bạn sẽ cố gắng tìm một cách tô (xây dựng cấu hình cụ
thể) cho bài toán, và sẽ rất nhanh chóng cảm thấy khó khăn với giá trị rất lớn của
các con số. Một số khác sẽ đi tìm một phương án tiếp cận nhẹ nhàng hơn . . .

Do mỗi màu chỉ được tô bới một trong hai màu đen hoặc trắng, nên số cách tô màu
2009 số này là 22009 . Ta đếm số cách tô sao cho có một cấp số cộng gồm 18 số cùng
màu. Rõ ràng, ta sẽ tô 18 số này cùng màu, và tô màu tùy ý cho 1991 số còn lại.
Như vậy, số cách tô dẫn đến xuất hiện một cấp số cộng có 18 số cùng màu sẽ nhỏ
hơn 2a · 21991 = a · 21992 , do chắc chắn sẽ có những cách trùng nhau trong các cách
tô này (ở đây, kí hiệu a là số lượng cấp số cộng gồm 18 số).

Bài toán sẽ được giải quyết nếu ta chứng minh được số cách tô xuất hiện cấp số cộng
cùng màu nhỏ hơn tổng số cách tô. Khi đó sẽ có một cách tô mà không có 18 số
cùng màu nào lập thành một cấp số cộng). Tức ta cần chứng minh a · 21992 < 22009 ,
hay là a < 217 .

Bây giờ ta đếm số lượng cấp số cộng. Xét một cấp số cộng x, x + d, x + 2d, . . . ,
2009 − x
x + 17d, trong đó x, d > 0 và x + 17d ≤ 2009. Tức là d ≤ . Do x là một số
17
bất kì trong [1, 2009], nên bất đẳng thức của d cho ta
2009
2009 − k 2009 · 2010
a≤ ∑ = < 217 .
k=1 17 17 · 2

Điều kiện a < 217 được thỏa mãn và phép chứng minh kết thúc.

Bình luận. Phiên bản đầu tiên của bài toán này xuất hiện trong IMO Shortlist 1987
do Romania đề nghị, sau đó được chọn là bài 6 (bài khó nhất) của German TST
84 Trần Nam Dũng (chủ biên)

1988. Bài toán gốc được phát biểu như sau: Chứng minh rằng có thể tô màu các số
nguyên dương từ 1 đến 1987 bởi bốn màu sao cho mỗi số được tô bởi đúng một màu
và không tồn tại bất kì cấp số cộng nào chứa 10 số cùng màu.

Các bạn có thể tham khảo lời giải của bài này trong cuốn IMO Compendium (trang
496 - 497) hoặc Polya’s Footsteps (trang 146 - 147) (tuy nhiên cũng khá giống với
lời giải ở trên).
Bài 6.14. Tập hợp A ⊂ R được gọi là có tính chất T nếu A có không ít hơn bốn phần
tử và ab + cd thuộc A với mọi a, b, c, d phân biệt thuộc A.
(a) Hãy chỉ ra tập hợp A gồm bốn phần tử, có tính chất T.

(b) Có hay không tập hợp A ⊂ (0, +∞) gồm bốn phần tử và có tính chất T.
(Đại học Sư phạm)
Lời giải. Ta sẽ giải quyết câu (b) trước (điều đó sẽ giúp giải quyết câu (a) một cách
nhanh gọn). Giả sử tồn tại bốn số thực a, b, c, d thỏa mãn đề bài. Xét các đẳng thức

(ad + bc) − (ac + bd) = a(d − c) + b(c − d) = (b − a)(c − d) < 0,

(ab + cd) − (ad + bc) = a(b − d) + c(d − b) = (a − c)(b − d) > 0.


Do vậy
ad + bc < ac + bd < ab + cd.
Mặt khác, tất cả ba số kể trên đều thuộc vào tập T, nên chỉ có các trường hợp sau
xảy ra.

+ Trường hợp 1. ad + bc = b. Khi đó ta có ac + bd = c và ab + cd = d. Như vậy

b − d = (ad + bc) − (ab + cd) = (c − a)(b − d),

suy ra c − a = 1. Và như thế ta tìm được

0 = (ab + cd) − d = ab + d(c − 1) = a(b + d),

nhưng rõ ràng điều này vô lí vì 0 < a < b < c < d. Vậy trường hợp này bị loại.

+ Trường hợp 2. ad + bc = a.

- Nếu ab + cd = c, thì ac + bd = b và ta giải quyết tương tự trường hợp trên.

- Xét trường hợp ab + cd = d, lúc này ta có thể cho ac + bd = b (trường hợp kia
được giải quyết hoàn toàn tương tự). Khi đó

a − b = (ad + bc) − (ac + bd) = (b − a)(c − d).


Lời giải và bình luận đề thi các tỉnh, các trường Đại học năm học 2009-2010 85

Từ đây ta tìm được d − c = 1, dẫn tới

0 = (ac + bd) − b = ac + b(d − 1) = c(a + b),

và ta lại thu được điều vô lí.

Tóm lại, không tồn tại bốn số 0 < a < b < c < d thỏa mãn đề bài.

 kể trên, tasẽ thử cho a = 0. Khi đó c = 1,


Bây giờ trở lại câu (a). Từ cách xác định
1
bd = 1. Và ta có thể cho(a, b, c, d) = 0, , 1, 2 là thu được bộ số thỏa mãn đề
2
bài.
Bài 6.15. Cho số nguyên dương n > 10. Tìm m ∈ N∗ lớn nhất thỏa mãn điều kiện:
Tồn tại m tập con A j của tập A = {1, 2, 3, . . . , 2n}, mỗi tập con gồm n phần tử sao
cho |Ai ∩ A j ∩ Ak | ≤ 1 với mọi 1 ≤ i < j < k ≤ n.
(Đại học Khoa học tự nhiên)
Lời giải. Quá trình chứng minh được thực hiện theo bốn bước như sau.

+ Bước 1. m ≤ 8. Để có điều này, ta sẽ chứng minh mỗi phần tử thuộc vào không
quá bốn tập hợp. Ngược lại, giả sử tồn tại một phần tử (kí hiệu là 1) thuộc vào năm
tập hợp khác nhau (kí hiệu là A1 , A2 , . . . , A5 ). Xét các tập Bi = Ai \{1}. Khi đó, theo
giả thiết ta được |Bi ∩ B j ∩ Bk | = 0 với mọi 1 ≤ i < j < k ≤ 5, tức mỗi phần tử từ 2
đến 2n chỉ thuộc vào nhiều nhất hai tập B. Do vậy số phần tử tối đa của năm tập này
là 2(2n − 1). (1)

Mặt khác, |Bi | = n − 1, nên năm tập này có tổng số phần tử là 5(n − 1). (2)

Từ (1) và (2), ta suy ra 5(n − 1) ≤ 2(2n − 1), hay n ≤ 3 (vô lí). Vậy điều giả sử là
sai, hay mỗi phần tử chỉ thuộc vào tối đa bốn tập hợp. Như vậy, m tập hợp có tối đa
8n phần tử, tức m ≤ 8.

+ Bước 2. m ≤ 6. Gọi K4 là số các phần tử thuộc vào đúng bốn tập hợp. Kết hợp với
giả thiết cùng với kiểm tra trực tiếp, ta được k ≤ 6, tức tổng số phần tử của m tập hợp
sẽ không vượt quá 6 · 4 + (2n − 6) · 3 = 6n + 6. Như thế, mn ≤ 6n + 6, hay m ≤ 6.

+ Bước 3. m ≤ 5. Định nghĩa K3 tương tự như K4 . Theo giả thiết, ta có nhận xét: giá
trị lớn nhất của K3 chính là số cách chọn ba số khác nhau trong sáu số từ 1 đến 6
(và do đó, K3 ≤ C63 = 20). Do m ≤ 6 nên K4 ≤ 3 (K4 kí hiệu ở bước 2). Ngoài ra, để
ý rằng, nếu có một phần tử i thuộc vào bốn tập hợp (chẳng hạn là A1 , A2 , A3 , A4 ),
thì sẽ không có phần tử j nào khác thuộc vào (Ai , A j , Ak ) (1 ≤ i < j < k ≤ 4), tức
K3 phải giảm đi bốn phần tử. Như vậy, tổng số phần tử lớn nhất của các tập hợp sẽ
là 4.K4 + (20 − 4K4 ) · 3 + (2n − 20) · 2 ≤ 4n + 20, hay mn ≤ 4n + 20. Kết hợp với
n > 10 (bây giờ mới thấy được tại sao n > 10), ta phải có m ≤ 5.
86 Trần Nam Dũng (chủ biên)

+ Bước 4. m ≤ 4. Tiếp tục đánh giá tương tự bước 3, ta được

mn ≤ 4K4 + (10 − 4K4 ) · 3 + (2n − 10) · 2 ≤ 4n + 10.

Từ đó suy ra m ≤ 4.

Cuối cùng, ta sẽ dựng bốn tập hợp thỏa mãn đề bài. Xét các tập sau đây

A1 = {1, 2, . . . , n}, A2 = {n + 1, n + 2, . . . , 2n},


A3 = {1, 3, . . . , 2n − 1}, A4 = {2, 4, . . . , 2n},

thì rõ ràng tất cả các điều kiện của bài toán được thỏa mãn. Vậy giá trị lớn nhất của
m là 4.

Bình luận. Các bước đánh giá m trong bài này thể hiện vẻ đẹp của toán rời rạc:
luôn biến ảo đến khó lường, và luôn phải nhạy cảm để phát hiện những mấu chốt
cuối cùng. Điều để lại ấn tượng nhiều nhất khi làm bài này là việc sử dụng các bất
đẳng thức yếu hơn để chứng minh các bất đẳng thức mạnh hơn - một điều không
hề thường gặp trong các bài toán của chúng ta, cũng là thử thách cho sự kiên trì và
nhạy bén trong tất cả các tình huống có thể xảy ra.

Bài này được ra trong đề thi APMC 2001 (Austrian-Polish Mathematical Competi-
tion).
Bài 6.16. Cho A = {1, 2, . . . , 2n}. Một tập con của A được gọi là tốt nếu nó có
đúng hai phần tử x, y và |x − y| ∈ {1, n}. Tìm số các tập hợp {A1 , A2 , . . . , An } thoả
mãn điều kiện Ai là tập con tốt với mọi i = 1, 2, . . . , n và A1 ∪ A2 ∪ · · · ∪ An = A.
(Phổ thông Năng khiếu)
Lời giải. Từ giả thiết, ta sẽ viết lại bài toán như sau (các bạn tự kiểm tra tính tương
đương của bài toán này so với bài ban đầu): “Cho một hình chữ nhật kích thước 2 × n
được chia thành các ô vuông đơn vị. Đánh số các ô từ trái qua phải là 1, 2, . . . , n
(hàng 1) và n + 1, n + 2, . . . , 2n (hàng 2). Lát chúng bằng các quân domino 1 × 2
sao cho chúng phủ kín hình chữ nhật và không có hai quân nào đè lên nhau. Ngoài
ra, với n lẻ, ta được bổ sung thêm một quân domino “đặc biệt” có thể phủ kín hai ô
n và n + 1. Đếm số cách lát thỏa mãn đề bài”.

Với bài toán này, xét Sn là số cách lát thỏa mãn đề bài với hình chữ nhật kích thước
2 × n. Ta sẽ tìm cách xây dựng công thức truy hồi cho Sn .

Giả sử ta đã lát được hình chữ nhật 2 × (n + 1) bằng các quân domino. Xét quân
domino phủ lên ô vuông n. Có ba khả năng xảy ra.

+ Khả năng 1. Quân domino đó phủ lên hai ô (n, 2n). Rõ ràng phần còn lại là một
hình chữ nhật kích thước 2 × n, và số cách lát trong tình huống này là Sn .
Lời giải và bình luận đề thi các tỉnh, các trường Đại học năm học 2009-2010 87

+ Khả năng 2. Quân domino đó phủ lên hai ô (n, n + 1). Như vậy, buộc phải có một
quân domino phủ lên hai ô (2n − 1, 2n) và khi đó, phần còn lại là một hình chữ nhật
kích thước 2 × (n − 1). Tức số cách lát trong tình huống này là Sn−1 .

+ Khả năng 3. Quân domino đó phủ lên hai ô (n, n + 1) (với n lẻ). Khi đó, phần còn
lại chỉ có thể lát được bằng các quân domino nằm ngang (nếu có một quân domino
nào nằm dọc thì nó sẽ chia hình chữ nhật thành hai phần, mỗi phần có một số lẻ ô
chưa được lát (do quân domino “đặc biệt” gây ra)). Tức trong trường hợp này chỉ có
một cách lát duy nhất.

Như vậy ta xây dựng được công thức truy hồi như sau

S2k = S2k−1 + S2k−2 − 1 (lưu ý rằng khi n chẵn thì không có quân domino “đặc
biệt” nên phải bớt đi một cách của S2k−1 ).

S2k+1 = S2k + S2k−1 (lập luận tương tự với quân domino “đặc biệt”).

Và bằng quy nạp ta sẽ thu được S2k = F2k , S2k+1 = F2k+1 + 1, trong đó Fk là số
Fibonacci thứ k của dãy Fibonacci được xác định bởi công thức

F0 = F1 = 1, Fn+2 = Fn+1 + Fn .

Cuối cùng ta được công thức tổng quát


" √ !n √ !n #
1 1+ 5 1− 5 1 − (−1)n
Sn = √ − + .
5 2 2 2

Bình luận. Đây chỉ là biến thể của một bài toán quen thuộc: “Có bao nhiêu cách lát
đường đi 2 × n bằng các quân domino?”. Tuy nhiên cách phát biểu làm cho bài toán
khó hình dung hơn. Đây cũng là một đặc điểm rất điển hình của bài toán tổ hợp:
cùng một bài toán có thể có nhiều cách phát biểu khác nhau, và quan trọng là ta phải
tìm được cách phát biểu thuận lợi cho lời giải.
88 Trần Nam Dũng (chủ biên)
Chương 7

Dãy số

“Toán học là cánh cửa và là chìa khóa để đi vào các ngành khoa học khác.”
Roger Bacon

7.1 Đề bài
1 3 1
7.1. Cho dãy (un ) xác định bởi u1 = và un+1 = u2n − u3n . Chứng minh rằng dãy
2 2 2
(un ) có giới hạn. Tìm giới hạn đó.

7.2. Chứng minh rằng với mỗi số nguyên dương n cho trước thì phương trình x2n+1 =
x + 1 có đúng một nghiệm thực. Gọi nghiệm đó là xn . Tính lim xn .
n→∞

7.3. Cho dãy số (an ) xác định bởi a1 = a2 = 12 2 và
q q
an+1 = −an a2n−1 + 1 + an−1 a2n + 1
q p
với mọi n ≥ 2. Chứng minh rằng 2 + 2 a2n + 1 là số nguyên dương với mọi n ≥ 1.

7.4. Giả sử dãy số nguyên dương {an } thỏa mãn a4 = 4 và

1 1 1 (n + 3)an
+ +···+ =
a1 a2 a3 a2 a3 a4 an an+1 an+2 4an+1 an+2

với mọi n nguyên, n ≥ 2. Chứng minh rằng an = n với mọi n nguyên dương.
xn2 + 3
7.5. Cho dãy số {xn } xác định bởi x1 = 3 và xn+1 = với mọi n ≥ 1. Chứng
3xn
minh rằng dãy {xn } có giới hạn và tìm giới hạn đó.

89
90 Trần Nam Dũng (chủ biên)

7.6. Cho dãy số nguyên {an } thỏa mãn các điều kiện

(1) |an+2 − an | ≤ 2 với mọi n thuộc N∗ ;

(2) Tồn tại dãy {bn } sao cho am + an = bm2 +n2 với mọi m, n thuộc N∗ .

Chứng minh rằng trong bảy số hạng bất kì của {an } luôn tồn tại hai số hạng bằng
nhau.

7.7. Cho dãy số (xn ) được xác định như sau

2 xn
x1 = , xn+1 = ∀n ≥ 1.
3 2(2n + 1)xn + 1
n
Tính lim
n→∞
∑ xi .
i=1

7.8. Cho dãy số (xn ) thỏa mãn

x1 + 2x2 + · · · + (n − 1)xn−1
x1 = 2, xn = ∀n ≥ 2.
n(n2 − 1)

Tìm lim un với un = (n + 1)3 xn .


n→∞

7.9. Cho dãy số thực {xn } xác định bởi x1 = 1, xn+1 = 2xn với n ≥ 1. Chứng minh
dãy số {xn } có giới hạn. Tìm giá trị giới hạn này.

7.10. Tìm công thức tổng quát của dãy số sau

u1 = 12 , u2 = 22 + 42 , u3 = 32 + 52 + 72 , ...
5Un + 4
7.11. Cho dãy số (Un ) xác định bởi U1 = 5, Un+1 = với mọi n ∈ N∗ .
Un + 2
(1) Chứng minh rằng Un > 4 với mọi n thuộc N∗ .

(2) Tìm số hạng tổng quát của (Un ).

7.12. Cho ba số nguyên không âm a, b, c thỏa mãn 2


p điều kiện a = b + 1. Dãy số
2 2
(un ) được xác định như sau u0 = 0, un+1 = aun + bun + c với mọi n ∈ N. Chứng
minh mọi số hạng của dãy đều nguyên.
Lời giải và bình luận đề thi các tỉnh, các trường Đại học năm học 2009-2010 91

7.2 Lời giải


1 3 1
Bài 7.1. Cho dãy (un ) xác định bởi u1 = và un+1 = u2n − u3n . Chứng minh rằng
2 2 2
dãy (un ) có giới hạn. Tìm giới hạn đó.
(Ninh Bình)

Lời giải. Trước hết ta chứng minh 0 < un+1 < un < 1. Ta sẽ sử dụng phép quy nạp
5 1
theo n. Ta có 0 < u2 = < = u1 < 1 nên khẳng định của ta đúng với n = 1. Giả
16 2
sử khẳng định đúng với n = k, khi đó xét

3 1 uk (uk − 1)(uk − 2)
uk+1 − uk = u2k − u3k − uk = − < 0.
2 2 2
u2k (3 − uk )
Như vậy uk+1 < uk < 1 và uk+1 = > 0 nên khẳng định được chứng minh.
2

Từ khẳng định trên, ta thấy rằng dãy số trên là dãy giảm và bị chặn dưới bởi 0 nên
3 1
nó có giới hạn. Giả sử giới hạn đó là a thì chuyển hệ thức un+1 = u2n − u3n qua
2 2
3a2 − a3
giới hạn, ta được a = , hay a(a − 1)(a − 2) = 0. Từ đây ta tìm được a = 0
2
1
(vì theo chứng minh trên thì a < a1 = ).
2

Nhận xét. Ta cũng có thể suy ra lim un = 0 bằng cách sử dụng nhận xét sau
n→∞
 n−1
u2 (3 − u ) 3 3
n−1 n−1
|un | = < un−1 < u1 .

2 4 4
 
3 n−1
Do lim u1 = 0 nên lim un = 0.

n→∞ 4 n→∞

Bài 7.2. Chứng minh rằng với mỗi số nguyên dương n cho trước thì phương trình
x2n+1 = x + 1 có đúng một nghiệm thực. Gọi nghiệm đó là xn . Tính lim xn .
n→∞
(Bình Định)

Lời giải. Nếu x < −1 thì x2n+1 < x < x + 1. Nếu −1 ≤ x ≤ 0 thì

x2n+1 − x = (−x)(1 − x2n ) < 1,

suy ra x2n+1 < x + 1. Nếu 0 < x ≤ 1 thì x2n+1 ≤ x < x + 1. Vậy nếu x là nghiệm của
phương trình x2n+1 = x + 1 thì ta phải có x > 1. Đặt fn (x) = x2n+1 − x − 1. Ta có
fn0 (x) = (2n + 1)x2n − 1 > 0 trên [1, ∞) suy ra hàm f tăng trên nửa khoảng này. Vì
92 Trần Nam Dũng (chủ biên)

f (1) = −1 < 0 và f (2) = 22n+1 − 3 > 0 nên phương trình này có nghiệm xn thuộc
(1, 2). Theo lý luận trên, nghiệm này là suy nhất.

Xét fn+1 (x) = x2n+3 − x − 1. Ta có fn+1 (1) = −1 < 0 và


fn+1 (xn ) = xn2n+3 − xn − 1 = xn2n+3 − xn2n+1 > 0.
Từ đó ta suy ra 1 < xn+1 < x + n. Dãy {xn } giảm và bị chặn dưới bởi 1, suy ra dãy
{xn } có giới hạn hữu hạn a, hơn nữa a ≥ 1. Ta chứng minh a = 1. Thật vậy, giả sử
a > 1. Khi đó xn ≥ a với mọi n và ta tìm được n đủ lớn sao cho xn2n+1 ≥ a2n+1 > 3.
Trong khi đó ta có xn + 1 < x1 + 1 < 3. Mâu thuẫn vì fn (xn ) = 0.

Bài 7.3. Cho dãy số (an ) xác định bởi a1 = a2 = 12 2 và
q q
an+1 = −an a2n−1 + 1 + an−1 a2n + 1
q p
với mọi n ≥ 2. Chứng minh rằng 2 + 2 a2n + 1 là số nguyên dương với mọi n ≥ 1.
(Hải Phòng)
Lời giải. Ta có
 q q 2
a2n+1 + 1 2
= −an an−1 + 1 + an−1 a2n + 1 + 1
q
= 2(an an−1 )2 + a2n + a2n−1 + 1 − 2an an−1 (a2n + 1)(a2n−1 + 1)
q 2
= (a2n + 1)(a2n−1 + 1) − an an−1 .

Do đó, q q
a2n+1 + 1 = (a2n + 1)(a2n−1 + 1) − an an−1 .
p
Đặt un = a2n + 1. Khi đó un ≥ 1 và dãy (un ) thỏa mãn
q
u1 = u2 = 17, un+1 = un un−1 − (u2n − 1)(u2n−1 − 1).
p
Ta cần chứng minh 2(un + 1) nguyên với mọi n ≥ 1.

Từ u3 = 1, bằng nguyên lý quy p nạp ta chứng minh được u3k−1 = u3k−2 = 17 và


u3k = 1 với mọi k ≥ 1. Do đó 2(un + 1) nhận một trong hai giá trị 2 hoặc 6, tức
nó có giá trị nguyên. Bài toán được chứng minh.
Bài 7.4. Giả sử dãy số nguyên dương {an } thỏa mãn a4 = 4 và
1 1 1 (n + 3)an
+ +···+ =
a1 a2 a3 a2 a3 a4 an an+1 an+2 4an+1 an+2
với mọi n nguyên, n ≥ 2. Chứng minh rằng an = n với mọi n nguyên dương.
Lời giải và bình luận đề thi các tỉnh, các trường Đại học năm học 2009-2010 93

Lời giải. Ta viết lại hệ thức truy hồi dưới dạng

(n + 2)an−1 1 n+3
+ = ,
4an an+1 an an+1 an+2 an+1 an+2

hay
(n + 3)a2n − 4
(n + 2)an+2 = (∗)
an−1
với mọi n ≥ 3. Cho n = 2 vào đẳng thức ban đầu, ta được 4(a1 + 4) = 5a1 a22 suy ra
rằng a1 | 16 và 5 | a1 + 4. Từ đó suy ra a1 = 16, a2 = 1 hoặc a1 = 1, a2 = 2.

+ Trường hợp 1. a1 = 16, a2 = 1. Trong (∗), cho n = 3, 4, 5 thì ta tìm được 5a5 =
6a23 − 4, a3 a6 = 18 và 7a7 = 2a25 − 1. Vì a3 ≡ ±2 (mod 5) và a3 là ước số của 18
nên a3 = 3 hoặc a3 = 18. Kiểm tra trực tiếp cho thấy cả hai trường hợp đều không
phải là nghiệm của bài toán.

+ Trường hợp 2. a1 = 1, a2 = 2. Khi đó thay n = 3, 4 vào (∗), ta được 5a5 + 2 = 3a23 ,


a3 a6 = 18. Một lần nữa ta lại có a3 ≡ ±2 (mod 5) và a3 | 18 nên suy ra hai trường
hợp a3 = 3, a6 = 6 hoặc a3 = 18, a6 = 1. Trong trường hợp thứ hai ta có a5 = 194,
9a2 − 4
mâu thuẫn với đẳng thức 8a8 = 6 . Trường hợp thứ nhất ta suy ra a5 = 5 và
a5
như vậy ai = i với mọi i = 1, 2, 3, 4, 5, 6. Bây giờ bằng quy nạp dễ dàng chứng minh
rằng an = n với mọi n nguyên dương.

Bình luận. Bài này là đề thi chọn đội tuyển Bulgaria năm 2006. Đây là dạng toán về
dãy số nguyên. Có thể phát biểu bài toán dưới dạng khác: Tìm tất cả các giá trị a1 ,
a2 , a3 để dãy số cho bởi công thức truy hồi ban đầu và điều kiện a4 = 4 chứa gồm
toàn các số nguyên.

Dưới đây là một số bài toán tương tự.

1. Hãy xác định tất cả các hàm số f : N∗ → N∗ thoả mãn đẳng thức

f (n) + f (n + 1) = f (n + 2) f (n + 3) − 1996 ∀n ∈ N∗ .

(VMO 1996)

2. Hỏi có bao nhiêu hàm số f : N∗ → N∗ thoả mãn đồng thời hai điều kiện sau

f (n) f (n + 2) = f 2 (n + 1) + 1997 ∀n ∈ N∗ .

(VMO 1997)
94 Trần Nam Dũng (chủ biên)

xn2 + 3
Bài 7.5. Cho dãy số {xn } xác định bởi x1 = 3 và xn+1 = với mọi n ≥ 1. Chứng
3xn
minh rằng dãy {xn } có giới hạn và tìm giới hạn đó.
(Đại học Sư phạm)

xn2 + 3 2 3
Lời giải. Nhận thấy mỗi số hạng trong dãy đều dương và xn+1 = ≥ > 1,
3xn 3
2
n > 1 với n ≥ 1. Gọi a là nghiệm lớn hơn 1 của phương trình 2x − 3 = 0, tức
nên xr
3
a= . Ta sẽ chứng minh a là giới hạn của dãy. Thật vậy, ta có
2
2
xn + 3 a2 + 3 |xn − a||axn − 3|

|xn+1 − a| =
− = .
3xn 3a |3axn |
|axn − 3| 1
Do < nên từ đây ta suy ra
|3axn | 2
|xn − a| |x1 − a|
|xn+1 − a| < < ··· < .
2 2n
r
|x1 − a| 3
Vì lim = 0 nên ta có lim |xn+1 − a| = 0. Vậy lim xn = .
n→∞ 2n n→∞ n→∞ 2

Bình luận. Bài 7.1, 7.5 ta đã sử dụng phương pháp dùng định nghĩa để tìm giới hạn
dãy số, đây là một phương pháp đơn giản, dễ sử dụng nhưng nó có tác dụng rất lớn
trong giải quyết một số bài toán tính giới hạn, đặc biệt là các dãy truy hồi.

Ta cũng có thể giải theo cách khác là xét tính đơn điệu của các hàm số, tuy nhiên
lời giải theo cách này sẽ dài hơn vì có thể phải xét đến các dãy con tùy thuộc vào sự
đơn điệu.
Bài 7.6. Cho dãy số nguyên {an } thỏa mãn các điều kiện
(1) |an+2 − an | ≤ 2 với mọi n thuộc N∗ ;
(2) Tồn tại dãy {bn } sao cho am + an = bm2 +n2 với mọi m, n thuộc N∗ .
Chứng minh rằng trong bảy số hạng bất kì của {an } luôn tồn tại hai số hạng bằng
nhau.
(Đại học Sư phạm)
Lời giải. Ta có đẳng thức sau với mọi m, k

(m + k)2 + (mk − 1)2 = (m − k)2 + (mk + 1)2 ,

suy ra với mọi m ≥ k + 1 thì

am+k + amk−1 = am−k + amk+1 .


Lời giải và bình luận đề thi các tỉnh, các trường Đại học năm học 2009-2010 95

Từ đó ta có
|am+k − am−k | = |amk+1 − amk−1 | ≤ 2.
Đặt n = m − k, suy ra với mọi n nguyên dương ta có |an+2k − an | ≤ 2. Như vậy
|am − an | ≤ 2 (1)
với mọi m, n nguyên dương cùng tính chẳn lẻ.

Đặt x = max{a2k }, y = min{a2k }. Từ bất đẳng thức (1) suy ra |x − y| ≤ 2. Vậy các
số hạng chẵn của dãy chỉ có thể nhận tối đa 3 giá trị khác nhau. Tương tự các số lẻ
của dãy chỉ có thể nhận tối đa 3 giá trị khác nhau. Suy ra mọi số hạng của dãy chỉ có
thể nhận tối đa 6 giá trị khác nhau. Từ đây sử dụng nguyên lý Dirichlet, ta có ngay
điều phải chứng minh.

Bình luận. Yêu cầu đề bài đưa ra ban đầu có thể gây sốc. Tuy nhiên, nếu trấn tĩnh
lại thì có thể hiểu ra ngay rằng ta cần chứng minh dãy số an có không quá 6 giá trị.
Điều kiện thứ hai cho chúng ta thông tin an + am = a p + aq nếu m2 + n2 = p2 + q2 .
Về việc khai thác hằng đẳng thức để giải phương trình hàm, có thể xem thêm bài
toán sau: Tìm tất cả các hàm số f : N → N sao cho f (m2 + n2 ) = f 2 (m) + f 2 (n) với
mọi m, n thuộc N.
Bài 7.7. Cho dãy số (xn ) được xác định như sau
2 xn
x1 = , xn+1 = ∀n ≥ 1.
3 2(2n + 1)xn + 1
n
Tính lim
n→∞
∑ xi .
i=1
(Kon Tum)
1
Lời giải. Đặt un = , khi đó từ công thức truy hồi của (xn ) ta có
xn
3
u1 = , un+1 = un + 2(2n + 1).
2
1
Từ đây, bằng quy nạp toán học, ta dễ dàng chứng minh được un = 2n2 − . Như vậy
2
2 1 1
xn = 2 = − .
4n − 1 2n − 1 2n + 1
Và ta tính được
n  
1 1 1 1 1
lim ∑ xi = lim 1 − + − + · · · + −
n→∞
i=1
n→∞ 3 3 5 2n − 1 2n + 1
 
1
= lim 1 − = 1.
n→∞ 2n + 1
96 Trần Nam Dũng (chủ biên)

Bài 7.8. Cho dãy số (xn ) thỏa mãn

x1 + 2x2 + · · · + (n − 1)xn−1
x1 = 2, xn = ∀n ≥ 2.
n(n2 − 1)

Tìm lim un với un = (n + 1)3 xn .


n→∞
(Nghệ An)

Lời giải. Trước hết ta có để ý rằng

(n + 1)3 [x1 + 2x2 + · · · + (n − 1)xn−1 ]


lim un = lim
n→∞ n→∞ n(n2 − 1)
(n + 1)3
= lim · lim [x1 + 2x2 + · · · + (n − 1)xn−1 ]
n→∞ n(n2 − 1) n→∞

= lim [x1 + 2x2 + · · · + (n − 1)xn−1 ].


n→∞

Vì thế để tính lim un ta chỉ cần tìm cách tính lim [x1 + 2x2 + · · · + (n − 1)xn−1 ] là đủ.
n→∞ n→∞

x1 + 2x2 + · · · + (n − 1)xn−1
Do nxn = nên
n2 − 1
n−2 n−2

n−1 n−2 n−2 ∑ ixi (n − 1)2 ∑ ixi


∑ ixi = ∑ ixi + (n − 1)xn−1 = ∑ ixi + (n −i=11)2 − 1 = i=1
n(n − 2)
i=1 i=1 i=1
(n − 1) (n − 2)2 · · · 22 x1
2 4(n − 1)
= ··· = = .
n(n − 2) · (n − 1)(n − 3) · · · 3 · 1 n

Từ đó suy ra

4(n − 1)
lim [x1 + 2x2 + · · · + (n − 1)xn−1 ] = lim = 4.
n→∞ n→∞ n
Vậy lim un = 4.
n→∞

Bài 7.9. Cho dãy số thực {xn } xác định bởi x1 = 1, xn+1 = 2xn với n ≥ 1. Chứng
minh dãy số {xn } có giới hạn. Tìm giá trị giới hạn này.
(Huế)
√ x
Lời giải. Đặt f (x) = 2 thì dãy số có dạng x1 = 1 và xn+1 = f (xn ). Ta thấy f (x)

là hàm số tăng và x2 = 2 > 1 = x1 nên xi+1 = f (xi ) ≥ f (xi−1 ) = xi . Theo nguyên
lý quy nạp, {xn } là dãy số tăng.
Lời giải và bình luận đề thi các tỉnh, các trường Đại học năm học 2009-2010 97

Tiếp theo, ta chứng minh xn < 2 với mọi n ≥ 1. Điều này đúng với n = 1. Giả sử
√ xk √ 2
ra đã có xk < 2 thì rõ ràng xk+1 = 2 < 2 = 2. Theo nguyên lý quy nạp
toán học, ta có xn < 2 với mọi n.

Vậy dãy {xn } tăng và bị chặn trên bởi


√2 nên
xn dãy có giới hạn hữu hạn. Gọi
√ alà giới
a
hạn đó thì chuyển đẳng thức xn+1 = 2 sang giới hạn, ta được a = 2 , hay
ln a ln 2
= . Ngoài ra ta cũng có a ≤ 2.
a 2
ln x ln 2 ln x
Xét phương trình = . Khảo sát hàm số ta thấy rằng phương trình trên
x 2 x
chỉ có một nghiệm nhỏ hơn e và một nghiệm lớn hơn e. Vì 2 là một nghiệm của
phương trình nên rõ ràng chỉ có một nghiệm duy nhất của phương trình thoả mãn
điều kiện ≤ 2. Từ đó suy ra a = 2.

Bài 7.10. Tìm công thức tổng quát của dãy số sau

u1 = 12 , u2 = 22 + 42 , u3 = 32 + 52 + 72 , ...

(Bến Tre)

Lời giải. Dễ dàng nhận thấy un = n2 + (n + 2)2 + (n + 4)2 + · · · + (3n − 2)2 . Đặt
A = un và B = (n + 1)2 + (n + 3)2 + · · · + (3n − 1)2 . Thế thì ta có
3n−1 3n−1 n−1
(3n − 1)3n(6n − 1) (n − 1)n(2n − 1)
A+B = ∑ i2 = ∑ i2 − ∑ i2 = −
i=n i=1 i=1 6 6
n(26n2 − 12n + 1)
= ,
3

B − A = n + (n + 1) + (n + 2) + · · · + (3n − 1) = n(4n − 1).
Do đó,
n(13n2 − 12n + 2)
un = A = .
3

5Un + 4
Bài 7.11. Cho dãy số (Un ) xác định bởi U1 = 5, Un+1 = với mọi n ∈ N∗ .
Un + 2
(1) Chứng minh rằng Un > 4 với mọi n thuộc N∗ .

(2) Tìm số hạng tổng quát của (Un ).

(Phú Thọ)
98 Trần Nam Dũng (chủ biên)

Lời giải. (1) Bằng nguyên lý quy nạp ta sẽ chứng minh Un > 4. Với n = 1, ta có
U1 = 5 > 4. Giả sử khẳng định đúng với n = k, ta có
5Uk + 4 Uk − 4
Uk+1 = = 4+ > 4,
Uk + 2 Uk + 2

nên khẳng định đúng với n = k + 1. Theo nguyên lý quy nạp ta có điều phải chứng
minh.
Un − 4
(2) Dễ thấy Un+1 − 4 = , suy ra
Un + 2
1 Un + 2 6
= = + 1.
Un+1 − 4 Un − 4 Un − 4
1
Đặt vn = , ta được v1 = 1 và vn+1 = 6vn + 1. Giải phương trình sai phân này
Un − 4
6n − 1 4 · 6n + 1
ta tìm được vn = . Từ đó suy ra Un = n .
5 6 −1
mãn điều kiện a2 = b + 1. Dãy số
Bài 7.12. Cho ba số nguyên không âm a, b, c thỏa p
(un ) được xác định như sau u0 = 0, un+1 = aun + bu2n + c2 với mọi n ∈ N. Chứng
minh mọi số hạng của dãy đều nguyên.
(Bình Định)

Lời giải. Từ hệ thức truy hồi, ta có


q
un+1 − aun = bu2n + c2 ,

(un+1 − aun )2 = bu2n + c2 ,


u2n+1 − 2aun+1 un + (a2 − b)u2n − c2 = 0,
u2n+1 − 2aun+1 un + u2n − c2 = 0.
Thay n + 1 bởi n − 1, ta cũng có

u2n−1 − 2aun−1 un + u2n − c2 = 0.

Mặt khác dễ thấy un+1 6= un−1 với mọi n ≥ 1. Vì thế từ trên ta suy ra un+1 và un−1
là hai nghiệm của phương trình x2 − 2un x + u2n − c2 = 0. Theo định lý Viette, ta có
un+1 + un−1 = 2un , hay un+1 = 2un − un−1 . Từ hệ thức này và từ u0 = 0, u1 = c ta
suy ra được un nhận giá trị nguyên với mọi n.

Bình luận. Các bài 7.7, 7.11, 7.12: Mấu chốt của bài toán này là tìm ra các phương
trình sai phân thích hợp, khi đã tìm được thì việc giải các bài toán trở nên đơn giản.
Phần II

Một số bài giảng toán

99
Chương 8

Giải phương trình hàm bằng cách


lập phương trình

“Cuộc sống là chuỗi những phương trình mà ta kiếm tìm lời giải.”

12 Giảibài toán bằng cách lập phương trình và hệ phương trình là một phương pháp
thông dụng trong các bài toán đại số. Ý tưởng là để tìm một ẩn số nào đó, ta đưa
vào các ẩn số phụ, sử dụng các dữ kiện đã cho tạo ra mối liên hệ giữa các ẩn số đó
(các phương trình), giải hệ phương trình, tìm ra giá trị của ẩn số cần tìm. Phương
pháp tương tự cũng có thể áp dụng cho các bài toán hình học tính toán (chẳng hạn
bài toán giải tam giác, tứ giác), các bài toán đếm (phương pháp dãy số phụ).

Trong bài này, chúng ta đề cập tới phương pháp lập phương trình, hệ phương trình
để giải các bài toán phương trình hàm. Ý tưởng chung cũng là để tìm một giá trị
f (x) hoặc f (a) nào đó, ta sử dụng phương trình hàm để tìm ra mối liên kết giữa các
đại lượng, nói cách khác, tạo ra các phương trình số. Giải các phương trình số này,
ta có thể tìm ra f (x) hoặc f (a) với a là một giá trị nào đó.

Với những phương trình hàm có hai (hoặc nhiều hơn) phương trình điều kiện, ta có
thể tìm cách kết hợp các phương trình đó để tìm ra f (x). Phương pháp cơ bản vẫn
là tạo ra các mối liên kết, hay các phương trình bằng cách tính một giá trị bằng hai
cách khác nhau.

Ví dụ 8.1. Tìm tất cả các hàm số f : R → R thoả mãn điều kiện

(i) f (−x) = − f (x) với mọi x thuộc R;


1 Bàiviết được viết bởi TS. Trần Nam Dũng.
2 Tríchbài viết Giải phương trình hàm bằng cách lập phương trình, Kỷ yếu Hội nghị Khoa học kỷ
niệm 25 seminar Giải tích và Toán sơ cấp, Bắc Giang 11/2009.

101
102 Trần Nam Dũng (chủ biên)

(ii) f (x + 1) = f (x) + 1 với mọi x thuộc R;


 
1 f (x)
(iii) f = 2 với mọi x khác 0.
x x

Lời giải. Tất cả các điều kiện đều trên một biến x. Trong trường hợp này, ta có thể
dùng một chút khái niệm về đồ thị để hiểu con đường đi đến lời giải. Ta xem các số
1
thực như các đỉnh của một đồ thị. Đỉnh x sẽ được nối với các đỉnh x + 1, −x, . Các
x
điều kiện đề bài sẽ cho chúng ta các mối liên hệ giữa giá trị của hàm số tại các đỉnh
được nối bởi một cạnh. Nếu chúng ta tìm được một chu trình thì một cách tự nhiên,
chúng ta sẽ có một phương trình (để tránh hàm số có hai giá trị khác nhau).

Ta thử tìm một chu trình như vậy

1 1 1 x x+1 1 1
x → x+1 → →− → 1− = → = 1 + → → x.
x+1 x+1 x+1 x+1 x x x

Đặt y = f (x) thì từ chu trình ở trên, ta lần lượt có


   
1 y+1 1 y+1
f (x + 1) = y + 1, f = 2
, f − =− ,
x+1 (x + 1) x+1 (x + 1)2

y+1
1−
(x + 1)2 x2 + 2x − y
   
x y+1 x+1
f = 1− , f =  2 = ,
x+1 (x + 1)2 x x x2
x+1
 
1 2x − y
f = , f (x) = 2x − y.
x x2
Từ đó suy ra 2x − y = y, tức là y = x. Vậy f (x) = x.

Trong lý luận trên, ta cần đến điều kiện x khác 0 và −1. Tuy nhiên từ hai điều kiện
f (−x) = − f (x), f (x + 1) = f (x) + 1 ta dễ dàng suy ra f (0) = 0 và f (−1) = 1. Vậy
f (x) = x là tất các nghiệm của bài toán.

Ví dụ 8.2. Tìm tất cả các hàm số f : R → R thoả mãn điều kiện

f (x2 − y) = x f (x) − f (y) với mọi x, y thuộc R.

Lời giải. Thay x = y = 0 vào phương trình hàm, ta được f (0) = − f (0), suy ra
f (0) = 0. Thay y = 0 vào phương trình hàm, ta được

f (x2 ) = x f (x). (1)


Lời giải và bình luận đề thi các tỉnh, các trường Đại học năm học 2009-2010 103

Từ đó suy ra
f (x2 − y) = f (x2 ) − f (y).
Thay x = 0, ta được f (−y) = − f (y). Thay y bằng −y, ta được

f (x2 + y) = f (x2 ) − f (−y) = f (x2 ) + f (y)

với mọi x, y. Từ đó, kết hợp với tính chất hàm lẻ, ta suy ra f (x + y) = f (x) + f (y)
với mọi x, y. Bây giờ ta có f ((x + 1)2 ) một mặt có thể tính theo công thức (1), tức là
bằng (x + 1) f (x + 1) = (x + 1)[ f (x) + f (1)]. Mặt khác, ta có thể khai triển

f ((x + 1)2 ) = f (x2 + 2x + 1) = f (x2 ) + 2 f (x) + f (1) = x f (x) + 2 f (x) + f (1).

Từ đó ta được phương trình (x + 1)[ f (x) + f (1)] = x f (x) + 2 f (x) + f (1), suy ra
f (x) = f (1)x. Đặt f (1) = a, ta được f (x) = ax. Thử lại vào phương trình ta thấy
nghiệm đúng.

Vậy f (x) = ax với a ∈ R là tất cả các nghiệm của bài toán.

Phương pháp tạo ra các mối liên kết cũng có thể áp dụng hiệu quả trong các bài toán
phương trình hàm trên Q, N, Z. Ta xem xét một số ví dụ.
Ví dụ 8.3. Tìm tất cả các hàm số f : Q+ → Q+ thoả mãn các điều kiện
(i) f (x + 1) = f (x) + 1 với mọi x thuộc Q+ ;

(ii) f (x2 ) = f 2 (x) với mọi x thuộc Q+ .


Lời giải. Từ điều kiện (ii), ta suy ra được f (1) = 1. Sử dụng kết quả này kết hợp với
điều kiện (i) ta dễ dàng suy ra f (n) = n với mọi n thuộc Z+ và f (r + n) = f (r) + n
p
với mọi r thuộc Q+ và n thuộc Z+ . Bây giờ ta tính f (r) với r = , p, q ∈ Z+ . Ý
q
tưởng ta sẽ tính f ((r + q)2 ) theo f (r) bằng hai cách. Trước hết

f ((r + q)2 ) = f 2 (r + q) = ( f (r) + q)2 . (1)

Mặt khác

f ((r + q)2 ) = f (r2 + 2p + q2 ) = f (r2 ) + 2p + q2 = f 2 (r) + 2p + q2 . (2)


p
Từ (1) và (2) ta suy ra f 2 (r) + 2q f (r) + q2 = f 2 (r) + 2p + q2 , do đó f (r) = = r.
q

Vậy f (r) = r với mọi r thuộc Q+ .


Ví dụ 8.4. Tìm tất cả các hàm số f : N → N sao cho

f (m2 + n2 ) = f 2 (m) + f 2 (n) với mọi m, n thuộc N.


104 Trần Nam Dũng (chủ biên)

Lời giải. Cho m = n = 0, ta được f (0) = 2 f 2 (0), suy ra f (0) = 0. Cho m = 1, n = 0,


ta được f (1) = 0 hoặc f (1) = 1. Ta xét trường hợp f (1) = 1, trường hợp f (1) = 0
xét tương tự. Với f (1) = 1, ta lần lượt tính được

f (2) = f (12 + 12 ) = f 2 (1) + f 2 (1) = 2,


f (4) = f (22 + 02 ) = f 2 (2) + f 2 (0) = 4,
f (5) = f (22 + 12 ) = f 2 (2) + f 2 (1) = 5.

Nhưng làm sao để tính, chẳng hạn f (3)? Rõ ràng f (3) không thể tính được theo sơ
đồ trên được, vì 3 không biểu diễn được dưới dạng tổng của hai bình phương.

Ta nhớ lại một bài toán lớp 3. Có một cái cân đĩa với hai quả cân 1kg, 5kg và một
bao đường nặng 10kg. Hãy cân ra 7kg đường bằng một lần cân. Rõ ràng, với cách
cân thông thường thì ta chỉ cân được 1kg đường, 4kg đường (5 − 1), 5kg đường và
6kg đường. Tuy nhiên, nếu tinh ý một chút, ta có thể có phương án cân được 7kg
đường như sau: Đặt vào đĩa bên trái quả cân 1kg và 10kg đường, đĩa bên phải là
quả cân 5kg, sau đó chuyển dần đường từ bên trái sang bên phải sao cho cân cân
bằng, khi đó số đường còn lại ở đĩa bên phải là 7kg!

Bây giờ ta cũng thủ thuật tương với bài toán này. Ta không tính được trực tiếp f (3)
nhưng ta lại có f 2 (5) = f (25) = f (32 +42 ) = f 2 (3)+ f 2 (4). Từ đó ta được f (3) = 3.

Tương tự như vậy ta có thể tính được f (6) nhờ vào đẳng thức 62 + 82 = 102 , trong
đó f (8) = f (22 + 22 ) = 2 f 2 (2) = 8, f (10) = f (32 + 12 ) = f 2 (3) + f 2 (1) = 10.

Tiếp tục, để tính f (7), ta để ý 72 + 12 = 50 = 52 + 52 , từ đó f (7) = 7. Cũng như thế,


do 112 + 22 = 102 + 52 nên ta suy ra f (11) = 11.

Cách làm này có thể tổng quát hoá như thế nào? Ý tưởng là nếu m2 + n2 = p2 + q2
(1) thì f 2 (m) + f 2 (n) = f 2 (p) + f 2 (q). Do đó nếu ta đã tính được f (n), f (p), f (q)
thì f (m) cũng sẽ tính được.

Làm thế nào để có được những đẳng thức dạng (1) ở dạng tổng quát, cho phép ta
chứng minh f (n) = n với mọi n bằng quy nạp? Chú ý rằng (1) có thể viết lại thành
(m − p)(m + p) = (q − n)(q + n) = N. Do đó nếu chọn những số N có hai cách phân
tích thành tích của những số có cùng tính chẵn lẻ, ta sẽ tìm được nghiệm cho (1).
Chọn N = 8k = 2 · 4k = 4 · 2k và N = 16k = 4 · 4k = 8 · 2k, ta được hệ

m − p = 2, m + p = 4k, q − n = 4, q + n = 2k,


m − p = 4, m + p = 4k, q − n = 8, q + n = 2k.
Lời giải và bình luận đề thi các tỉnh, các trường Đại học năm học 2009-2010 105

Từ đó được các hằng đẳng thức tương ứng

(2k + 1)2 + (k − 2)2 = (2k − 1)2 + (k + 2)2 ,


(2k + 2)2 + (k − 4)2 = (2k − 2)2 + (k + 4)2 .
Từ hai đẳng thức này, với chú ý là ta đã chứng minh được f (n) = n với n = 0, 1, 2,
3, 4, 5, 6, ta dễ dàng chứng minh bằng quy nạp được rằng f (n) = n với mọi n ∈ N.

Trường hợp f (1) = 0, cũng bằng cách lý luận nêu trên ta suy ra f (n) = 0 với mọi n
thuộc N.

Bài tập
1. Tìm tất cả các hàm số f : Q → Q thoả mãn các điều kiện
(i) f (x + 1) = f (x) + 1 với mọi x thuộc Q;
(ii) f (x3 ) = f 3 (x) với mọi x thuộc Q.

2. Tìm tất cả các hàm f : R\0 → R thoả mãn đồng thời các điều kiện
(i) f (1) = 1;
     
1 1 1
(ii) f =f +f với mọi x, y mà xy(x + y) 6= 0;
x+y x y
(iii) (x + y) f (x + y) = xy f (x) f (y) với mọi x, y mà xy(x + y) 6= 0.

3. Tìm tất cả các hàm số f : R → R thoả mãn

f (x5 − y5 ) = x2 f (x3 ) − y2 f (y3 ) với mọi x, y thuộc R.

4. Tìm tất cả các hàm số f : Z → Z thoả mãn điều kiện

f (a3 + b3 + c3 ) = f 3 (a) + f 3 (b) + f 3 (c) với mọi a, b, c thuộc Z.

5. Cho hàm số f : R → R thoả mãn điều kiện


(i) f (x2 ) = f 2 (x) với mọi x thuộc R;
(ii) f (x + 1) = f (x) + 1 với mọi x thuộc R.
Chứng minh rằng f (x) = x.
106 Trần Nam Dũng (chủ biên)

Tài liệu tham khảo


[1] Nguyễn Văn Mậu, Phương trình hàm, Nhà xuất bản Giáo dục 2001.

[2] Nguyễn Trọng Tuấn, Bài toán hàm số qua các kỳ thi Olympic, Nhà xuất bản
Giáo dục 2005.

[3] Phan Đức Chính, Lê Đình Thịnh, Phạm Tấn Dương, Tuyển tập các bài toán sơ
cấp, Tập 1, Đại số, Nhà xuất bản Đại học và Trung học chuyên nghiệp 1977.

[4] Phan Huy Khải, Các bài toán về hàm số, Nhà xuất bản Giáo dục 2007.

[5] B. J. Venkatachala, Functional Equations – A Problem Solving Approach,


PRISM 2002.

[6] Pierre Bornsztein, Mobinool Omarjee, Cours – Equations fonctionelles, Elec-


tronic Edition 2003.

[7] Titu Andreescu, Iurie Boreico, Functional Equations, Electronic Edition 2007.
Chương 9

Dãy truy hồi loại un+1 = f (un)

Cách tính chất chung


Cho I là một khoảng đóng của R, f : I → I là một ánh xạ.

(a) Giả sử f đơn điệu trên I.

+ Trường hợp f tăng trên I. Vì với mọi n nguyên dương thì un+1 − un = f (un ) −
f (un−1 ) nên ta thấy rằng un+1 − un cùng dấu với u1 − u0 . Chính xác hơn,

u0 ≤ u1 ⇒ u1 ≤ u2 ⇒ · · · ⇒ un ≤ un+1 ⇒ · · ·

u0 ≥ u1 ⇒ u1 ≥ u2 ⇒ · · · ⇒ un ≥ un+1 ⇒ · · ·
Như vậy (un ) đơn điệu và có chiều biến thiên phụ thuộc vào vị trí tương đối của u0
và u1 . Trong mỗi ví dụ chỉ còn phải xem (un ) bị chặn dưới hay bị chặn trên.

+ Trường hợp f giảm trên I. Ánh xạ f ◦ f tăng trên I, vậy theo trường hợp trên, các
dãy con với chỉ số chẵn và chỉ số lẻ đều đơn điệu (và có chiều ngược nhau).

(b) Giả sử f liên tục trên I. Nếu un → L thì L thuộc I, chuyển qua giới hạn khi n
dần đến vô cùng trong biểu thức un+1 = f (un ), ta suy ra f (L) = L. Thường thì ta có
thể giải phương trình f (L) = L (ẩn là L thuộc I) và từ đó xác định được các giới hạn
“khả dĩ” của (un ).

Ta nói một phần tử x của I là một điểm bất động của f khi và chỉ khi f (x) = x.
1 Trích từ Giáo trình Giải tích 1, Jean-Marie Monier, Nhà xuất bản Giáo dục 1999.

107
108 Trần Nam Dũng (chủ biên)

Các ví dụ minh họa


Ví dụ 9.1. Khảo sát sự hội tụ của dãy (un ) được xác định bởi công thức u0 = 1 và
un
un+1 = 2 với mọi n = 0, 1, 2, . . .
un + 1

Lời giải. Trước hết, một phép quy nạp đơn giản cho thấy rằng với mọi n thuộc N, un
thuộc [0, +∞).

u3n
Với mọi n thuộc N, un+1 − un = − ≤ 0, vậy (un ) giảm.
u2n + 1

Vì (un ) giảm và bị chặn dưới bởi 0 nên nó hội tụ đến một số thực L và L ≥ 0. Chuyển
L
qua giới hạn khi n tiến tới +∞, ta có L = 2 , từ đó L = 0. Cuối cùng ta được
L +1
un → 0.

Ví dụ 9.2.Khảo sát sự hội tụ của dãy (un ) được xác định bởi công thức u0 > 0 và
1 a2
un+1 = un + với mọi n ∈ N, trong đó a là một hằng số dương cho trước.
2 un

Lời giải. Trước hết, một phép quy nạp đơn giản cho thấy với mọi n thuộc N, un tồn
tại và thuộc (0, +∞).

a2
 
1
Điểm bất động duy nhất thuộc (0, +∞) của hàm số f (x) = un + là x = a.
2 un

u2n + a2 − 2aun (un − a)2


Với mọi n thuộc N, un+1 − a = = ≥ 0.
2un 2un

a2 − u2n
Với mọi n thuộc N∗ , un + 1 − un = ≤ 0.
2un

Vậy (un ) giảm và bị chặn dưới bởi a nên hội tụ đến một số thực L thuộc [L, +∞), số
thực đó chỉ có thể là a theo lời giải của f (x) = x. Cuối cùng un → a.

Ví dụ 9.3. Khảo sát sự hội tụ của dãy (un ) được xác định bởi công thức u0 > 0 và
u2 + 8
un+1 = n với mọi n ∈ N.
6
Lời giải. Phép quy nạp đơn giản chứng tỏ rằng với mọi n thuộc N, un > 0.

Phép giải phương trình f (x) = x với x thuộc R+ cho thấy f có hai điểm bất động
là 2 và 4. Khảo sát hàm số f (x), ta thấy f tăng trên (0, +∞) và các khoảng đóng
[0, 2], [2, 4], [4, +∞) đều ổn định đối với f (nghĩa là f ([0, 2]) thuộc [0, 2] . . . ).
Lời giải và bình luận đề thi các tỉnh, các trường Đại học năm học 2009-2010 109

Vì f tăng, (un ) đơn điệu nên chiều biến thiên phụ thuộc vào dấu của u1 − u0 . Vì
(x − 2)(x − 4)
f (x) − x = , nên dấu của u1 − u0 phụ thuộc vào vị trí tương đối của u0
6
so với 2 và 4.

+ Trường hợp 1. u0 thuộc [0, 2]. Ở đây u1 ≥ u0 , vậy bằng một phép quy nạp đơn giản
ta có với mọi n thuộc N, un+1 ≥ un . Hơn nữa, với mọi n thuộc N, un thuộc [0, 2].
Vậy (un ) tăng và bị chặn trên bởi 2, nên hội tụ đến số thực L thuộc [0, 2]. Ta đã thấy
L thuộc {2, 4}. Vậy L = 2.

+ Trường hợp 2. u0 thuộc [2, 4). Bằng cách tương tự ta thấy rằng (un ) giảm và bị
chặn dưới bởi 2 nên hội tụ đến một số thực L thuộc [2, u0 ] thuộc[2, 4). Ta đã biết L
thuộc {2, 4}, vậy L = 2.

+ Trường hợp 3. u0 = 4. Dãy (un ) không đổi và bằng 4, hội tụ đến 4.

+ Trường hợp 4. u0 thuộc (4, +∞). Ở đây (un ) tăng. Nếu (un ) hội tụ đến một số thực
L thì ta có L ≥ u0 > 4, mâu thuẫn với L thuộc {2, 4}. Do đó (un ) tăng và phân kỳ,
vậy un → +∞.

Ta nói rằng 2 là điểm bất động hút và 4 là điểm bất động đẩy của f .

Ví dụ 9.4. Khảo sát sự hội tụ của dãy (un ) được xác định bởi công thức u0 = 1 và
1
un+1 = với mọi n = 0, 1, 2, . . .
2 + un
Lời giải. Phép quy nạp đơn giản cho thấy rằng với mọi n thuộc N, un > 0.

1
Cho f : R+ → R+ , x → . Phép giải phương trình f (x) = x (với x > 0) cho thấy
2+x √
có một và chỉ một điểm bất động, ký hiệu là A và A = 2 − 1.

Với mọi n thuộc N, ta có



1 1 |un − A| 1
|un+1 − A| =
− = ≤ |un − A|.
2 + un 2 + A (2 + un )(2 + A) 4

1
Bằng một phép quy nạp đơn giản, ta suy ra với mọi n thuộc N, |un − A| ≤ n |u0 − A|.
4
Vậy un → A.

Ở đây không cần khảo sát các dãy con với chỉ số chẵn và chỉ số lẻ.

Ví dụ 9.5. Khảo sát sự hội tụ của dãy (un ) được xác định bởi công thức u0 ≥ 0 và
2
un+1 = với mọi n ∈ N.
1 + u2n
110 Trần Nam Dũng (chủ biên)

Lời giải. Một phép quy nạp đơn giản chỉ ra rằng un ≥ 0 với mọi n tự nhiên.
2
Xét f : [0, +∞) → [0, +∞), f (x) = là một hàm liên tục. Ta có với mọi x
1 + x2
thuộc [0, +∞), phương trình f (x) = x chỉ có một nghiệm duy nhất là x = 1. Vậy nếu
un hội tụ thì chỉ có thể hội tụ đến 1.
4x
Ánh xạ f khả vi trên [0, +∞), f 0 (x) = − ≤ 0 với mọi x thuộc [0, +∞), vậy
(1 + x2 )2
f giảm. Vì f 0 (1) = −1, ta không thể lập luận như trong ví dụ 4.

Ta sẽ chứng minh rằng u2p → 1 và u2p+1 → 1. Cho g = f ◦ f : [0, +∞) → [0, +∞),
2(1 + x2 )2
g(x) = . Ta tính
(1 + x2 )2 + 4
(x − 1)3 (x2 + x + 2)
g(x) − x = − .
(1 + x2 )2 + 4
+ Trường hợp 1. u0 thuộc [0, 1]. Khi ấy với mọi p thuộc N, u2p thuộc [0, 1] và u2p+1
thuộc [1, +∞). Vậy, với mọi p thuộc N,
u2p+2 − u2p = g(u2p ) − u2p ≥ 0, u2p+3 − u2p+1 = g(u2p+1 ) − u2p+1 ≤ 0.
Do đó (u2p ) và (u2p+1 ) giảm. Hơn nữa, vì với mọi p thuộc N, u2p ≤ 1 ≤ u2p+1 , nên
ta suy ra rằng (u2p ) hội tụ đến một phần tử L thuộc [0, +∞) và (u2p+1 ) hội tụ đến
một phần tử L0 thuộc [0, +∞). Vì g liên tục trên [0, +∞) và vì x = 1 là nghiệm thuộc
[0, +∞) duy nhất của phương trình g(x) = x, nên ta suy ra L = L0 = 1. Cuối cùng
un → 1.

+ Trường hợp 2. u0 thuộc [1, +∞). Vì u1 = f (u0 ) thuộc [0, 1], ta quy về trường hợp
trên (bằng cách thay u0 bởi u1 ) và ta có cùng một kết luận un → 1.

Bài tập
1. Khảo sát sự hội tụ của các dãy sau
2
(a) u0 = 1 và un+1 = 1 − với mọi n ∈ N.
un
3 + u2n
(b) u0 > 0 và un+1 = với mọi n ∈ N.
2(un + 1)
(c) u0 tùy ý và un+1 = u2n + 2un với mọi n ∈ N.
2. Khảo sát dãy (un ) được xác định bởi
6
u0 ≥ 0, un+1 = ∀n ∈ N.
2 + u2n
Lời giải và bình luận đề thi các tỉnh, các trường Đại học năm học 2009-2010 111

3. Khảo sát các dãy (un ), (vn ) được xác định bởi
p p
u0 = v0 = 0, un+1 = 3 − vn , vn+1 = 3 − un ∀n ∈ N.
112 Trần Nam Dũng (chủ biên)
Chương 10

Các định lý tồn tại trong giải tích


và định lý cơ bản của đại số

1 Trongbài viết nhỏ này, chúng ta đề cập đến một số định lý cơ bản của giải tích có
nội dung tồn tại (tồn tại nghiệm, tồn tại cực trị . . . ) và cuối cùng, sẽ sử dụng chúng
để chứng minh định lý cơ bản của đại số: một đa thức bậc lớn hơn hay bằng 1 có
hệ số phức luôn có ít nhất một nghiệm phức. Cách chứng minh đơn giản, dễ hiểu,
không quá hình thức sẽ giúp học sinh hiểu rõ các định lý và không cảm thấy sợ
chúng. Chúng ta cũng xem xét một số ứng dụng của các định lý này trong việc giải
quyết các bài toán ở bậc phổ thông.

Bổ đề về dãy các đoạn thẳng lồng nhau


Bổ đề đơn giản này đóng một vai trò khá quan trọng trong việc chứng minh các
kết quả sâu sắc khác của giải tích. Bổ đề được phát biểu như sau: Nếu [a1 , b1 ] ⊂
[a2 , b2 ] ⊂ · · · ⊂ [an , bn ] ⊂ · · · là dãy các đoạn thẳng lồng nhau có dn = bn − an → 0
thì tồn tại duy nhất một điểm ξ thuộc tất cả các đoạn thẳng trên.

Bổ đề này có thể chứng minh khá dễ dàng dựa vào định lý: một dãy đơn điệu và bị
chặn thì có giới hạn. Cụ thể, dãy {an } sẽ là dãy tăng và bị chặn trên, còn dãy {bn }
sẽ là dãy giảm và bị chặn dưới. Cả hai dãy này sẽ có cùng giới hạn là điểm ξ .

Định lý về sự tồn tại giới hạn của dãy đơn điệu và bị chặn, về phần mình, lại được
chứng minh dựa vào một kết quả cơ bản sau: một tập hợp các số thực bị chặn trên
(hay bị chặn dưới) thì có cận trên đúng (cận dưới đúng). Ở đây, M được gọi là cận
trên đúng của tập hợp S nếu:

(1) x ≤ M với mọi x thuộc S;


1 Bài viết được viết bởi TS. Trần Nam Dũng.

113
114 Trần Nam Dũng (chủ biên)

(2) Với mọi ε > 0, tồn tại x thuộc S sao cho x > M − ε.

Định lý tưởng chừng như hiển nhiên này là một kết quả rất sâu sắc và không đơn
giản chút nào. Ta công nhận định lý này và coi đây là định lý nền tảng của giải tích.

Định lý Cauchy về giá trị trung gian


Định lý Cauchy về giá trị trung gian phát biểu rằng: một hàm số liên tục trên một
đoạn nhận mọi giá trị trung gian. Điều này có nghĩa rằng nếu hàm số liên tục nhận
hai giá trị khác nhau, thì nó nhận mọi giá trị nằm giữa hai giá trị này.

Đồ thị của một hàm số liên tục, nói nôm na có tính chất là nó có thể vẽ mà không
dứt nét bút khỏi mặt giấy. Còn định nghĩa chặt chẽ như sau. Ta nói hàm số f liên
tục tại điểm x0 , nếu với mọi ε > 0, tồn tại δ > 0 sao cho nếu |x − x0 | < δ thì
| f (x) − f (x0 )| < ε. Hàm số được gọi là liên tục trên một đoạn, nếu nó liên tục tại
mọi điểm của đoạn. Từ định nghĩa này suy ra, nếu hàm số khác 0 tại một điểm nào
đó, thì nó sẽ giữ nguyên dấu tại một khoảng (hay nửa khoảng, nếu điểm đó là đầu
mút của đoạn thẳng) chứa điểm này. Ta chỉ cần đến tính chất này.

Để chứng minh định lý Cauchy, thực chất ta chỉ cần chứng minh: một hàm số liên
tục trên một đoạn, nhận ở hai đầu mút các giá trị trái dấu, sẽ nhận giá trị 0 trên đoạn
này.

Ta chứng minh định lý Cauchy trong cách phát biểu này, tìm kiếm nghiệm của hàm
số bằng phương pháp “chia để trị”. Ta chia đoạn thẳng thành hai phần. Nếu như tại
điểm này hàm số bằng 0 thì định lý được chứng minh. Nếu như tại điểm này hàm số
khác 0, thì trên một trong hai đoạn thẳng, hàm số sẽ nhận các giá trị trái dấu tại hai
đầu mút. Ta lại chia đoạn thẳng này làm đôi và cứ tiếp tục như thế. Nếu như trong
quá trình thực hiện ta không gặp một điểm giữa có giá trị hàm số tại đó bằng 0 thì
ta sẽ thu được dãy các đoạn thẳng lồng nhau [a1 , b1 ] ⊂ [a2 , b2 ] ⊂ · · · ⊂ [an , bn ] ⊂ · · ·
có độ dài dần đến 0. Theo bổ đề về các đoạn thẳng lồng nhau, tồn tại điểm ξ thuộc
tất cả các đoạn thẳng. Theo tính chất về bảo toàn dấu, giá trị hàm số tại ξ phải bằng
0. Định lý Cauchy được chứng minh.

Từ định lý Cauchy suy ra một kết quả đơn giản nhưng khá quan trọng về nghiệm
của đa thức: Mọi đa thức bậc lẻ với hệ số thực đều có ít nhất một nghiệm thực. Thật
vậy, mọi đa thức là hàm số liên tục trên toàn trục số. Giả sử f (x) = x2n+1 + a2n x2n +
· · · + a1 x + a0 . Khi đó, với x dương, ta có
 a2n a0 
f (x) = x2n+1 1 + + · · · + 2n+1 .
x x
Lời giải và bình luận đề thi các tỉnh, các trường Đại học năm học 2009-2010 115

x2n+1
Như thế, với x đủ lớn, f (x) sẽ lớn hơn , tức là f (x) là một số dương. Hoàn toàn
2
tương tự, có thể chứng minh rằng với x đủ nhỏ thì f (x) sẽ âm. Như thế, theo định lý
Cauchy về giá trị trung gian, f (x) có nghiệm.

Định lý Cauchy còn có một hệ quả khác: một hàm liên tục từ đoạn thẳng vào chính
nó có điểm bất động (nghĩa là: nếu f là một hàm liên tục trên [a, b], a < b và a ≤
f (x) ≤ b với mọi x thuộc [a, b] thì tồn tại điểm x0 thuộc [a, b] sao cho f (x0 ) = x0 ).
Bạn đọc có thể tự chứng minh kết quả này.

Định lý Veierstrass về cực trị của hàm số liên tục trên một
đoạn
Định lý Veiestrass và các mở rộng của nó có nhiều ứng dụng trong toán học. Định
lý này được phát biểu khá đơn giản như sau: hàm liên tục trên một đoạn thẳng sẽ
đạt được giá trị lớn nhất và giá trị nhỏ nhất trên đoạn này.

Ta sẽ chứng minh định lý này. Giả sử f (x) là hàm liên tục trên một đoạn thẳng nào
đó. Không mất tính tổng quát, giả sử đó là đoạn I = [0, 1]. Trước hết ta chứng minh
rằng f bị chặn trên I. Giả sử ngược lại và f có thể nhận trên I các giá trị lớn tuỳ ý.
Khi đó với mọi số nguyên dương n, tồn tại điểm xn thuộc I sao cho f (xn ) > n. Như
vậy trên I ta xây dựng được một dãy vô hạn các điểm. Chia đoạn thẳng ra làm đôi.
Trên một trong hai đoạn thẳng sẽ có chứa vô số điểm. Lại chia đoạn đó ra làm đôi và
cứ tiếp tục như thế. Theo bổ đề về dãy các đoạn thẳng lồng nhau, tồn tại một điểm
thuộc vào tất cả các đoạn thẳng này. Từ định nghĩa liên tục suy ra trên một đoạn nhỏ
chứa điểm này, hàm số bị chặn, nhưng điều này trái với cách xây dựng điểm này.

Ta đã chứng minh rằng f (x) bị chặn trên. Giả sử f không đạt giá trị lớn nhất. Điều
này có nghĩa là tồn tại số M sao cho f (x) < M với mọi x thuộc I, đồng thời f (x)
nhận các giá trị gần M tuỳ ý. Với mỗi số nguyên dương m, tồn tại điểm ym sao cho
1
f (ym ) > M − . Ta lại xây dựng một tập hợp vô hạn các điểm. Tiếp tục chia đoạn
m
thẳng I làm hai phần và làm giống như phần chứng minh tính bị chặn của f (x) ở
trên. Và cũng như ở trên, ta tìm được điểm ζ thuộc vào tất cả các đoạn thẳng. Theo
cách xây dựng và từ định nghĩa liên tục, ta thấy f (ζ ) phải bằng M. Tương tự chứng
minh cho giá trị nhỏ nhất. Định lý Veierstrass được chứng minh.

Mở rộng định lý Veierstrass

q f = f (x1 , x2 ), trong đó x1 , x2 là các số thực. Ví dụ một hàm như


Xét hàm hai biến
vậy là hàm số x12 + x22 - khoảng cách từ điểm có toạ độ (x1 , x2 ) trên mặt phẳng
116 Trần Nam Dũng (chủ biên)

x2 ), (x10 , x20 )) giữa hai điểm (x1 , x2 ) và (x10 , x20 )


đến gốc toạ độ. Khoảng cách d((x1 ,q
trên mặt phẳng cho bởi công thức (x1 − x10 )2 + (x2 − x20 )2 . Hàm hai biến f được
gọi là liên tục tại điểm (x1∗ , x2∗ ) nếu với mọi ε > 0, tồn tại số δ > 0 sao cho nếu
d((x1 , x2 ), (x1∗ , x2∗ )) < δ thì | f (x1 , x2 ) − f (x1∗ , x2∗ )| < ε. Hàm số được gọi là liên tục
trên hình vuông max{|x1 |, |x2 |} ≤ a, nếu nó liên tục tại mọi điểm của hình vuông
này.

Ta sẽ cần đến một mở rộng sau đây của định lý Veierstrass: hàm số liên tục trên hình
vuông đạt giá trị lớn nhất và giá trị nhỏ nhất. Cách chứng minh định lý này hoàn
toàn tương tự như cách chứng minh nêu trên, điểm khác biệt duy nhất là cần phải
chia hình vuông thành bốn phần.

Và bây giờ, phép chứng minh định lý cơ bản của đại số sẽ được chia thành hai phần.
Trong phần đầu, ta sẽ lặp lại lý luận nêu trên để chứng minh mođun của đa thức đạt
giá trị nhỏ nhất của nó. Và tiếp theo, thay cho định lý Cauchy về giá trị trung gian,
ta sẽ sử dụng bổ đề D’Alamber.

Định lý cơ bản của đại số


Trước hết ta cần xây dựng mặt phẳng phức. Một cách hình thức ta đưa vào “số” i,
có bình phương bằng −1. Số này không có trên đường thẳng thực. Ta vẽ trên mặt
phẳng hai đường thẳng: một đường nằm ngang (mà ta gọi là đường thẳng thực) và
một đường khác đi qua gốc toạ độ và vuông góc với đường nằm ngang (mà ta gọi là
đường thẳng ảo). Số i, nằm ở trên đường thẳng ảo nằm ở nửa mặt phẳng phía trên
và cách gốc toạ độ khoảng cách 1, được gọi là đơn vị ảo.

Như vậy, số 1 được cho tương ứng với véc-tơ (1, 0) và số i - véc-tơ (0, 1). Điểm
(a, b) của mặt phẳng tương ứng với số phức z = a + bi. Các số phức có thể cộng và
nhân theo quy tắc tự nhiên, giống như số thực: nếu z = a+bi, z0 = a0 +b0 i, thì z+z0 =
(a+a0 )+(b+b0 )i, z·z0 = (a+bi)(a√0 +b0 i) = (aa0 −bb0 )+(ab0 +a0 b)i. Khoảng cách
từ điểm z = a + bi đến 0 (tức là số a2 + b2 ) được gọi là mođun của số z và ký hiệu
là |z|. Đa thức bậc n là biểu thức có dạng p(z) = an zn + an−1 zn−1 + · · · + a1 z + a0 .
Các hệ số ak là các số√phức (trường hợp đặc biệt là các số thực). Đa thức z2 − 2 có
hai nghiệm thực là ± 2, đa thức z2 + 1 có hai nghiệm ảo là ±i, còn đa thức iz + 1
có một nghiệm là i.

Định lý cơ bản của đại số. Đa thức bậc n ≥ 1 có nghiệm phức.

Chứng minh. Giả sử p(z) = a0 + a1 z + · · · + an zn là đa thức bậc n với hệ số phức


(n ≥ 1), trong đó an 6= 0. Xét hàm hai biến f (z) = |p(z)|. Hàm số này liên tục. Ta
Lời giải và bình luận đề thi các tỉnh, các trường Đại học năm học 2009-2010 117

sẽ chứng minh rằng hàm số này “tăng đến vô cùng”. Thật vậy

n
an−1 a0
f (z) = |an ||z| 1 + +···+ .
an z an zn
an−1 a0 1
Nếu như giá trị |z| đủ lớn thì mô-đun của + ··· + n
nhỏ hơn và nghĩa là
an z an z 2
|an ||z|n
f (z) ≥ , như vậy (với |z| đủ lớn bằng R) f (z) sẽ lớn hơn f (0). Từ đó suy ra
2
rằng giá trị nhỏ nhất của f không thể đạt được bên ngoài đường tròn bán kính R tâm
ở 0 và, hơn thế, không thể đạt được ở ngoài hình vuông bất kì chứa đường tròn này.
Nhưng theo định lý Veierstrass, hàm số liên tục f phải đạt giá trị nhỏ nhất trong
hình vuông này. Giả sử điểm đó là z∗ . Không mất tính tổng quát, có thể giả sử z∗ = 0
(nếu không đổi biến từ z thành z − z∗ ). Như thế, giả sử f đạt giá trị nhỏ nhất tại điểm
0.

Nếu f (0) = 0 thì định lý được chứng minh. Ta chứng minh rằng trường hợp f (0) > 0
không thể xảy ra.

Bổ đề D’Alamber. Giá trị nhỏ nhất của mô-đun một đa thức đại số bậc n ≥ 0, đạt
tại điểm 0 không thể khác 0.

Thật vậy, giả sử ngược lại f (0) = |a0 | > 0 và giả sử k ≥ 1 là chỉ số nhỏ nhất sao cho
ak khác 0. Gọi ξ là một nghiệm của phương trình a0 + ak zk = 0. Đặt tak+1 ξ k+1 +
· · · + t n−k an ξ n = g(t) thì lúc đó
|p(tξ )| = |a0 + ak t k ξ k + ak+1t k+1 ξ k+1 + · · · + ant n ξ n |
= |a0 − t k [a0 + g(t)]| < |a0 | = |p(0)|,
a0
vì với t > 0 đủ nhỏ, |g(t)| < . Mâu thuẫn. Như vậy bổ đề được chứng minh và
2
nghĩa là định lý cơ bản của đại số đã được chứng minh.

Định lý cơ bản của đại số, còn được gọi là định lý Gauss - D’Alamber là một trong
những kết quả quan trọng và nổi tiếng nhất trong toán học. Có rất nhiều cách chứng
minh cho định lý này và trên đây là một trong những cách chứng minh sơ cấp nhất,
thông qua các định lý liên quan đến tính chất của hàm số liên tục, cụ thể là định lý
Cauchy và định lý Veierstrass. Tiếp theo, chúng ta sẽ tiếp tục tìm thấy các ứng dụng
của định lý Veirstrass trong việc chứng minh các kết quả cơ bản khác của giải tính
liên quan đến phép tính vi phân.

Bổ đề Fermat
Bổ đề Fermat cùng với định lý Veierstrass là cơ sở của chuỗi các định lý đẹp đẽ
và sâu sắc liên quan đến đạo hàm và vi phân. Định lý được phát biểu như sau: nếu
118 Trần Nam Dũng (chủ biên)

hàm số f (x) liên tục trên [a, b], khả vi trên khoảng (a, b) và đạt cực trị tại điểm
ξ ∈ (a, b) thì f 0 (ξ ) = 0.

Rõ ràng ta chỉ cần chứng minh cho trường hợp ξ là điểm cực tiểu. Để chứng minh
bổ đề Fermat, ta xét đạo hàm bên trái và đạo hàm bên phải của f tại điểm ξ :
f (x) − f (ξ ) f (x) − f (ξ )
f 0 (ξ + ) = lim , f 0 (ξ − ) = lim .
x→ξ + x−ξ x→ξ − x−ξ

Chú ý rằng, do f đạt cực tiểu tại điểm ξ nên với x đủ gần ξ thì f (x) − f (ξ ) luôn
không âm. Vì vậy giá trị dưới dấu lim ở đẳng thức thứ nhất luôn không âm, còn ở
đẳng thức thứ hai luôn không dương. Vì thế đạo hàm bên phải tại điểm ξ không âm,
còn đạo hàm bên trái tại điểm ξ luôn không dương. Vì f khả vi nên hai đạo hàm
này bằng nhau và vì thế bắt buộc phải bằng 0. Bổ đề Fermat được chứng minh.

Từ kết quả này, ta sẽ lần lượt thu được các định lý Rolle, Langrange và Cauchy dưới
đây.

Các định lý Rolle - Lagrange - Cauchy


Định lý Rolle. Giả sử hàm số f (x) liên tục trên [a, b], khả vi trên (a, b). Ngoài ra,
giả sử rằng f (a) = f (b). Khi đó trên khoảng (a, b) tồn tại điểm ξ sao cho f 0 (ξ ) = 0.

Nói một cách khác, giữa hai giá trị bằng nhau của một hàm khả vi luôn có nghiệm
của đạo hàm hàm số này.

Để chứng minh định lý Rolle, trước hết ta áp dụng định lý Veirestrass cho hàm liên
tục f (x). Hàm số này đạt giá trị lớn nhất M và giá trị nhỏ nhất m trên đoạn [a, b].
Có thể xảy ra hai trường hợp.
(a) M = m. Khi đó f (x) là hàm hằng trên [a, b] và với mọi ξ thuộc (a, b), f 0 (ξ ) =
0.
(b) M > m. Do f (a) = f (b) nên một trong hai giá trị M và m phải đạt được tại
một điểm ξ thuộc (a, b). Nhưng khi đó, hàm số f (x) đạt cực trị tại điểm này
và theo bổ đề Fermat, ta có f 0 (ξ ) = 0.
Như vậy định lý đã được chứng minh.

Từ định lý Rolle, ta suy ra định lý Lagrange, hay tương đương là công thức Lagrange.

Định lý Lagrange. Cho f (x) là hàm liên tục trên [a, b], khả vi trên (a, b). Khi đó
tồn tại ξ thuộc (a, b) sao cho
f (b) − f (a)
f 0 (ξ ) = hay f (b) − f (a) = f 0 (ξ )(b − a).
b−a
Lời giải và bình luận đề thi các tỉnh, các trường Đại học năm học 2009-2010 119

Công thức đầu tiên có một ý nghĩa hình học đơn giản là trên đường cong y = f (x),
giữa hai điểm A(a, f (a)) và B(b, f (b)) có một điểm C sao cho tiếp tuyến của đường
cong tại C song song với dây cung AB.

Công thức ở dạng thứ hai được gọi là công thức Lagrange về số gia hữu hạn. Nó còn
có thể viết dưới dạng
f (x) = f (x0 ) + f 0 (ξ )(x − x0 )
chính là công thức Taylor khai triển đến bậc thấp nhất. Từ đây cũng suy ra công thức
tính gần đúng bằng vi phân:

f (x + ∆x) ∼ f (x) + f 0 (x)∆x.

Để chứng minh định lý Lagrange, ta chỉ cần xét hàm số

f (b) − f (a)
g(x) = f (x) − (x − a),
b−a

rồi áp dụng định lý Rolle cho hàm số này (do g(a) = g(b) = f (a)).

Như vậy, định lý Langrange được chứng minh thông qua định lý Rolle. Mặt khác,
định lý Rolle chính là một trường hợp đặc biệt của định lý Lagrange. Định lý sau
đây mở rộng định lý Lagrange:

Định lý Cauchy. Nếu mỗi một trong hai hàm số f (x) và g(x) đều liên tục trên [a, b],
khả vi trên (a, b) và ngoài ra g0 (x) khác 0 với mọi x thuộc (a, b) thì trên (a, b) tồn
tại điểm ξ sao cho
f (b) − f (a) f 0 (ξ )
= 0 .
b−a g (ξ )
Chúng tôi dành việc chứng minh định lý Cauchy cho bạn đọc. Chú ý là định lý
Lagrange chính là một trường hợp riêng của định lý Cauchy, khi g(x) = x.

Từ các định lý cơ bản trên đây, ta còn suy ra nhiều hệ quả và định lý quan trọng khác
như quy tắc L’Hopitale về khử dạng vô định, công thức Taylor . . .

Cuối cùng chúng ta xem xét những định lý và bài tập có thể giải quyết được bằng
cách áp dụng những định lý này.

Một số định lý và bài tập áp dụng


x2
1. Cho parabol (P) : y = x2 − 2x và ellip (E) : x2/9 + y2 = 1.
9
(1) Chứng minh rằng (P) cắt (E) tại bốn điểm phân biệt A, B, C, D.
120 Trần Nam Dũng (chủ biên)

(2) Chứng minh rằng bốn điểm A, B, C, D cùng nằm trên một đường tròn,
tìm tâm và bán kính của đường tròn đó.
(Đề thi Đại học Ngoại thương 1997)

2. Cho hai đa thức P(x) = 4x3 − 2x2 − 15x + 9 và Q(x) = 12x3 + 6x2 − 7x + 1.
(1) Chứng minh rằng mỗi đa thức đã cho đều có ba nghiệm thực phân
biệt.
(2) Ký hiệu α và β tương ứng là nghiệm lớn nhất của P(x) và Q(x).
Chứng minh rằng α 2 + 3β 2 = 4.
(Đề thi VMO 2003)

3. Các số thực p, q phải thoả mãn điều kiện gì để đa thức x3 + px + q có ba


nghiệm thực phân biệt?

4. Trong mặt phẳng cho ba tia Ox, Oy, Oz và đoạn thẳng có độ dài 2p. Chứng
minh rằng tồn tại duy nhất bộ ba điểm A, B, C tương ứng thuộc Ox, Oy, Oz
sao cho chu vi các tam giác OAB, OBC, OCA đều bằng nhau và bằng 2p.
(Đề thi chọn đội tuyển Việt Nam 1983)
x
5. Phương trình sin x = có bao nhiêu nghiệm thực?
8
6. (Quy tắc Descartes về dấu) Cho P(x) = a0 + a1 x + a2 x2 + · · · + an xn là một
đa thức có hệ số thực. Gọi k là số lần đổi dấu trong dãy các hệ số khác 0 của
P(x) (giữ đúng thứ tự và bỏ các hệ số bằng0).Khi đó số nghiệm dương của
k
đa thức P(x) bằng k − 2s, trong đó 0 ≤ s ≤ . Hãy chứng minh.
2

7. Chứng minh rằng nếu đa thức P(x) với hệ số thực có tất cả các nghiệm đều
thực thì đa thức P(x) + P0 (x) cũng có tất cả các nghiệm đều thực.
1
8. Chứng minh rằng đạo hàm các bậc của hàm số √ chỉ có các nghiệm
1 + x2
thực, hơn nữa là các nghiệm đơn và mỗi nghiệm của đạo hàm bậc n nằm giữa
hai nghiệm của đạo hàm bậc n + 1.

9. Giả sử rằng đa thức bậc bốn P(x) có bốn nghiệm dương. Chứng minh rằng
phương trình
 
1 − 4x 1 − 4x
P(x) + 1 − P0 (x) − P00 (x) = 0
x2 x2

cũng có bốn nghiệm dương.


(Đề thi chọn đội tuyển Việt Nam 1994)
Lời giải và bình luận đề thi các tỉnh, các trường Đại học năm học 2009-2010 121

10. Chứng minh rằng với mọi n nguyên dương và với mọi x, ta có
1 1 1
1 + cos x + cos 2x + cos 3x + · · · + cos nx ≥ 0.
2 3 n

11. (Quy tắc L’Hopitale) Cho hai hàm số f (x) và g(x) xác định và khả vi khắp
nơi trong một lân cận nào đó của điểm a, ngoại trừ có thể là điểm a. Giả sử
rằng
lim f (x) = lim g(x) = 0,
x→a x→a

và đạo hàm g0 (x) khác 0 khắp nơi trong lân cận nói trên của điểm a. Khi đó
f 0 (a) f (a)
nếu tồn tại giới hạn lim 0 , thì cũng tồn tại giới hạn lim , và ta có
x→a g (a) x→a g(a)

f (a) f 0 (a)
lim = lim 0 .
x→a g(a) x→a g (a)

Hãy chứng minh.

12. (Mở rộng định lý Rolle) Cho 0 < a < b. Nếu f (x) bằng 0 tại n + 1 điểm của
đoạn [a, b] và tất cả các nghiệm của đa thức a0 + a1 x + a2 x2 + · · · + an xn đều
thực thì tại một điểm ξ nào đó thuộc (a, b) ta có đẳng thức

a0 f (ξ ) + a1 f 0 (ξ ) + · · · + an f (n) (ξ ) = 0.

13. (Mở rộng công thức Lagrange) Cho hàm số f (x) liên tục và khả vi hai lần tại
lân cận điểm x0 . Chứng minh rằng với mọi x thuộc lân cận này, tồn tại ξ nằm
giữa x0 và x sao cho

f 00 (ξ )(x − x0 )2
f (x) = f (x0 ) + f 0 (ξ )(x − x0 ) + .
2

14. Hàm số f (x) khả vi hai lần trên toàn trục số và bị chặn. Chứng minh rằng tồn
tại điểm x0 sao cho f 00 (x0 ) = 0.

15. Cho f (x) là hàm số liên tục trên R và tuần hoàn với chu kỳ 1, tức là f (x +1) =
f (x) với mọi x. Chứng minh rằng tồn tại số x0 sao cho f (x0 + π) = f (x0 ).

16. Cho hàm số f là hàm số liên tục từ R vào R sao cho | f (x) − f (y)| ≥ |x − y|
với mọi x, y thuộc R. Chứng minh rằng f là toàn ánh.

Phần chính của bài này được viết dựa trên bài báo “Các định lý tồn tại và định lý cơ
bản của đại số” của GS V. Tikhomirov đăng trên tạp chí Kvant, số 4/2005.
122 Trần Nam Dũng (chủ biên)

Tài liệu tham khảo


[1] V. Tikhomirov, Các định lý tồn tại và định lý cơ bản của đại số, Kvant, số
4/2005, trang 2-6 (tiếng Nga).

[2] G. Polya, G. Sege, Các định lý và bài toán của giải tích, Nhà xuất bản Khoa
học, Matcơva 1978 (tiếng Nga).

[3] V. Ilyn, E. Poznyak, Cơ sở giải tích toán học, Nhà xuất bản Khoa học, Matx-
cơva 1998 (tiếng Nga).

[4] Tạp chí Toán Học và Tuổi Trẻ, Các bài thi Olympic Toán Trung học phổ thông
Việt Nam (1990-2006), Nhà xuất bản Giáo dục 2007.

[5] Tạp chí Toán Học và Tuổi Trẻ, The Vietnamese Mathematical Olympiad (1990-
2006), Selected Problems, Nhà xuất bản Giáo dục 2007.

[6] V. Sadovnichi, A. Podkolzin, Các bài toán Olympic sinh viên, Nhà xuất bản
Khoa học, Matxcơva 1978 (tiếng Nga).

[7] Bách khoa toàn thư mở wikipedia.

[8] Paulo Ney de Souza, Jorge-Nuno Silva, Berkeley Problems in Mathematics,


Springer 2001.
Chương 11

Phép chứng minh phản chứng

1Chứng minh phản chứng có thể nói là một trong những vũ khí quan trọng của toán
học. Nó cho phép chúng ta chứng minh sự có thể và không có thể của một tính chất
nào đó, nó cho phép chúng ta biến thuận thành đảo, biến đảo thành thuận, nó cho
phép chúng ta lý luận trên những đối tượng mà không rõ là có tồn tại hay không. Ví
dụ kinh điển nhất về phép chứng minh phản chứng thuộc về Euclid với phép chứng
minh.

Định lý. Tồn tại vô số số nguyên tố.

Ở đây, Euclid đã giả sử ngược lại rằng tồn tại hữu hạn số nguyên tố p1 , p2 , . . . , pn .
Ông xét tích N = p1 p2 · · · pn + 1. N phải có ít nhất một ước số nguyên tố p. Khi đó,
do p1 , p2 , . . . , pn là tất cả các số nguyên tố nên tồn tại i sao cho p = pi . Nhưng khi
đó p | 1, mâu thuẫn.

Bài tập

1. Chứng minh rằng tồn tại vô số số nguyên tố dạng 4k + 3.

2. Chứng minh rằng tồn tại vô số số nguyên tố dạng 4k + 1.

Một chứng minh nổi tiếng khác bằng phương pháp phản chứng chính là chứng minh
của Euler cho định lý nhỏ Fermat với trường hợp n = 4.

Định lý. Phương trình x4 + y4 = z4 (1) không có nghiệm nguyên dương.

Ông đã giả sử rằng phương trình (1) có nghiệm nguyên dương. Khi đó, theo nguyên
lý cực hạn, tồn tại nghiệm (x0 , y0 , z0 ) với x0 + y0 + z0 nhỏ nhất. Sau đó, bằng cách
1 Tríchbài giảng: Các phương pháp và kỹ thuật chứng minh, chương trình Gặp gỡ Toán học 2010
tổ chức tại Đại học Quốc gia thành phố Hồ Chí Minh từ ngày 25-31/1/2010.

123
124 Trần Nam Dũng (chủ biên)

sử dụng cấu trúc nghiệm của phương trình Pythagore x2 + y2 = z2 , ông đi đến sự tồn
tại của một nghiệm (x1 , y1 , z1 ) có x1 + y1 + z1 < x0 + y0 + z0 . Mâu thuẫn.

Phương pháp này thường được gọi là phương pháp xuống thang.

Bài tập

3. Chứng minh rằng phương trình x3 + 3y3 = 9z3 không có nghiệm nguyên
dương.

4. Chứng minh rằng phương trình x2 + y2 + z2 = 2xyz không có nghiệm nguyên


dương.

Chứng minh sử dụng mệnh đề phản đảo cũng là một phương án chứng minh phản
chứng hay được sử dụng. Cơ sở của phương pháp là để chứng minh A → B, ta có thể
chứng minh B̂ → Â. Về mặt bản chất thì hai phép suy diễn này có vẻ giống nhau,
nhưng trong thực tế thì lại khá khác nhau. Ta thử xem xét một vài ví dụ.
x
Ví dụ 11.1. Chứng minh rằng hàm số f (x) = √ là một đơn ánh từ R vào R.
2
x +1

Ví dụ 11.2. Chứng minh rằng nếu (p − 1)! + 1 là số nguyên tố thì p là số nguyên


tố.

Trong ví dụ 11.1, rõ ràng việc chứng minh x1 6= x2 suy ra f (x1 ) 6= f (x2 ) khó khăn
hơn việc chứng minh f (x1 ) = f (x2 ) suy ra x1 = x2 , dù rằng về mặt logic, hai điều
này là tương đương.

Trong ví dụ 11.2, gần như không có cách nào khác ngoài cách chứng minh nếu p là
hợp số, p = rs thì (p − 1)! + 1 không chia hết cho p.

Bài tập

5. Cho hàm số f : R → R thoả mãn các điều kiện sau


(i) f đơn điệu;
(ii) f (x + y) = f (x) + f (y) với mọi x, y thuộc R.
Chứng minh rằng tồn tại số thực a sao cho f (x) = ax với mọi x thuộc R.

6. Cho a, b, c là các số thực không âm thoả mãn điều kiện a2 +b2 +c2 +abc = 4.
Chứng minh rằng
a + b + c ≤ 3.
Lời giải và bình luận đề thi các tỉnh, các trường Đại học năm học 2009-2010 125

Trong việc chứng minh một số tính chất bằng phương pháp phản chứng, ta có thể
có thêm một số thông tin bổ sung quan trọng nếu sử dụng phản ví dụ nhỏ nhất. Ý
tưởng là để chứng minh một tính chất A cho một cấu hình P, ta xét một đặc trưng
f (P) của P là một hàm có giá trị nguyên dương. Bây giờ giả sử tồn tại một cấu hình
P không có tính chất A, khi đó sẽ tồn tại một cấu hình P0 không có tính chất A với
f (P0 ) nhỏ nhất. Ta sẽ tìm cách suy ra điều mâu thuẫn. Lúc này, ngoài việc chúng ta
có cấu hình P0 không có tính chất A, ta còn có mọi cấu hình P với f (P) < f (P0 ) đều
có tính chất A.

Ví dụ 11.3. Cho ngũ giác lồi ABCDE trên mặt phẳng toạ độ có toạ độ các đỉnh đều
nguyên.

(a) Chứng minh rằng tồn tại ít nhất một điểm nằm trong hoặc nằm trên cạnh của
ngũ giác (khác với A, B, C, D, E) có toạ độ nguyên.

(b) Chứng minh rằng tồn tại ít nhất một điểm nằm trong ngũ giác có toạ độ
nguyên.

(c) Các đường chéo của ngũ giác lồi cắt nhau tạo ra một ngũ giác lồi nhỏ
A1 B1C1 D1 E1 bên trong. Chứng minh rằng tồn tại ít nhất một điểm nằm trong
hoặc trên biên ngũ giác lồi A1 B1C1 D1 E1 .

Câu (a) có thể giải quyết dễ dàng nhờ nguyên lý Dirichlet: Vì có năm điểm nên tồn
tại ít nhất hai điểm X, Y mà cặp toạ độ (x, y) của chúng có cùng tính chẵn lẻ (ta chỉ
có bốn trường hợp (chẵn, chẵn), (chẵn, lẻ), (lẻ, chẵn) và (lẻ, lẻ)). Trung điểm Z của
XY chính là điểm cần tìm.

Sang câu (b) lý luận trên đây chưa đủ, vì nếu XY không phải là đường chéo mà là
cạnh thì Z có thể sẽ nằm trên biên. Ta xử lý tình huống này như sau. Để ý rằng
nếu XY là một cạnh, chẳng hạn là cạnh AB thì ZBCDE cũng là một ngũ giác lồi có
các đỉnh có toạ độ đều nguyên và ta có thể lặp lại lý luận nêu trên đối với ngũ giác
ZBCDE, . . . Ta có thể dùng đơn biến để chứng minh quá trình này không thể kéo
dài mãi, và đến một lúc nào đó sẽ có một ngũ giác có điểm nguyên nằm trong.

Tuy nhiên, ta có thể trình bày lại lý luận này một cách gọn gàng như sau: Giả sử
tồn tại một ngũ giác nguyên mà bên trong không chứa một điểm nguyên nào (phản
ví dụ). Trong tất cả các ngũ giác như vậy, chọn ngũ giác ABCDE có diện tích nhỏ
nhất (phản ví dụ nhỏ nhất). Nếu có nhiều ngũ giác như vậy thì ta chọn một trong
số chúng. Theo lý luận đã trình bày ở câu a), tồn tại hai đỉnh X, Y có cặp toạ độ
cùng tính chẵn lẻ. Trung điểm Z của XY sẽ có toạ độ nguyên. Vì bên trong ngũ giác
ABCDE không có điểm nguyên nào nên XY phải là một cạnh nào đó. Không mất
tính tổng quát, giả sử đó là AB. Khi đó ngũ giác ZBCDE có toạ độ các đỉnh đều
nguyên và có diện tích nhỏ hơn diện tích ngũ giác ABCDE. Do tính nhỏ nhất của
126 Trần Nam Dũng (chủ biên)

ABCDE (phản ví dụ nhỏ nhất phát huy tác dụng!) nên bên trong ngũ giác ZBCDE
có một điểm nguyên T. Điều này mâu thuẫn vì T cũng nằm trong ngũ giác ABCDE.

Bài tập

7. Giải phần (c) của ví dụ 11.3.

8. Chứng minh rằng nếu (a, b) = 1 thì tồn tại u, v sao cho au + bv = 1.
(Định lý Bezout)

Phương pháp phản chứng thường hay được sử dụng trong các bài toán bất biến hoặc
bài toán phủ hình để chứng minh sự không thực hiện được. Sau đây chúng ta xem
xét hai ví dụ như vậy.

Ví dụ 11.4. Xét hình vuông 7 × 7 ô. Chứng minh rằng ta có thể xoá đi một ô để phần
còn lại không thể phủ kín bằng 15 quân trimino kích thước 1 × 3 và 1 quân trimino
hình chữ L.
1
Ví dụ 11.5. Cho trước các hàm số f1 (x) = x2 + 2x, f2 (x) = x + , f3 (x) = x2 − 2x.
x
Cho phép thực hiện các phép toán cộng hai hàm số, nhân hai hàm số, nhân một hàm
số với một hằng số tuỳ ý. Các phép toán này có thể tiếp tục được thực hiện nhiều lần
1
trên fi và trên các kết quả thu được. Chứng minh rằng có thể thu được hàm số từ
x
các hàm số f1 , f2 , f3 bằng các sử dụng các phép toán trên nhưng điều này không
thể thực hiện được nếu thiếu một trong ba hàm f1 , f2 , f3 .

Bài tập

9. Hình vuông 5 × 5 bỏ đi ô trung tâm. Chứng minh rằng có thể phủ phần còn
lại bằng tám quân trimino 1 × 3 nhưng không thể phủ được bằng tám quân
trimino hình chữ L. Tìm tất cả các giá trị k sao cho có thể phủ phần còn lại
bằng k quân trimino 1 × 3 và 8 − k trimino hình chữ L.

10. Trên vòng tròn ban đầu theo một thứ tự tuỳ ý có bốn số 1 và năm số 0. Ở
khoảng giữa hai chữ số giống nhau ta viết số 1 và ở khoảng giữa hai chữ số
khác nhau ta viết số 0. Các số ban đầu bị xoá đi. Hỏi sau một số lần thực hiện
như vậy ta có thể thu được một bộ gồm chín số 0?
Chương 12

Nguyên lý Dirichlet

12 Nguyên lý Dirichlet ở dạng cổ điển thường được dùng để chứng minh tồn tại theo
kiểu không xây dựng (non-constructive), tức là biết đối tượng tồn tại nhưng không
chỉ ra cụ thể.

Nguyên lý Dirichlet trong số học


Trong số học, nguyên lý Dirichlet thường liên quan đến các bài toán chia hết, nguyên
tố cùng nhau. Ví dụ các bài toán kinh điển sau.

Ví dụ 12.1. Chọn ra n + 1 số từ 2n số nguyên dương đầu tiên.

(a) Chứng minh rằng trong các số được chọn, có hai số phân biệt x, y nguyên tố
cùng nhau.

(b) Chứng minh rằng trong các số được chọn, có hai số x > y mà x chia hết cho
y.

Ví dụ 12.2. Chứng minh rằng từ n số nguyên bất kỳ luôn có thể chọn ra một số hoặc
một số số có tổng chia hết cho n.

Ví dụ 12.3. Chứng minh rằng nếu p là số nguyên tố dạng 4k + 1 thì tồn tại các số
nguyên a, b sao cho p = a2 + b2 .
(Định lý Fermat-Euler về tổng hai bình phương)

Chứng minh. Vì p có dạng 4k +1 nên theo kết quả của định lý ở phần đầu, tồn tại số
nguyên N sao cho N 2 + 1 chia hết cho p, hay nói cách khác, N 2 ≡ −1 (mod p). Xét
 √  √  2
các số dạng x +Ny với x, y là các số nguyên thuộc 0, p . Có tất cả p +1
1 Bài viết được viết bởi TS. Trần Nam Dũng.
2 Trích bài giảng: Các phương pháp và kỹ thuật chứng minh, trình bày tại chương trình Gặp gỡ toán

học 2010 do Đại học Quốc gia thành phố Hồ Chí Minh tổ chức từ ngày 25/1-31/1/2010.

127
128 Trần Nam Dũng (chủ biên)

√  2
số như vậy. Vì p + 1 > p nên theo nguyên lý Dirichlet, tồn tại hai cặp số
(x, y) 6= (x0 , y0 ) sao cho x + Ny ≡ x0 + Ny0 (mod p). Từ đây suy ra

x − x0 ≡ N(y0 − y) (mod p),

dẫn tới
(x − x0 )2 ≡ N 2 (y0 − y)2 (mod p).
Bây giờ, nhớ lại rằng N 2 ≡ −1(mod p), ta suy ra

(x − x0 )2 ≡ −(y0 − y)2 (mod p),

hay
(x − x0 )2 + (y0 − y)2 ≡ 0 (mod p).
Cuối cùng, chú ý rằng 0 < (x − x0 )2 + (y0 − y)2 < 2p ta suy ra điều phải chứng minh.

Ngoài kỹ thuật kinh điển với chuồng và thỏ, ta có thể sử dụng một biến thể của
nguyên lý Dirichlet như sau:

Tính chất. Nếu A, B là các tập hợp thoả mãn điều kiện |A| + |B| > |A ∪ B| thì
A ∩ B 6= 0.

Sau đây là một áp dụng của tính chất này.


Ví dụ 12.4. Chứng minh rằng nếu p là số nguyên tố dạng 4k + 3 thì tồn tại các số
nguyên x, y sao cho x2 + y2 + 1 chia hết cho p.
p−1
Chứng minh. Đặt ri = i2 mod p với i = 1, 2, . . . , và si = −1 − i2 mod p,
2
p−1
i = 1, 2, . . . , thì dễ dàng chứng minh được rằng ri đôi một phân biệt và si đôi
2
một phân biệt. Hơn nữa, ri và si đều thuộc {1, 2, . . . , p − 1}.
n o n o p−1
Đặt A = r1 , . . . , r p−1 , B = s1 , . . . , s p−1 thì |A| = |B| = và
2 2 2
|A ∪ B| ≤ p − 1.

Xảy ra hai trường hợp.

Trường hợp 1. Nếu |A ∪ B| < p − 1 thì theo tính chất nên trên, ta có A ∩ B 6= ∅, tức
là tồn tại i, j sao cho ri = s j , tương đương với i2 ≡ −1 − j2 (mod p), hay i2 + j2 + 1
chia hết cho p.

Trường hợp 2. Nếu |A ∪ B| = p − 1 thì A ∩ B = ∅ và như vậy, các số r1 , r2 , . . . , r p−1 ,


2
s1 , s2 , . . . , s p−1 đôi một phân biệt và ta có
2

r1 + r2 + · · · + r p−1 + s1 + s2 + · · · + s p−1 = 1 + 2 + · · · + p − 1 ≡ 0 (mod p).


2 2
Lời giải và bình luận đề thi các tỉnh, các trường Đại học năm học 2009-2010 129

Điều này mâu thuẫn vì theo định nghĩa của ri và si , ta có

r1 + r2 + · · · + r p−1 + s1 + s2 + · · · + s p−1 ≡
2 2
 2  2 !
p − 1 p − 1
≡ 12 + 22 + · · · + + (−1 − 12 ) + · · · + −1 −
2 2
p−1
≡− (mod p).
2
Vậy trường hợp 2 không xảy ra, và như thế ta rơi vào trường hợp 1. Ta có điều phải
chứng minh.

Ghi chú. Lý luận A ∨ B và B ⇒ A được gọi là Tam đoạn luận rời.

Bài tập
1. Xét dãy số Fibonacci xác định bởi F1 = F2 = 1, Fn+1 = Fn + Fn−1 với mọi
n ≥ 2. Chứng minh rằng với mọi số nguyên dương m > 1, tồn tại vô số số
hạng của dãy số chia hết cho m.

2. Từ khoảng (22n , 23n ) chọn ra 22n−1 + 1 số lẻ. Chứng minh rằng trong các số
được chọn, tồn tại hai số mà bình phương mỗi số không chia hết cho số còn
lại.

3. Chứng minh rằng


(a) Không tồn tại số nguyên dương n sao cho 10n + 1 chia hết cho 2003.
(b) Tồn tại các số nguyên dương m, n sao cho 10m + 10n + 1 chia hết cho
2003.

4. Dãy số nguyên dương a1 , a2 , . . . , an , . . . thoả mãn điều kiện

1 ≤ an+1 − an ≤ 2001

với mọi n = 1, 2, 3, . . . Chứng minh rằng tồn tại vô số cặp số p, q sao cho
q > p và aq chia hết cho a p .
(Vietnam TST 2001)

Nguyên lý Dirichlet trong đại số


Trong đại số nguyên lý Dirichlet được thể hiện qua tính chất cơ bản sau: Nếu trên
đoạn [a, b] có n số thực x1 , x2 , . . . , xn (n ≥ 2) thì tồn tại các chỉ số i 6= j sao cho
b−a
|xi − x j | ≤ .
n−1
130 Trần Nam Dũng (chủ biên)

Ví dụ 12.5. Chứng minh rằng giữa bảy số thực bất kỳ luôn tìm được hai số x và y
sao cho
x−y 1
0< ≤√ .
1 + xy 3
Chứng minh. Gọi  các số đã cho là a1 , a2 , . . . , a7 . Với mỗi số thực a, tồn tại số
π π
α thuộc khoảng − , sao cho a = tan α. Giả sử a1 = tan α1 , a2 = tan α2 , . . . ,
2 2
a7 = tan α7 . Theo tính chất nêu trên, trong bảy số α1 , α2 , . . . , α7 tồn tại hai số có
π
hiệu không vượt quá . Giả sử hai số này là α và β , trong đó α > β . Khi đó
6
tan α − tan β π 1
0< = tan(α − β ) ≤ tan = √ .
1 + tan α tan β 6 3
Như vậy các số x = tan α và y = tan β là các số cần tìm.

Định lý Kronecker về sự trù mật là một định lý có nhiều ứng dụng trong giải tích,
đại số, giải tích phức. Dưới đây ta xét chứng minh rất sơ cấp của định lý này (ở dạng
tương đương).

Định lý Kronecker. Nếu α là số vô tỷ thì tập hợp S = {{nα} | n ∈ N∗ } trù mật trong
[0, 1].

Chứng minh. Ta cần chứng minh rằng với mọi khoảng (a, b) ⊂ [0, 1], tồn tại số
nguyên dương n sao cho {nα} ∈ (a, b).

1
Trước hết, ta tìm số nguyên dương N sao cho N > . Bây giờ xét N + 1 số {α},
b−a
{2α}, . . . , {(N + 1)α} thuộc đoạn [0, 1]. Theo nguyên lý Dirichlet, tồn tại hai số
1
{pα}, {qα} với 1 ≤ p < q ≤ N + 1 sao cho |{pα} − {qα}| ≤ .
N

Để ý rằng {qα} − {pα} = qα − [qα] − pα + [pα] = (q − p)α − [qα] + [pα]. Do đó


nếu {qα} − {pα} > 0 thì {(q − p)α} = {qα} − {pα}, còn nếu {qα} − {pα} < 0
thì {(q − p)α} = 1 − ({qα} − {pα}).

Ta xét hai trường hợp.

1
Trường hợp 1. > {qα} − {pα} > 0. Khi đó đặt k = q − p, ta tìm được số nguyên
N
1
dương k sao cho 0 < {kα} < .
N
Bây giờ ta gọi m là số nguyên dương nhỏ nhất sao cho m{kα} > a thì ta có

(m − 1){kα} ≤ a.
Lời giải và bình luận đề thi các tỉnh, các trường Đại học năm học 2009-2010 131

1
Do đó m{kα} ≤ a + {kα} < a + < b. Suy ra m{kα} ∈ (a, b) ⊂ [0, 1].
N
Nhưng mkα = m[kα] + m{kα}. Do 0 < m{kα} < 1 nên từ đây ta có {mkα} =
m{kα}. Đặt n = mk, ta có {nα} ∈ (a, b).
1
Trường hợp 2. 0 > {qα}−{pα} > − . Khi đó đặt k = q− p, ta tìm được số nguyên
N
1 1
dương k sao cho 1 − < {kα} < 1. Đặt β = 1 − {kα} thì 0 < β < . Chứng minh
N N
tương tự như trên, ta tìm được số nguyên dương m sao cho mβ thuộc (1 − b, 1 − a),
tức là
1 − b < m(1 − {kα}) < 1 − a.
Từ đây ta có
a < m{kα} − m + 1 < b,
hay
a < {m{kα}} < b.
Cuối cùng, ta có mkα = m[kα] + m{kα} nên {mkα} = {m{kα}} nên đặt n = mk ta
có điều phải chứng minh.

Một tình huống rất đơn giản khác của nguyên lý Diriclet lại có những ứng dụng rất
hiệu quả trong nhiều bài toán chứng minh bất đẳng thức, đặc biệt là các bất đẳng
thức có điều kiện. Đó là chú ý sau: Với m là một số thực cho trước và n ≥ 3 số thực
a1 , a2 , . . . , an bất kỳ thì luôn tìm được hai số trong các số này nằm cùng một phía
đối với m.

Gọi hai số đó là x và y thì ta có bất đẳng thức hiển nhiên sau: (x − m)(y − m) ≥ 0, từ
đó xy + m2 ≥ m(x + y). Như vậy, ta đã so sánh được hai đại lượng không cùng bậc
với nhau. Sau đây chúng ta sẽ xem xét một số ví dụ áp dụng.
Ví dụ 12.6. Cho x, y, z là các số thực dương thoả mãn điều kiện x + y + z + 1 = 4xyz.
Chứng minh rằng
xy + yz + zx ≥ x + y + z.
Chứng minh. Ý tưởng đầu tiên ta nghĩ đến là xử lý điều kiện bằng phép thế
x+y+1
z=
4xy − 1
vào bất đẳng thức cần chứng minh, viết lại thành
x+y+1
(x + y − 1) ≥ x + y − xy.
4xy − 1
Đến đây, đường lối chứng minh đã hình thành. Vế trái rõ ràng ≥ 1 với mọi x, y còn
vế phải sẽ ≤ 1 nếu x, y nằm cùng phía nhau đối với 1. Nhưng điều này, theo nhận
xét đơn giản ở trên ta luôn có thể chọn được.
132 Trần Nam Dũng (chủ biên)

Ví dụ 12.7. Cho a, b, c là các số thực không âm. Chứng minh rằng

(a2 + 2)(b2 + 2)(c2 + 2) ≥ 3(a + b + c)2 .

Chứng minh. Áp dụng bất đẳng thức Cauchy-Schwarz, ta có

(a + b + c)2 ≤ (a2 + 1 + 1)(1 + b2 + c2 ) = (a2 + 2)(b2 + c2 + 1).

Như vậy ta chỉ còn cần chứng minh

(b2 + 2)(c2 + 2) ≥ 3(b2 + c2 + 1),

hay
(b2 − 1)(c2 − 1) ≥ 0.
Điều này luôn có được nếu ta chọn b2 , c2 cùng phía nhau đối với 1.

Bài tập
1 1 1
5. Cho a, b, c > 0, x = a + , y = b + , z = c + . Chứng minh rằng
b c a
xy + yz + zx ≥ 2(x + y + z).

6. Cho a, b, c là các số thực không âm thoả mãn điều kiện a2 +b2 +c2 +abc = 4.
Chứng minh rằng
0 ≤ ab + bc + ca − abc ≤ 2.
(USAMO 2001)

7. Với i = 1, 2, . . . , 7 các số ai , bi là các số thực không âm thoả mãn điều kiện


ai + bi ≤ 2. Chứng minh rằng tồn tại các chỉ số i 6= j sao cho

|ai − a j | + |bi − b j | ≤ 1.

8. Cho x, y, z là các số thực dương thoả mãn điều kiện xy + yz + zx + xyz = 4.


Chứng minh rằng
x + y + z ≥ xy + yz + zx.
(VMO 1996)

9. Tìm số thực k nhỏ nhất sao cho

xyz + 2 + k[(x − 1)2 + (y − 1)2 + (z − 1)2 ] ≥ x + y + z

với mọi x, y, z > 0.


Lời giải và bình luận đề thi các tỉnh, các trường Đại học năm học 2009-2010 133

Nguyên lý Dirichlet trong tổ hợp


Tổ hợp là mảnh đất màu mỡ nhất cho các phương pháp và kỹ thuật chứng minh. Và
nguyên lý Dirichlet không phải là một ngoại lệ. Trong tổ hợp, một đặc điểm đặc
trưng là sự bùng nổ tổ hợp của các trường hợp, vì vậy, nguyên lý Dirichlet cùng với
các nguyên lý khác như nguyên lý cực hạn, nguyên lý bất biến chính là những công
cụ quan trọng để chúng ta định hướng trong “biển” các trường hợp.

Nguyên lý Dirichlet thường được sử dụng trong các bài toán đồ thị, tô màu, các bài
toán về thi đấu thể thao (đồ thị có hướng), quen nhau (đồ thị vô hướng).

Ví dụ 12.8. Trong một giải bóng chuyền có tám đội tham gia, thi đấu vòng tròn một
lượt. Chứng minh rằng tìm được bốn đội A, B, C, D sao cho A thắng B, C, D, B
thắng C, D và C thắng D.
Chứng minh. Trong bóng chuyền không có hoà, do đó tám đội thi đấu vòng tròn
một lượt thì sẽ có tất cả 28 trận thắng. Theo nguyên lý Dirichlet, tồn tại đội bóng A
có ít nhất bốn trận thắng. Xét bốn đội thua A, bốn đội này đấu với nhau sáu trận, do
đó tồn tại một đội thắng ít nhất hai trận (trong số các trận đấu giữa bốn đội này với
nhau). Giả sử đó là B và C, D là hai đội thua B. Cuối cùng, nếu C thắng D thì A, B,
C, D là bốn đội cần tìm, còn nếu D thắng C thì bốn đội cần tìm là A, B, D, C.

Bài toán Ramsey là một trong những bài toán kinh điển mà những trường hợp cơ sở
của nó rất thú vị và phù hợp với mức độ toán sơ cấp.
Ví dụ 12.9. Chứng minh rằng trong một nhóm sáu người bất kỳ có ba người đôi
một quen nhau hoặc ba người đôi một không quen nhau.
Ví dụ 12.10. Trong một nhóm gồm 2n + 1 người với mỗi n người tồn tại một người
khác n người này quen với tất cả họ. Chứng minh rằng trong nhóm người này có
một người quen với tất cả mọi người.
Chứng minh. Ta chứng minh rằng trong nhóm người này có n + 1 người đôi một
quen nhau. Rõ ràng có hai người quan nhau và nếu như có k người đôi một quen
nhau (trong đó k ≤ n) thì tồn tại một người khác trong số họ quen với k người này.
Từ đó suy ra tồn tại n + 1 người đôi một quen nhau A1 , A2 , . . . , An+1 .

Xét n người còn lại. Theo điều kiện, tồn tại một người Ai quen với tất cả n người
này. Nhưng khi đó Ai quen với tất cả mọi người.

Bí quyết thành công của nguyên lý Dirichlet chính là kỹ thuật “xây chuồng” và “tạo
thỏ”. Trong nhiều bài toán, chuồng là gì, thỏ là gì khá rõ ràng, nhưng trong nhiều
bài toán, xây chuồng và tạo thỏ là cả một sự tinh tế. Ta phải biết “chọn các thành
phần chính” và “hướng đến mục tiêu”.
134 Trần Nam Dũng (chủ biên)

Ví dụ 12.11. Các số từ 1 đến 200 được chia thành 50 tập hợp. Chứng minh rằng
trong một các tập hợp đó có ba số là độ dài ba cạnh của một tam giác.
Chứng minh. Thoạt nhìn bài toán có vẻ khá rối. Nhưng nếu để ý rằng với 0 < a <
b < c thì điều kiện cần và đủ để a, b, c là độ dài ba cạnh của một tam giác là a+b > c
thì bài toán trở nên đơn giản hơn. Rõ ràng nếu chỉ xét các số từ 100 đến 200 thì ba
số bất kỳ đều là độ dài ba cạnh của một tam giác (a + b ≥ 100 + 101 = 201 > c). Từ
đó chỉ cần xét 101 con thỏ là các số từ 100 đến 200 rồi áp dụng nguyên lý Dirichlet
cho 50 cái chuồng tập hợp là xong. Ở đây, rõ ràng các số từ 1 đến 99 chỉ có tác dụng
gây nhiễu.
Ví dụ 12.12. Trên bàn cờ quốc tế có tám quân xe, đôi một không ăn nhau. Chứng
minh rằng trong các khoảng cách đôi một giữa các quân xe, có hai khoảng cách
bằng nhau. Khoảng cách giữa hai quân xe bằng khoảng cách giữa tâm các ô vuông
mà quân các quân xe đứng.
Chứng minh. Trước hết ta mô hình hoá bài toán. Để ý rằng khoảng cách giữa ô
(p, q) và ô (m, n) bằng (p − m)2 + (q − n)2 . Ta cần chứng minh rằng nếu p1 , p2 ,
. . . , p8 là một hoán vị của (1, 2, 3, . . . , 8) thì tồn tại các tập chỉ số {m, n} 6= {p, q}
sao cho
(m − n)2 + (pm − pn )2 = (p − q)2 + (p p − pq )2 .
Tám quân xe tạo ra 28 khoảng cách. Nhưng nếu ta tìm hai khoảng cách bằng nhau
giữa cả 28 quân xe này thì ta sẽ gặp khó khăn. Ta giới hạn trong việc tìm các cặp
chỉ số dạng {n, n + 1}. Có bảy cặp như vậy. Khi đó, ta chỉ cần tìm n 6= m sao cho
(pn+1 − pn )2 = (pm+1 − pm )2 . Vì 1 ≤ pi ≤ 8 nêu (pn+1 − pn )2 chỉ có thể là một
trong bảy giá trị 12, 22, . . . , 72. Vì thế chỉ có thể xảy ra hai trường hợp.

Trường hợp 1. Tồn tại n 6= m sao cho (pn+1 − pn )2 = (pm+1 − pm )2 . Khi đó các cặp
quân xe tại ô (n, pn ), (n + 1, pn+1 ) và (m, pm ), (m + 1, pm+1 ) là các cặp xe cần tìm.

Trường hợp 2. Các số (pn+1 − pn )2 đôi một phân biệt. Khi đó tồn tại n sao cho
(pn+1 − pn )2 = 4. Lúc này, xoay hàng thành cột, ta lại đi đến việc hoặc tồn tại n 6= m
sao cho (qn+1 − qn )2 = (qm+1 − qm )2 hoặc tồn tại k sao cho (qk+1 − qk )2 = 22 . Trong
trường hợp thứ nhất, bài toán được giải quyết tương tự như trường hợp 1 ở trên. Trong
trường hợp thứ hai, các quân xe tại ô (n, pn ), (n + 1, pn+1 ) và (qk+1 , k + 1), (qk , k)
là các cặp xe cần tìm.

Bài tập
10. Các số 1, 2, 3, . . . , 100 có thể là thành viên của 12 cấp số nhân nào đó được
không?
11. Trong một đa giác lồi có chứa không ít hơn m2 + 1 điểm nguyên. Chứng minh
rằng trong đa giác lồi này tìm được m + 1 điểm nguyên cùng nằm trên một
đường thẳng.
Lời giải và bình luận đề thi các tỉnh, các trường Đại học năm học 2009-2010 135

12. Chứng minh rằng trong chín người bất kỳ, hoặc có ba người đôi một quen
nhau, hoặc có bốn người đôi một không quen nhau.
13. Chọn ra 69 số nguyên dương từ tập hợp E = {1, 2, . . . , 100}. Chứng minh
rằng tồn tại bốn số a < b < c < d trong các số được chọn sao cho a+b+c = d.
Kết luận bài toán còn đúng không nếu ta thay 69 bằng 68?
14. Các ô vuông của bảng 100 × 100 được tô bằng bốn màu sao cho trên mỗi hàng
và trên mỗi cột có đúng 25 ô có cùng một màu. Chứng minh rằng tồn tại hai
dòng và hai cột sao cho bốn ô nằm ở giao của chúng được tô khác màu.

Nguyên lý Dirichlet trong hình học


Trong hình học, nguyên lý Dirichlet thường được sử dụng trong các bài toán liên
quan đến độ dài cạnh, diện tích, độ lớn của góc, các bài toán trên lưới nguyên. Ở
đây chúng tôi chỉ giới hạn trong việc giới thiệu một ứng dụng đẹp của nguyên lý
Dirichlet về “chồng hình” trong hình học và một số bài tập.

Định lý Minkowsky là một ví dụ rất thú vị về ứng dụng của hình học trong lý thuyết
số. Chúng ta bắt đầu từ một kết quả rất đơn giản nhưng hữu ích.

Bổ đề 1. Trên mặt phẳng cho hình F có diện tích lớn hơn 1. Khi đó tồn tại hai điểm


A, B thuộc F, sao cho véc-tơ AB có tọa độ nguyên.

Chứng minh. Lưới nguyên cắt hình G thành các mẩu nhỏ. Chồng các mẩu này lên
nhau, do tổng diện tích của các mẩu lớn hơn 1, nên có ít nhất hai mẩu có điểm chung
(đây chính là một biến thể của nguyên lý Dirichlet). Gọi A, B là hai điểm nguyên
thuỷ ứng với điểm chung này thì A, B là hai điểm cần tìm.

Bổ đề 2. (Bổ đề Minkowsky) Trên mặt phẳng cho hình lồi F nhận gốc tọa độ làm
tâm đối xứng và có diện tích lớn hơn 4. Khi đó nó chứa một điểm nguyên khác gốc
tọa độ.
1
Chứng minh. Xét phép vị tự tâm O, tỷ số , biến F thành G. Do G có diện tích
2 −

lớn hơn 1 nên theo bổ đề 1, tồn tại hai điểm A, B thuộc G sao cho véc-tơ AB có toạ
độ nguyên. Gọi A0 là điểm đối xứng với A qua O. Do hình G đối xứng qua gốc toạ
độ nên A0 thuộc G. Do G lồi nên trung điểm M của A0 B thuộc G. Gọi N là điểm đối
xứng của O qua M thì N thuộc F và ON = AB, suy ra N là điểm nguyên khác O. Bổ
đề được chứng minh.

Định lý 3. (Định lý Minkowsky) Cho a, b, c là các số nguyên, trong đó a > 0 và


ac − b2 = 1. Khi đó phương trình ax2 + 2bxy + cy2 = 1 có nghiệm nguyên.
136 Trần Nam Dũng (chủ biên)

Bài tập

15. Với giá trị nào của n tồn tại n điểm M1 , M2 , . . . , Mn sao cho tất cả các góc
Mi M j Mk đều không tù?

16. Cho chín điểm nằm trong hình vuông cạnh 1. Chứng minh rằng tồn tại một
1
tam giác có đỉnh tại các điểm đã cho có diện tích không vượt quá .
8
Chương 13

Cauchy-Bunyakovski-Schwarz
Inequality

12 Together with Arthimetic mean – Geometric mean (AM-GM), Schur, Jensen and
Holder inequalities, Cauchy-Bunyakovski-Schwarz inequality (CBS) is a fundamen-
tal result, with remarkable applications. The main question is how do we recognise
an inequality that can be solved using this method? It is very hard to say this clearly,
but it is definitely good to think of CBS inequality whenever we have sums of radi-
cals or sums of squares and especially when we have expressions involving radicals.

Let us first consider some problems in which it is better to apply the direct form of
CBS inequality
!2 ! !
n n n
∑ ai bi ≤ ∑ a2i ∑ b2i .
i=1 i=1 i=1
The main difficulty is to choose ai and bi . We will see that in some cases this is
trivial, while in the other cases it is very difficult. Let us solve some problems now:

Ví dụ 13.1. Prove that if x, y, z are real numbers such that x2 + y2 + z2 = 2, then the
following inequality holds
x + y + z ≤ 2 + xyz.
(IMO Shorlist, proposed by Poland)
Proof. Why do we shoud think of CBS inequality? The reason: the relation we are
asked to prove can be written as x(1 − yz) + y + z ≤ 2 and we are bound to consider
the sum x2 + y2 + z2 . However, there are lots of ways to apply CBS. The choice
[x(1 − yz) + y + z]2 ≤ [x2 + y2 + z2 ][2 + (1 − yz)2 ]
1 Bài viết được viết bởi TS. Trần Nam Dũng và Gabriel Dospinescu.
2 Bài viết này được trích từ nguyên bản tiếng Anh trên tạp chí Toán học và tuổi trẻ.

137
138 Trần Nam Dũng (chủ biên)

does not help. So, maybe it is better to look at y + z as a single number. Observe that
we have the equality when x = 1, y = 1 and z = 0 (for example), the choice
q
x(1 − yz) + y + z ≤ [x2 + (y + z)2 ][1 + (1 − yz)2 ]

becomes natural. So, we must prove that

2(1 + yz)(2 − 2yz + y2 z2 ) ≤ 4,

or
y3 z3 ≤ y2 z2 ,
which is easy, since 2 ≥ y2 + z2 ≥ 2yz.

Another non-trivial application of CBS inequality is the following problem.


Ví dụ 13.2. Let a, b, c, x, y, z be positive real numbers such that ax + by + cz = xyz.
Prove that √ √ √
x + y + z > a + b + b + c + c + a.
a b c
Proof. We write + + = 1 and now the substitution a = yzu, b = zxv and
yz zx xy
c = xyw becomes natural. So, we must prove that
p p p
z(yu + xv) + x(zv + yw) + y(zu + xw) < x + y + z

for u + v + w = 1.

One can see the form of the CBS inequality


hp p p i2
z(yu + xv) + x(zv + yw) + y(zu + xw) ≤
≤ (x + y + z)(yu + xv + zv + yw + zu + xw),

and the latter is of course smaller than (x + y + z)2 , since u + v + w = 1.

We have seen that CBS inequality can be applied when we have sums. What about
products? The following example will show that we need some imagination in this
case:

Ví dụ 13.3. Let n (n ≥ 2) be an integer and let a1 , a2 , . . . , an be positive real


numbers. Prove the inequality

(a31 + 1)(a32 + 1) · · · (a3n + 1) ≥ (a21 a2 + 1) · · · (a2n a1 + 1).

(Czech – Slovak – Polish Match, 2001)


Lời giải và bình luận đề thi các tỉnh, các trường Đại học năm học 2009-2010 139

Proof. We try to apply CBS inequality for each factor of the product in the RHS.
It is natural to write (1 + a21 a2 )2 ≤ (1 + a31 )(1 + a1 a22 ), since we need 1 + a31 , which
appears in the LHS. Similarly, we can write

(1 + a22 a3 )2 ≤ (1 + a32 )(1 + a2 a23 ), ..., (1 + a2n a1 )2 ≤ (1 + a3n )(1 + an a21 ).

Multiplying we obtain

[(a21 a2 + 1) · · · (a2n a1 + 1)]2 ≤ [(a31 + 1) · · · (a3n + 1)][(1 + a1 a22 ) · · · (1 + an a21 )]. (∗)

Well, it seems that CBS inequality does not work for this one. False! We use again
the same argument to find that

[(1 + a1 a22 ) · · · (1 + an a21 )]2 ≤ [(a31 + 1) · · · (a3n + 1)][(a21 a2 + 1) · · · (a2n a1 + 1)]. (∗∗)

Thus, if (a21 a2 + 1) · · · (a2n a1 + 1) ≥ (1 + a1 a22 ) · · · (1 + an a21 ), then (∗) will give the
answer, otherwise (∗∗) will.

It is now time to solve some harder problems.

Ví dụ 13.4. Given x > 0, y > 0 such that x2 + y3 ≥ x3 + y4 . Prove that

x3 + y3 ≤ 2.

(Russia, 1999)
Proof. The idea is to majorize x3 + y3 with A(x3 + y4 ) for a certain A, which seems
reasonable, looking at the exponents. We can try some tricks with CBS and AM-
GM:
 2 2
3 3 2 3 4 3 2 2 3 3 2 x + y2 + x3 + y3
(x + y ) ≤ (x + y )(x + y ) ≤ (x + y )(x + y ) ≤ .
2

Thus we have established that x3 + y3 ≤ x2 + y2 . However

(x2 + y2 )2 ≤ (x + y)(x3 + y3 ),

and so q
x 2 + y2 ≤ x + y ≤ 2(x2 + y2 ).

Hence x2 + y2 ≤ 2 and consequently x3 + y3 ≤ 2.

Ví dụ 13.5. Prove that if x, y, z ∈ [−1, 1] satisfying x + y + z + xyz = 0, then


√ p √
x + 1 + y + 1 + z + 1 ≤ 3.
140 Trần Nam Dũng (chủ biên)

Proof. We first try CBS inequality in the obvious form:


√ p √ p
x + 1 + y + 1 + z + 1 ≤ 3(x + y + z + 3).

But is the RHS smaller than 3? Well, if x + y + z ≤ 0, it is. Let us suppose it is not
the case. Thus xyz < 0. Let z < 0. It follows that x, y ∈ (0, 1]. We will not give up
and try to use again CBS inequality, but for the first two radicals:
√ p √ p √
x + 1 + y + 1 + z + 1 ≤ 2x + 2y + 4 + z + 1.

We have to prove that p √


2x + 2y + 4 + z + 1 ≤ 3,
which is equivalent to

2(x + y) −z
√ ≤ √ ,
2 + 2x + 2y + 4 1 + z + 1

−2z(1 + xy) −z
√ ≤ √ ,
2 + 2x + 2y + 4 1 + z + 1
√ p
2xy + 2(1 + xy) z + 1 ≤ 2x + 2y + 4.
(1 − x)(1 − y)
Since 1 + z = , everything comes down to proving that
1 + xy
r
p x+y
xy + (1 − x)(1 − y)(1 + xy) ≤ 1 + .
2
We would like to use CBS inequality such that 1 − x vanishes from the LHS. Specif-
ically:
p √ p √ p
xy + (1 − x)(1 − y)(1 + xy) = x · xy2 + 1 − x · 1 + xy − y − xy2
r
p x+y
≤ 1 + xy − y ≤ 1 ≤ 1 + .
2

Maybe the hardest example of them all is the following problem:

Ví dụ 13.6. Prove that for all positive real numbers a, b, c, x, y, z, we have

a b c xy + yz + zx
(y + z) + (z + x) + (x + y) ≥ 3 · .
b+c c+a a+b x+y+z

(Walther Janous, Crux Mathematicorum)


Lời giải và bình luận đề thi các tỉnh, các trường Đại học năm học 2009-2010 141

Proof. We have
 
a y+z
∑ b + c (y + z) + ∑(y + z) = ∑a ∑ b+c
 
1  y+z
= ∑(b + c) ∑
2 b+c
1 √  2
≥ ∑ y+z .
2
We will show that
1 √ 2 3 ∑ yz
∑ y+z ≥ +2∑x (1)
2 ∑x
from which our result will follow.

(1) is equivalent to
h  p i  2
x + x 2 + xy + yz + zx ≥ 3 ∑ yz + 2 .
∑ ∑x ∑x
We have q q 2
2
∑ x + (xy + yz + zx) ≥ ∑x + 9 ∑ yz.

Hence, it is enough to show that

q 2 2
∑x ∑x + 9 ∑ yz ≥ 3 ∑ yz + 2 ∑x ,

which becomes obvious when we square both sides.

We will now take a look at most used trick in past year constest problems. It is a
direct variant of CBS inequality:
!2
n

n ∑ ak
a2 k=1
∑ bkk ≥ n ,
k=1
∑ bk
k=1

for all real numbers ak and positive numbers bk (k = 1, 2, . . . , n).

An easy application of this trick is the following problem given at the Tournament
of The Towns competition.
142 Trần Nam Dũng (chủ biên)

Ví dụ 13.7. Prove that for all positive real numbers a, b, c, we have the following
inequality

a3 b3 c3 a2 + b2 + c2
+ + ≥ .
a2 + ab + b2 b2 + bc + c2 c2 + ca + a2 a+b+c

(a2 + b2 + c2 )2
Proof. If we write the RHS as , we will know what we
(a + b + c)(a2 + b2 + c2 )
have to do
2
a3 (a2 )2 ∑ a2
∑ a2 + ab + b2 ∑ a(a2 + ab + b2 ) ≥
= .
∑ a(a2 + ab + b2 )
So, we will be able to prove the inequality if

∑ a(a2 + ab + b2 ) ≤ (a + b + c)(a2 + b2 + c2 ),
which is in fact verified with equality occur.

There are cases when it is impossible to find ai and bi . Let us discuss some problems
in which it is not easy at all to use the trick.

Ví dụ 13.8. Prove that for all positive real numbers a, b, c, we have the inequality

(b + c − a)2 (c + a − b)2 (a + b − c)2 3


2 2
+ 2 2
+ 2 2
≥ .
a + (b + c) b + (c + a) c + (a + b) 5
(Japan, 1997)

Proof. The most natural way would be:


 2  2
b+c b+c
−1 ∑ a −3
(b + c − a)2 a
∑ a2 + (b + c)2 = ∑  b + c 2 ≥  b + c 2
+1 ∑ +3
a a

because this way we obtain a nice inequality in three variables, whose properties are
b+c c+a a+b
well-known. Thus, we have to show that if x = ,y= ,z= , then
a b 2
3
(x + y + z − 3)2 ≥ (x2 + y2 + z2 + 3),
5
which is equivalent to
2
∑x − 15 ∑ x + 3 ∑ xy + 18 ≥ 0.
Lời giải và bình luận đề thi các tỉnh, các trường Đại học năm học 2009-2010 143

Unfortunately, we cannot use directly the fact that xy + yz + zx ≥ 12. So, we should
look for some thing like xy + yz + zx ≥ k(x + y + z). The best would be k = 2 (so as
to have an equality when x = y = z = 2). Indeed, after some computations this can
be written as
∑ a3 + 3abc ≥ ∑ ab(a + b),
which is Schur inequality. Hence, we can write
2 2
∑x − 15 ∑ x + 3 ∑ xy + 18 ≥ ∑x − 9 ∑ x + 18 ≥ 0,

the last one being obvious since x + y + z ≥ 6.

At IMO 2001, problem 2 was a challenge for the contestants. Here we suggest an
approach, which leads to a nice generalization.

Ví dụ 13.9. Given positive real number k ≥ 8. Show that for positive real numbers
a, b, c, we have

a b c 3
√ +√ +√ ≥√ .
2
a + kbc 2
b + kca 2
c + kab k+1

(Generalization of IMO 2001)

Proof. We have, by CBS inequality


 
a p 
2 + kbc ≥
2

∑ a2 + kbc ∑ a a ∑a .

Now, apply CBS inequality again for the second sum, we have
 p 2  √ p 
∑ a ∑(a3 + kabc)
 
a a 2 + kbc = a · a 3 + kabc ≤ .
∑ ∑
All we have to do now is to show that
3
(k + 1) ∑a ≥ 9 ∑(a3 + kabc).

But it is equivalent to

(k − 8)(a3 + b3 + c3 ) + 3(k + 1)(a + b)(b + c)(c + a) ≥ 27kabc,

which is obvious by AM-GM inequality.


144 Trần Nam Dũng (chủ biên)

Practice problems
1 1 1
1. Given x, y, z > 1 such that + + = 2. Prove that
x y z
√ √ p √
x + y + z ≥ x − 1 + y − 1 + z − 1.

(Iran, 1998)
1 √
 
2. Prove that if a1 , a2 , . . . , a6 ∈ √ , 3 , then we have
3
a1 − a2 a2 − a3 a3 − a4 a4 − a5 a5 − a6 a6 − a1
+ + + + + ≥ 0.
a2 + a3 a3 + a4 a4 + a5 a5 + a6 a6 + a1 a1 + a2
(Vasile Cirtoaje)

3. Given x ∈ [0, 1]. Show that


 p p 
x 13 1 − x2 + 9 1 + x2 ≤ 16.

(Olympiad of 30 April of Vietnam, 1996)

4. Prove that for 2n arbitrary real numbers a1 , a2 , . . . , an and x1 , x2 , . . . , xn , we


have v
n
u n ! n ! ! !
u
2 2 2 n n
∑ ai xi + ∑ ai ∑ xi ≥ n ∑ ai ∑ xi .
t
i=1 i=1 i=1 i=1 i=1

When does equality hold?


(Kvant 1989)

5. Given real numbers a, b, c, x, y, z such that (a + b + c)(x + y + z) = 3 and


(a2 + b2 + c2 )(x2 + y2 + z2 ) = 4. Prove that

ax + by + cz ≥ 0.

(Mathlinks contest 2005)


Phụ lục A

Đề luyện đội tuyển cho kỳ thi


VMO 2010
1

Đề số 1
Bài 1. Với những giá trị nào của b thì tồn tại a sao cho hệ phương trình
(
(x − 1)2 + (y + 1)2 = b
y = x2 + (2a + 1)x + a2

có nghiệm?

Bài 2. Cho dãy số {xn } xác định bởi

xn2 + 3
x1 = 3, xn+1 = ∀n = 1, 2, . . .
3xn

Chứng minh rằng dãy {xn } có giới hạn hữu hạn và tìm giới hạn đó.

Bài 3. Trên các cạnh AB và BC của tam giác ABC lấy các điểm M và N tương ứng.
Chứng minh rằng bốn điểm A, M, N và C nằm trên một đường tròn khi và chỉ khi
giao điểm các đoạn thẳng AN và CM nằm trên đoạn thẳng đi qua trực tâm các tam
giác ABC và BMN.

Bài 4. Các số nguyên dương a, b, c, d thoả mãn điều kiện

a2 + b2 + ab = c2 + d 2 + cd.
1 Các đề toán được biên soạn bởi TS. Trần Nam Dũng.

145
146 Trần Nam Dũng (chủ biên)

Chứng minh rằng số a + b + c + d là hợp số.

Bài 5. Xét một dãy số gồm các số 0 và các số 1. Xét các cặp số trong dãy này (không
nhất thiết kề nhau), trong đó số bên trái là số 1 và số bên phải là số 0. Giả sử trong
số các cặp này có đúng M cặp mà giữa số 1 và số 0 của cặp này có một số chẵn các
số và có đúng N cặp mà giữa số 1 và số 0 của cặp này có một số lẻ số. Chứng minh
rằng M lớn hơn hay bằng N. (Ví dụ với dãy số 1 1 0 1 0 0 thì M = 4, N = 4; với dãy
số 1 0 0 0 thì M = 2, N = 1).

Đề số 2
Bài 1. Cho x, y, z là các số thực dương thoả mãn điều kiện x + y + z + 1 = 4xyz.
Chứng minh rằng ta có bất đẳng thức

xy + yz + zx ≥ x + y + z.

Bài 2.

(a) Chứng minh rằng với mọi n ≥ 2, phương trình xn − xn−1 − · · · − x − 1 = 0 có


một nghiệm dương duy nhất.

(b) Ký hiệu nghiệm dương nói trên là xn . Hãy tìm lim xn .


n→∞

Bài 3. Đường tròn nội tiếp tam giác ABC tiếp xúc với các cạnh AB, BC và AC tại các
điểm K, L và M tương ứng. Kẻ các tiếp tuyến chung ngoài khác với các cạnh của
tam giác tới các đường tròn nội tiếp các tam giác BKL, CLM và AKM. Chứng minh
rằng các tiếp tuyến này đồng quy tại một điểm.

a2 b + b
Bài 4. Tìm tất cả các cặp số nguyên dương (a, b) sao cho số là số nguyên.
ab2 + 4

Bài 5. Cho tập hợp S = {1, 2, 3, . . . , n}. Tìm số cách chia tập S thành ba tập con
khác rỗng sao cho mỗi tập con không chứa hai số nguyên liên tiếp.

Đề số 3
Bài 1. Tìm tất cả các hàm số f : R → R thoả mãn điều kiện

f (x2 − y2 ) = x f (x) − y f (y)

với mọi x, y ∈ R.
Lời giải và bình luận đề thi các tỉnh, các trường Đại học năm học 2009-2010 147

Bài 2. Cho x, y, z là các số thực dương thoả mãn điều kiện


 
1 1 1
(x + y + z) + + = 10.
x y z

Tìm giá trị lớn nhất và giá trị nhỏ nhất của
 
3 3 3 1 1 1
(x + y + z ) 3 + 3 + 3 .
x y z

Bài 3. Cho tam giác ABC, K và L là hai điểm trên (AB), (AC) sao cho BK = CL. Gọi
P là giao điểm của CK và BL. Đường thẳng qua P song song với phân giác trong
của góc ∠BAC cắt AC tại M. Chứng minh rằng CM = AB.

Bài 4. Tìm tất cả các cặp số nguyên dương x, y sao cho (x2 + y)(y2 + x) là luỹ thừa
5 của một số nguyên tố.

Bài 5. Có 2009 tấm bìa trên đó có ghi các số từ 1 đến 2009, mỗi số trên một bìa.
Đầu tiên các tấm bìa được xếp thành một hàng theo một thứ tự nào đó. Người ta
thực hiện liên tiếp các phép biến đổi sau: đầu tiên nhìn vào tấm bìa đầu tiên bên
tay trái, nếu trên đó có ghi số k thì ta đảo k tấm bìa đầu tiên theo thứ tự ngược lại,
còn các tấm bìa khác thì để nguyên. Ví dụ nếu ta xét chín số từ 1 đến 9 thay vì
từ 1 đến 2009 và thứ tự ban đầu là 395672814 thì dãy các biến đổi sẽ lần lượt là
395672814 → 593672814 (đảo thứ tự ba số đầu) → 763952814 (đảo thứ tự năm số
đầu) → 825936714 (đảo thứ tự bảy số đầu) → 17639528 (đảo thứ tự tám số đầu).

Chứng minh rằng sau một số hữu hạn lần thực hiện, tấm bìa với số 1 sẽ được đưa
lên đầu và như vậy từ đó thứ tự của các tấm bìa sẽ không thay đổi nữa.

Đề số 4
Bài 1. Giải hệ phương trình
 2 2
 (2x − y) = 4 + z

(z − y)2 = 2 + 4x2 .

(z + 2x)2 = 3 + y2

Bài 2. Đa thức P(x) bậc n > 1 có n nghiệm phân biệt x1 , x2 , . . . , xn . Đạo hàm P0 (x)
có các nghiệm y1 , y2 , . . . , yn−1 . Chứng minh bất đẳng thức

x12 + x22 + · · · + xn2 y21 + y22 + · · · + y2n−1


> .
n n−1
148 Trần Nam Dũng (chủ biên)

Bài 3. Trên đường thẳng d cho ba điểm A, B, C theo thứ tự đó. Về một phía với d,
vẽ các nửa đường tròn (S1 ), (S2 ) đường kính AB, AC tương ứng. D là điểm trên S2
sao cho tam giác DBC cân tại D. Gọi O là tâm đường tròn tiếp xúc với (S1 ), (S2 ) và
BD. Chứng minh rằng OB vuông góc với d.

Bài 4. Cho n số x1 , x2 , . . . , xn có tích là p. Biết rằng với mọi i = 1, 2, . . . , n, số p − xi


là một số nguyên lẻ. Chứng minh rằng tất cả các số x1 , x2 , . . . , xn đều là số vô tỷ.

Bài 5. Cho bàn cờ 13 × 13 ô bị khuyết một ô ở giữa. Hỏi có thể phủ bàn cờ này bằng
các quân tetramino kích thước 1 × 4 hoặc 4 × 1 được hay không?

Đề số 5
Bài 1. Trên bảng ghi năm số. Cộng các số này theo từng cặp, ta được 10 số sau đây:
0, 2, 4, 4, 6, 8, 9, 11, 13, 15. Hỏi các số đã ghi trên bảng là những số nào?

Bài 2. Hàm số liên tục f : R → R thoả mãn điều kiện

g(x, y) = | f (x + y) − f (x) − f (y)|

bị chặn. Chứng minh rằng tồn tại số thực A sao cho hàm số h(x) = | f (x) − Ax| cũng
bị chặn.

Bài 3. Cho tam giác nhọn ABC. Trên các cạnh AC nối dài về C, CB nối dài về phía
B, BA nối dài về phía A lấy các điểm B1 , A1 , C1 sao cho tam giác A1 B1C1 đồng dạng
với tam giác ABC. Chứng minh rằng trực tâm tam giác A1 B1C1 trùng với tâm đường
tròn ngọai tiếp tam giác ABC.

Bài 4. Tìm tất cả các đa thức f (x) với hệ số nguyên sao cho với mọi n nguyên dương
ta có f (n) là ước của 2n − 1.

Bài 5. Có bao nhiêu số có n chữ số lập từ các chữ số {1, 2, 3, . . . , 9} có tích các
chữ số chia hết cho 10?

Đề số 6
Bài 1. Cho a, b, c là các số thực dương thoả mãn điều kiện a + b + c = 3. Chứng
minh rằng
1 1 1 3
2 2
+ 2 2
+ 2 2
≤ .
a +b +2 b +c +2 c +a +2 4
Bài 2. Tìm tất cả các giá trị của tham số a sao cho phương trình

log7 (7x − log7 a) = 2x


Lời giải và bình luận đề thi các tỉnh, các trường Đại học năm học 2009-2010 149

có nghiệm duy nhất.

Bài 3. Trong mặt phẳng, cho đường tròn (O) tâm O và đường tròn (O0 ) tâm O0 cắt
nhau tại hai điểm B, M. Các điểm A, C nằm trên đường tròn (O). Đường thẳng AB
cắt (O0 ) tại K khác B. Đường thẳng BC cắt đường tròn (O0 ) tại N khác B. Các đường
trung trực của các đoạn thẳng AK và CN cắt nhau tại I (khác M). Chứng minh rằng

∠IMB = 90◦ .

Bài 4. Trong cấp số cộng a1 , a2 , a3 , a4 , . . . chứa các số a21 , a22 và a23 . Chứng minh
rằng cấp số trên gồm các số hạng nguyên.

Bài 5. Trong một quốc gia có một số thành phố được nối với nhau bởi các con đường.
Các con đường chỉ cắt nhau tại các thành phố. Tại mỗi thành phố có treo một tấm
bảng trên đó ghi độ dài ngắn nhất của đường đi, xuất phát từ thành phố này và đi
qua tất cả các thành phố (đường đi có thể đi qua một số thành phố một vài lần và
không nhất thiết phải quay về thành phố xuất phát). Chứng minh rằng hai số bất kỳ
trên các bảng chênh lệch nhau không quá 1.5 lần.

Đề số 7
Bài 1. Tìm tất cả các hàm số f : R → R thoả mãn điều kiện

(i) f (x2 ) = f 2 (x) với mọi x thuộc R;

(ii) f (x + 1) = f (x) + 1 với mọi x thuộc R.



Bài 2. Cho a, b, c, x, y, z là các số thực thoả mãn điều kiện (a + b)z − (x + y)c = 6.
Tìm giá trị nhỏ nhất của biểu thức

P = a2 + b2 + c2 + x2 + y2 + z2 + ax + by + cz.

Bài 3. Trên các cạnh AB và AD của hình vuông ABCD lấy các điểm K và N tương
ứng sao cho AK · AN = 2BK · DN. Các đoạn thẳng CK và CN cắt đường chéo BD tại
các điểm L và M. Chứng minh rằng các điểm K, L, M, N và A nằm trên một đường
tròn.

Bài 4.

(a) Chứng minh rằng phương trình x2 + 5 = y3 không có nghiệm nguyên.

(b) Tìm tất cả các nghiệm nguyên dương của phương trình x2 + 2 = y3 .

Bài 5. Chọn một cách ngẫu nhiên 55 số từ 100 số nguyên dương đầu tiên. Chứng
minh rằng trong số các số được chọn tìm được hai số có hiệu bằng 9.
150 Trần Nam Dũng (chủ biên)
Phụ lục B

Hướng dẫn nội dung bồi dưỡng


học sinh thi chọn học sinh giỏi
Toán Quốc gia lớp 12 THPT

I. Yêu cầu tối thiểu về kiến thức

Ngoài các kiến thức toán theo Chương trình phổ thông (từ lớp 1 đến lớp 12) hiện
hành, các học sinh dự thi cần được trang bị thêm tối thiểu một số kiến thức sau:

1. Phần Số học

– Các khái niệm và kết quả lý thuyết được trình bày trong Chương I; §1, §2, §4
Chương II; §1, §2, §3 Chương III; Chương IV và Chương V cuốn “Bài giảng
số học” của nhóm Tác giả: Đặng Hùng Thắng (Chủ biên), Nguyễn Văn Ngọc,
Vũ Kim Thuỷ (NXB Giáo dục, 1994).

– Định lý nhỏ Fermat, Định lý Wilson.

– Định lý Euler và định lý Trung Quốc về các số dư.

1 Kèmtheo các Công văn số 11636/THPT ngày 25/12/2000 và 1403/THPT ngày 25/02/2002 của
Bộ Giáo dục và Đào tạo.

151
152 Trần Nam Dũng (chủ biên)

2. Phần Đại số - Giải tích


a. Bất đẳng thức
– Các bất đẳng thức đại số: bất đẳng thức Cauchy cho n (n ∈ Z, n ≥ 2) số thực
không âm; bất đẳng thức Bunyakowski cho hai bộ n số thực (n ∈ Z, n ≥ 2);
bất đẳng thức Chebyshev cho hai dãy n số thực (n ∈ Z, n ≥ 2); bất đẳng thức
Nesbitt cho ba số thực dương; bất đẳng thức Bernoulli mở rộng.

– Bất đẳng thức hàm lồi (Bất đẳng thức Jensen).

– Các bất đẳng thức tích phân được trình bày trong mục 3 của §2 Chương III
SGK Giải tích 12 (Sách chỉnh lý hợp nhất năm 2000, NXB Giáo dục).

– Kết quả của Ví dụ 1.4 trong §1 Chương V cuốn “Bất đẳng thức” của Tác giả
Phan Đức Chính (NXB Giáo dục, 1993).

b. Đa thức
– Khái niệm nghiệm bội của đa thức và một số kết quả đơn giản liên quan đến
nghiệm của một đa thức:
+ Định lý 1. Đa thức bậc n (n ∈ N∗ ) có tối đa n nghiệm thực, mỗi nghiệm
được kể số lần bằng số bội của nó.
+ Định lý 2. Nếu x0 là nghiệm của đa thức P(x) thì x0 + a là nghiệm của
đa thức P(x − a), với a ∈ R cho trước.
+ Định lý 3. Nếu x0 6= 0 là nghiệm của đa thức

P(x) = a0 xn + a1 xn−1 + · · · + an−1 x + an , a0 6= 0 và n ∈ N∗ ,


1
thì là nghiệm của đa thức
x0

Q(x) = an xn + an−1 xn−1 + · · · + a1 x + a0 .

+ Định lý 4. Nếu x0 là nghiệm bội k (k ∈ Z, k ≥ 2) của đa thức P(x) thì


x0 là nghiệm bội k − 1 của đa thức đạo hàm P0 (x).
+ Định lý 5. Nếu x0 là nghiệm hữu tỉ của đa thức với hệ số nguyên

P(x) = a0 xn + a1 xn−1 + · · · + an−1 x + an , a0 6= 0 và n ∈ N∗ ,


p
thì x0 phải có dạng ; trong đó p, q tương ứng là ước của an , a0 .
q
+ Định lý Viette thuận và đảo cho đa thức bậc n (n ∈ Z, n ≥ 2).

– Công thức nội suy Lagrange.


Lời giải và bình luận đề thi các tỉnh, các trường Đại học năm học 2009-2010 153

– Khái niệm đa thức khả quy, bất khả quy.

– Định lý Bezout về số dư trong phép chia một đa thức cho nhị thức bậc nhất
x − a.

– Đa thức Chebyshev và các tính chất được trình bày trong phần 1 Phụ lục 3
cuốn “Bất đẳng thức” của Tác giả Phan Đức Chính (NXB Giáo dục, 1993).

c. Dãy số - Hàm số
– Phương trình đặc trưng và công thức tính số hạng tổng quát của dãy số được
cho bởi hệ thức truy hồi tuyến tính.

– Các khái niệm: dãy con, dãy số tuần hoàn và chu kỳ của dãy số tuần hoàn.

– Mối liên hệ giữa tính hội tụ của một dãy số và tính hội tụ của các dãy con của
dãy số đó.

– Một số kết quả đơn giản về tính đơn điệu của hàm số:
+ Kết quả 1. Nếu f và g là các hàm số đồng biến (nghịch biến) trên tập
X thì f + g cũng là hàm số đồng biến (nghịch biến) trên tập X.
+ Kết quả 2. Giả sử f và g là các hàm số đồng biến (nghịch biến) trên tập
X. Khi đó:
(i) Nếu f và g chỉ nhận giá trị không âm (không dương) trên X thì f · g
sẽ là hàm số đồng biến trên tập X.
(ii) Nếu f và g chỉ nhận giá trị không dương (không âm) trên X thì f · g
sẽ là hàm số nghịch biến trên tập X.
+ Kết quả 3. Giả sử f là hàm số đồng biến và g là hàm số nghịch biến
trên tập X. Khi đó, nếu f chỉ nhận giá trị không âm (không dương) trên X và
đồng thời g chỉ nhận giá trị không dương (không âm) trên tập đó thì f · g sẽ là
hàm số nghịch biến (đồng biến) trên X.
+ Kết quả 4. Giả sử g là hàm số đồng biến (nghịch biến) trên tập X. Kí
hiệu g(X) là tập giá trị của hàm g với tập xác định X. Khi đó:
(i) Nếu f là hàm số đồng biến trên g(X)thì f (g(x)) sẽ là hàm số đồng
biến (nghịch biến) trên X.
(ii) Nếu f là hàm số nghịch biến trên g(X) thì f (g(x)) sẽ là hàm số nghịch
biến (đồng biến) trên X.
+ Kết quả 5. Nếu f là hàm số đồng biến trên R thì hai phương trình sau
sẽ tương đương với nhau:

f ( f (. . . ( f (x)) . . .)) = x và f (x) = x.


154 Trần Nam Dũng (chủ biên)

– Khái niệm chu kỳ cơ sở của hàm số tuần hoàn và một số kết quả liên quan đến
hàm tuần hoàn:
+ Định lý 6. Nếu hàm số f (x) tuần hoàn trên tập X với chu kỳ cơ sở T và
nếu
f (x) = f (x + A) ∀x ∈ X

thì phải có A = kT, với k ∈ Z.


+ Định lý 7. Nếu hàm số tuần hoàn f (x) có chu kỳ cơ sở T thì hàm số
T
f (ax) (a 6= 0) là hàm số tuần hoàn và có chu kỳ cơ sở .
a
+ Định lý 8. Nếu các hàm số f1 (x), f2 (x) tuần hoàn trên X và tương
ứng có các chu kỳ T1 , T2 thông ước với nhau thì các hàm số f1 (x) + f2 (x),
f1 (x) − f2 (x), f1 (x) f2 (x) cũng tuần hoàn trên X.

– Định nghĩa hàm số ngược.

– Định lý về giá trị trung gian của hàm số liên tục trên một đoạn.

– Kết quả các Bài toán 1-7 trong §1 Chương II cuốn “Phương trình hàm” của
Tác giả Nguyễn Văn Mậu (NXB Giáo dục, 1997).

3. Phần Lượng giác


– Hệ thức Charles cho các cung lượng giác.

– Bất phương trình lượng giác và tập nghiệm của các bất phương trình lượng
giác cơ bản.

– Các công thức đơn giản tính độ dài đường phân giác, bán kính đường tròn nội
tiếp, bán kính đường tròn bàng tiếp của một tam giác theo độ dài các cạnh và
giá trị lượng giác của các góc của tam giác ấy.

– Một số bất đẳng thức thông dụng trong tam giác:



3 3
• sin A + sin B + sinC ≤ ∀∆ABC.
2
3
• cos A + cos B + cosC ≤ ∀∆ABC.
2

• tan A + tan B + tanC ≥ 3 3 ∀∆ nhọn ABC.
Dấu đẳng thức trong các bất đẳng thức trên xảy ra khi và chỉ khi ∆ABC là tam
giác đều.
Lời giải và bình luận đề thi các tỉnh, các trường Đại học năm học 2009-2010 155

4. Phần Hình học


a. Hình học phẳng

– Khái niệm trọng tâm, tâm tỉ cự của một hệ điểm và toạ độ của chúng xét trong
hệ toạ độ Decartes.

– Tâm đẳng phương của ba đường tròn.

– Hàng điểm điều hoà và Chùm điều hoà: Định nghĩa và một số tính chất đơn
giản:
+ Hệ thức Newton, Hệ thức Decartes.
+ Định lý 9. Hai cạnh của một tam giác cùng các đường phân giác trong,
ngoài xuất phát từ đỉnh chung của hai cạnh ấy lập thành một chùm điều hoà.

– Định lý Ptolemy, Định lý Ceva, Định lý Menelauyt, Định lý Thales thuận và


đảo.

– Định nghĩa đường tròn Apoloniut, đường tròn Euler (đường tròn 9 điểm).

– Kết quả của các Ví dụ 1, 2 trong phần 4 §4 Chương II SGK Hình học 10 (Sách
chỉnh lý hợp nhất năm 2000, NXB Giáo dục).

– Hệ thức Euler trong tam giác:

d 2 = R2 − 2Rr,

trong đó d, R, r tương ứng là khoảng cách giữa tâm đường tròn ngoại tiếp và
tâm đường tròn nội tiếp, bán kính đường tròn ngoại tiếp, bán kính đường tròn
nội tiếp của một tam giác.

– Định nghĩa tích các phép biến hình và một số kết quả liên quan, định nghĩa
và các tính chất của phép đồng dạng: như đã được trình bày trong TLGKTĐ
Hình học lớp 11 Ban KHTN-THCB (NXB Giáo dục, 1997).

– Các kết quả lý thuyết liên quan tới các phép biến hình trong mặt phẳng được
trình bày trong cuốn “Các bài toán về hình học phẳng” (T.1 và T.2) của Tác
giả Praxolov V.V. (NXB Hải Phòng, 1994).

– Định nghĩa và các tính chất của phép nghịch đảo được trình bày trong phần
“Các kiến thức cơ bản” Chương 28 cuốn “Các bài toán về hình học phẳng”
(T.2) của Tác giả V.V. Praxolov (NXB Hải Phòng, 1994).
156 Trần Nam Dũng (chủ biên)

b. Hình học không gian


– Định lý Thales thuận và đảo.

– Định nghĩa khối đa diện đều, khối tứ diện gần đều, khối tứ diện trực tâm và
một số kết quả liên quan:
+ Định lý 10. Tứ diện ABCD là tứ diện gần đều khi và chỉ khi xảy ra ít
nhất một trong các điều sau:
(i) Các mặt của tứ diện có diện tích bằng nhau.
(ii) Bốn đường cao của tứ diện có độ dài bằng nhau.
(iii) Có ít nhất hai trong ba điểm sau trùng nhau: tâm mặt cầu nội tiếp,
tâm mặt cầu ngoại tiếp và trọng tâm của tứ diện.
+ Định lý 11. Tứ diện ABCD là tứ diện trực tâm khi và chỉ khi xảy ra ít
nhất một trong các điều sau:
(i) Các cặp cạnh đối của tứ diện vuông góc với nhau.
(ii) Chân đường vuông góc hạ từ một đỉnh xuống mặt đối diện là trực tâm
của mặt ấy.
(iii) Tổng bình phương độ dài của các cặp cạnh đối bằng nhau.

– Định lý về sự tồn tại của mặt cầu ngoại tiếp khối đa diện.

– Kết quả của Ví dụ 1 trong §1 Chương II SGK Hình học 12 (Sách chỉnh lý hợp
nhất năm 2000, NXB Giáo dục).

– Khái niệm trọng tâm, tâm tỉ cự của một hệ điểm và toạ độ của chúng xét trong
hệ toạ độ Decartes.

– Định nghĩa và tính chất của tích có hướng của hai vectơ, tích hỗn tạp của ba
vectơ cùng một số kết quả liên quan: như đã được trình bày trong §3 và §8
Chương II SGK Hình học 12 (Sách chỉnh lý hợp nhất năm 2000, NXB Giáo
dục).

5. Phần Tổ hợp
– Nguyên lý Dirichlet, Nguyên lý cực hạn (hay Nguyên lý khởi đầu cực trị).

– Định nghĩa ánh xạ, đơn ánh, toàn ánh, song ánh, ánh xạ tích.

– Các khái niệm và kết quả được trình bày trong §1, §2 và §3 của tài liệu “Về
một số vấn đề của giải tích tổ hợp trong chương trình THPT” (Biên soạn:
Nguyễn Khắc Minh. Tài liệu báo cáo tại Hội nghị tập huấn giáo viên giảng
dạy chuyên toán toàn quốc, Hà Nội-1997).
Lời giải và bình luận đề thi các tỉnh, các trường Đại học năm học 2009-2010 157

– Kết quả của các Bài toán 1, 4, 5 trong §4 của bài viết nói trên.

– Các khái niệm cơ bản của lý thuyết Graph: Graph; Đỉnh, đỉnh cô lập, cạnh
vô hướng, cạnh có hướng của Graph; Graph có hướng; Graph đơn vô hướng
hữu hạn; Graph đầy đủ; Graph bù; Graph con; Bậc của đỉnh trong Graph đơn
vô hướng hữu hạn; Graph thuần nhất; Đường đi, độ dài đường đi, đường đi
khép kín, xích (có tài liệu gọi là đường đi đơn giản), xích đơn, chu trình (có
tài liệu gọi là chu trình đơn giản), chu trình đơn, đường đi Euler, đường đi
Hamilton, chu trình Euler, chu trình Hamilton trong Graph đơn vô hướng hữu
hạn; Graph liên thông, cây, Graph Euler, Graph Hamilton; Thành phần liên
thông của Graph đơn vô hướng hữu hạn.

– Một số kết quả đơn giản của lý thuyết Graph:


+ Định lý 12. Số đỉnh bậc lẻ trong một Graph đơn vô hướng hữu hạn là
một số chẵn.
+ Định lý 13. Trong Graph đơn vô hướng n đỉnh (n ∈ Z, n ≥ 2) tồn tại ít
nhất hai đỉnh có cùng bậc.
+ Định lý 14. Nếu Graph G đơn vô hướng n đỉnh (n ∈ Z, n ≥ 2) có đúng
hai đỉnh cùng bậc thì G phải có đúng một đỉnh bậc 0 hoặc đúng một đỉnh bậc
n − 1.
+ Định lý 15. Mỗi Graph đơn vô hướng hữu hạn không liên thông đều bị
phân chia một cách duy nhất thành các thành phần liên thông.
+ Định lý 16. Nếu mỗi đỉnh của Graph G đơn vô hướng n đỉnh (n ∈ Z, n ≥
n
2) đều có bậc không nhỏ hơn thì G là Graph liên thông.
2
+ Định lý 17. Graph G đơn vô hướng hữu hạn là Graph Euler khi và chỉ
khi hai điều kiện sau được đồng thời thoả mãn:
(i) G là Graph liên thông.
(ii) Mọi đỉnh của G đều có bậc chẵn.
+ Định lý 18. Nếu tất cả các cạnh của một Graph đơn vô hướng đầy đủ
sáu đỉnh được tô bởi hai màu thì phải tồn tại ít nhất một chu trình đơn độ dài
3 có tất cả các cạnh cùng màu.

– Khái niệm “chiến lược thắng cuộc” trong các bài toán trò chơi.

II. Yêu cầu tối thiểu về kỹ năng


1. Biết vận dụng linh hoạt các kiến thức lý thuyết vào việc giải quyết các bài
toán cụ thể.
158 Trần Nam Dũng (chủ biên)

2. Biết phân tích một cách hợp lý các giả thiết để từ đó tìm ra hướng giải quyết
bài toán.

3. Đặc biệt, đối với các bài toán có thể giải được nhờ mô hình graph, cần:
– Biết cách chuyển bài toán ban đầu (hoặc một phần của bài toán ban
đầu) về bài toán tương đương trên mô hình graph;
– Biết cách sử dụng biểu diễn hình học của graph như một công cụ tạo ra
các gợi ý trực giác trong quá trình suy luận, tìm tòi lời giải cho bài toán;
– Biết sử dụng các khái niệm, thuật ngữ của lý thuyết graph để trình bày
lời giải một cách ngắn gọn, sáng sủa, chặt chẽ và chính xác.

III. Một số điểm lưu ý


1. Công thức nội suy Lagrange, Định lý Euler và định lý Trung Quốc về các số
dư, phép biến hình nghịch đảo trong mặt phẳng, các khái niệm, kết quả của
lý thuyết graph và các bài toán trò chơi là nội dung kiến thức không bắt buộc
đối với các học sinh dự thi ở Bảng B.

2. Về định lý Euler và định lý Trung Quốc: Chỉ yêu cầu học sinh hiểu đúng các
định lý này và biết vận dụng chúng trong các tình huống không phức tạp.

3. Học sinh dự thi (ở cả hai bảng A và B) được phép sử dụng các khái niệm, kết
quả đã nêu trong mục I như khái niệm, kết quả SGK.

4. Trong kì thi chọn học sinh giỏi quốc gia môn toán lớp 12 THPT các thí sinh
được phép sử dụng các kiến thức về số phức trong phạm vi chương trình môn
toán Ban KHTN-THCB (cũ).

5. Để học sinh đạt được các yêu cầu về kỹ năng, như đã nêu ở mục II, các giáo
viên bồi dưỡng cần chú ý:
– Giúp học sinh hiểu rõ bản chất toán học của các khái niệm, các kết quả.
– Giúp học sinh nắm được các ý tưởng toán học ẩn chứa trong lời giải
của các bài toán cụ thể.
– Luyện tập cho học sinh các bài toán mà lời giải của chúng thể hiện mối
liên quan giữa các phần kiến thức.
– Phân tích cho học sinh thấy con đường đi đến lời giải của các bài toán.
Điều này đặc biệt quan trọng trong việc luyện tập các bài toán tổ hợp.

6. Để nâng cao hiệu quả của việc bồi dưỡng, ngoài các tài liệu đã dẫn ở mục I,
các giáo viên bồi dưỡng có thể tham khảo các tài liệu sau:
[1] Đề thi vô địch các nước (19 nước) T.1, T.2, T.3, NXB Hải Phòng.
Lời giải và bình luận đề thi các tỉnh, các trường Đại học năm học 2009-2010 159

[2] H. Chúng, Graph và giải toán phổ thông, NXB Giáo dục, 1992.
[3] N. V. Mậu, Phương pháp giải phương trình và bất phương trình, NXB
Giáo dục.
[4] Tài liệu bồi dưỡng học sinh giỏi bậc THPT môn toán, Vụ THPT ấn
hành, 1997.
[5] Tạp chí Toán học và Tuổi trẻ.

You might also like